Sie sind auf Seite 1von 174

MATH2039 Analysis

1
0
0
1
0
1 f (b) − f (a)
0f 0 (c) =
1
0
1
0
1 b−a
0
1
0
1
0
1
0
1
0
1
0
1
0
1
0
1
0
1
0
1
0
1
0
1
0
1
0
1
0
1 0
1
0
1
0
1
0
1 0
1
0
1 0
1
0
1 0
1
0
1
0
1
0
1 0
1
0
1 0
1
0
1
0
1
1111111111111111111111
0000000000000000000000
a c b

Vesna Perišić
MATH2039 ANALYSIS

PERIŠIĆ

MATH2039: Analysis 1
Introductory Remarks 2
List of Symbols 3
Greek letters 4
1. Standard notation and terminology 5
2. Properties of real numbers 7
3. Bounded sets 13
4. Limits of Sequences 17
5. Completeness properties of the reals 33
6. Continuity 37
7. Differentiability 45
8. The Cauchy mean value theorem and l’Hopital’s rule 49
9. Integration 53
10. Series 64
11. Sequences and series of functions 79
12. Power series 82
13. Taylor series 86
14. The exponential function 90
Appendix 0 Some Historic notes 95
Appendix A: Solutions to selected exercises 96
Appendix B: Examination Papers 161
Appendix C: Learning Outcomes (LOs) 166
Appendix D: Further reading 167
Index 168
Contents

1
2 PERIŠIĆ

Introductory Remarks

About the authorship of the booklet: My name on the cover of this booklet
means that I am currently responsible for teaching MATH2039 Analysis and
that I have edited already existing text for this purpose. The lecture notes are
written by my predecessors, my colleagues J. Anderson, J.Brodzki, N.Wright
and B. Nucinkis. Special thanks go to them.

My thanks also go to the 2018-19 analysis cohort of students who were critical
users of the lecture notes, in particular Martin Dimitrov, Ria Dunn, Briony
Eldridge and Mikolaj Kacki who, as the members of the Lecture Notes working
group, made many useful comments and suggestions.

The booklet covers MATH2039 Analysis syllabus as specified in the Module


Specification. The learning outcomes are listed in the Appendix C. This anal-
ysis module builds on the foundations set by the first year modules, primarily
on calculus, which is a pre-requisite for the module. MATH2039 is a core or a
compulsory module for all second year mathematics courses.

The material is organizes in chapters. The chapters are kept short and followed
by relevant exercises. The booklet starts with a list of the most common mathe-
matical symbols used in analysis including the letters from the Greek alphabet.
After stating the notation, it also gives a revision of standard terminology intro-
duced earlier in the first year modules. These empresses importance of correct
notation and terminology.

These lecture notes work best together with the lectures. Reading the notes
(with paper) in advance will help you following and engaging with the lectures.
What you learn in the lectures will help you to understand the notes even better.

All the teaching material is only good as the people who learn from it. If there
are bits of these notes you like, or don’t like, please tell me. In particular tell
me if you find mistakes. The notes are supposed to be mathematically correct.

Vesna Perišić
September 2019
MATH2039 ANALYSIS 3

List of Symbols

Commonly used symbols gathered together for your reference.


Logic
⇒ – implies (if-then)
∨ – or (disjunction)
∧ – and (conjunction)
∼ – not (negation)

Quantifiers
∀ – for all
∃ – there exists

Sets
N – natural numbers {1, 2, 3, . . . }
Z – integers {. . . , −3, −2, −1, 0, 1, 2, 3, . . . }
Q – rational numbers e.g. 1, 12 , − 13 etc.
R – real numbers √
(rational numbers and irrational numbers e.g. π = 3.14159 . . . , 2 = 1.41421 . . . )

C – complex numbers a + ib where a, b are real and i = −1
∅ – the empty set {}

∈ – is an element of (is in e.g. −1 ∈ Z, 2 ∈ R)


/ – is not an element of (is in e.g. −1 ∈
/ N, 2 ∈ / Q)
∪ – union (things that are in either or both of the sets)
∩ – intersection (things that are in both sets)
⊆ – is a subset of a set (is contained in, meaning one set is inside another)
⊂ – is a proper subset of a set (is strictly contained in, meaning the sets are
not equal)
4 PERIŠIĆ

Greek letters

Numerous Greek letters are used throughout mathematics. Here is a table so


that you know what they all are and how they are called.
A B Γ ∆ E Z
α β γ δ ε ζ
Alpha Beta Gamma Delta Epsilon Zeta

H Θ I K Λ M
η θ ι κ λ µ
Eta Theta Iota Kappa Lambda Mu

N Ξ O Π P Σ
ν ξ o π ρ σ
Nu Xi Omicron Pi Rho Sigma

T Y Φ X Ψ Ω
τ υ φ or ϕ χ ψ ω
Tau Upsilon Phi Chi Psi Omega
MATH2039 ANALYSIS 5

1. Standard notation and terminology

By a set we shall mean any collection of objects, which are called elements of
the set. A set with three elements a, b, c will be denoted {a, b, c}. If we want to
give it a name we can write
S = {a, b, c}
To say that a is an element of A, or which is the same, a belongs to A, we write
a ∈ A. To indicate that a is not an element of A we write a 6∈ A. Sets can be
described by providing a list of their elements:
S = {1, 2, 3}
which works best for finite sets. For infinite sets we may use a similar notation,
as in the case of the natural numbers
N = {1, 2, 3, 4, . . . }
where the dots are meant to suggest that the list continues indefinitely. Other
sets of numbers which we shall use frequently are:
• Integers Z = {. . . , −3, −2, −1, 0, 1, 2, 3, . . . }
• Rational numbers (or fractions), denoted by Q, contain all elements of
the form p/q, where p and q are integers
p
Q = { | p, q ∈ Z }.
q

• Real numbers R, consist of the rational numbers Q and irrational num-


bers. The set of irrational numbers does not have a separate symbol
and it contains numbers which are not rational,√ that is can not be ex-
presses as the quotient of two integers, e.g. 2, π, e, etc. We will
discuss the definition of the real numbers in more detail later.
Another convenient and frequently used way to describe sets is by specifying a
condition that is to be satisfied by its elements. For instance, to define the set
of all numbers divisible by 7 we may write:
S = {n ∈ Z | n = 7k for some integer k}.
This should be read: S is the set of integers n such that n = 7k for some integer
k. It is important to know basic relations between sets. For example, A ⊂ B
means that A is a subset of B, that is, every element of A is also an element of
B. The union (also called the sum) of the sets A and B is the set A ∪ B which
contains all elements that belong to A or to B (including the possibility that
an element can be in both A and B). The intersection A ∩ B of the two sets A
and B is the set of all elements that belong to both sets, A and B.
It is possible to take unions or intersections of infinite families of sets. For ex-
ample the intersection of the intervals [0, 1], [0, 1/2], [0, 1/3], . . . is the set
\
[0, 1/n] = {x ∈ R | x ∈ [0, 1/n] ∀n ∈ N}.
n∈N
6 PERIŠIĆ

You might want to ask yourself what this set really is.
Similarly we could take the union of the sets [0, 1/2], [0, 2/3], . . . , [0, n/(n +
1)], . . . . This is
[  n
  
n

0, = x ∈ R | ∃n ∈ N for which x ∈ 0, .
n+1 n+1
n∈N
Again you might want to ask yourself what this set is. Note that defining these
sets did not involve ‘limits’ or letting ‘n tend to infinity,’ although answering
the questions posed above will involve limits.
A function f from a set A to a set B, denoted f : A → B, is a rule that assigns
to each element a of the set A a unique element of the set B. The value of f on
an element a is denoted f (a), the set A is called the domain of f. The set of all
elements b of B that are the image of some element a in A under the function f
is called the range or the image of f and is denoted f (A). Note that the image
of f need not be the whole set B.
We say that a function f : A → B is an injection if and only if any element of B
can be the image of at most one element of A. In other words, we require that
if f (x) = f (y) then this must imply that x = y. The function f is a surjection,
or onto if every element of B is the image of some element in A, i.e. when
the image of f is all of B: f (A) = B. Finally, f is a bijection if and only if
it is both an injection and surjection. In this case we say that f establishes a
one-to-one correspondence between the sets A and B.
When describing properties of sets, we shall need to use the quantifiers. These
useful symbols allow one to state that a property is to hold for all or for some
element of a given set. The symbol ∀ is read ‘for all’, and a typical example of
its usage is the statement
∀x ∈ R, x2 ≥ 0
which should be read: For all real numbers x, x squared is greater than or equal
to zero.
When we want to say that there exists an element satisfying a certain property
we use the symbol ∃, which is read: there exists, as in the statement:
∃x ∈ R, x − 1 = 0.
This should be read: There exists a real number x such that x − 1 equals
zero.
MATH2039 ANALYSIS 7

2. Properties of real numbers

Definition 2.1. A field is a set F equipped with two (binary) operations :


• addition (denoted +) so that for any pair a and b of elements of F ,
their sum a + b is defined, and
• multiplication (denoted ·) so that for any pair a and b of elements of
F , their product a · b is defined,
that satisfy the following conditions:
• F is a commutative group under addition: this means
(i) for every three elements a, b and c in F , we have a + (b + c) =
(a + b) + c (associativity);
(ii) for every pair a and b of elements in F , a+b = b+a (commutativity);
(iii) there is an element 0 of F (called the additive identity) so that
a + 0 = a for all a ∈ F ;
(iv) for each element a ∈ F , there is an element −a (called the addi-
tive inverse) such that a + (−a) = 0 (so also (−a) + a = 0 by
commutativity);
• the set F ∗ = F − {0} is a commutative group under multiplica-
tion: this means
(i) for every three elements a, b and c in F ∗ , we have a·(b·c) = (a·b)·c
(associativity);
(ii) for every pair a and b of elements in F ∗ , a·b = b·a (commutativity);
(iii) there is an element 1 of F ∗ (called the multiplicative identity)
so that a · 1 = a for all a ∈ F ∗ ;
(iv) for each element a ∈ F ∗ , there is an element a−1 (called the mul-
tiplicative inverse) such that a · a−1 = 1 (so also a−1 · a = 1 by
commutativity);
• addition and multiplication are related by the distributive law: that
is, for any three elements a, b, and c of F , we have a·(b+c) = a·b+a·c.
Example 2.2. • The real numbers R with its usual operations of addi-
tion and multiplication is a field. (In fact, we can think of the axioms
defining a field as being abstracted from the familiar properties of the
real numbers.)
• The rational numbers Q also form a field, (a subfield of R).
• The integers Z do not form a field, as there is no multiplicative inverse
for 2 (or 3 or −57).
Example 2.3. Use the definition of a field to prove that if F is a field then
∀a ∈ F , a · 0 = 0 · a = 0.
8 PERIŠIĆ

Since 0 is the additive identity we have


x+0=x
for all elements x in F . In particular, taking x = 0 we know that
0 + 0 = 0.
We multiply both sides on the left by a to get
a · (0 + 0) = a · 0.
Now apply the distributive law to the left hand side to get
a · 0 + a · 0 = a · 0.
Every element of F has an additive inverse, so there is an element −(a · 0) such
that
a · 0 + (−(a · 0)) = 0.
Adding −(a · 0) to both sides of the above equation a · 0 + a · 0 = a · 0 we get
(a · 0 + a · 0) + (−(a · 0)) = a · 0 + (−(a · 0)).
Associativity allows us to move the parentheses so we have
a · 0 + (a · 0 + (−(a · 0))) = a · 0 + (−(a · 0))
and since a · 0 + (−(a · 0)) = 0, this simplifies to a · 0 + 0 = 0. Since 0 is the
additive identity we get a · 0 = 0 as desired.
To prove that 0 · a = 0, we just repeat this argument, multiplying on the right
by a instead of on the left. (To see that a · 0 = 0 · a, we note that both are
equal to 0, and hence are equal to each other.)
Example 2.4. Use the definition of a field to prove that if F is a field then
∀a ∈ F , −(−a) = a and ∀a, b ∈ F , −(a + b) = (−a) + (−b).

First we’ll show that for a ∈ F , we have −(−a) = a. What do we know about
−(−a)? The additive inverse axiom says that if we add this to (−a) then we
get 0, thus
(−a) + (−(−a)) = 0.
Let us add a to both sides to get a+((−a)+(−(−a))) = a. Now by associativity
of addition we can move the parentheses to get
(a + (−a)) + (−(−a)) = a.
As (−a) is the additive inverse to a we have a + (−a) = 0, so we get
0 + (−(−a)) = a.
Finally as 0 is the additive identity we get
−(−a) = a.

Now we’ll show that for a and b in F we have −(a + b) = (−a) + (−b). Again,
what do we know? We have
a + (−a) = 0 and b + (−b) = 0
MATH2039 ANALYSIS 9

and adding these together we get


(a + (−a)) + (b + (−b)) = 0 + 0.
As 0 is the additive identity we have 0 + 0 = 0 on the right hand side. By
associativity and commutativity we can rearrange the left hand side to get
(a + b) + ((−a) + (−b)) = 0.
Now add −(a + b) to both sides. This gives us

−(a + b) + (a + b) + ((−a) + (−b)) = −(a + b) + 0.
Finally rearranging the parentheses again, and using the fact that 0 is the
additive identity we get
(−a) + (−b) = −(a + b).

Exercise 2.5. Prove that each of the following statements holds in a field F ,
using only the axioms of a field.
(1) a · (−b) = (−a) · b = −(a · b) for all a, b ∈ F ;
(2) (−1) · a = −a for all a ∈ F .
Example 2.6. Let p be a prime number, and consider the integers modulo
p, usually denoted Zp or Z/pZ. This is a field.
The integers modulo p means that we only allow numbers 0, 1, . . . p−1 and then
we return to 0. For example if we add 1 to p − 1 we get 0, and if we multiply
p − 1 by 2 we get p − 2 (you’d expect 2p − 2, but the extra p loops back to zero).
Here are the multiplication and addition tables for Z5 .
+ 0 1 2 3 4 × 0 1 2 3 4
0 0 1 2 3 4 0 0 0 0 0 0
1 1 2 3 4 0 1 0 1 2 3 4
2 2 3 4 0 1 2 0 2 4 1 3
3 3 4 0 1 2 3 0 3 1 4 2
4 4 0 1 2 3 4 0 4 3 2 1
Exercise 2.7. Let n ≥ 4 be an integer that is not prime. Show that the
integers modulo n, Zn , is not a field (which axiom, or axioms fail?).
Definition 2.8. An order on a set X is a relation <, called less than, on X
satisfying the following conditions:
• (trichotomy) for all a, b in X, exactly one of the following holds
(i) a < b
(ii) a = b
(iii) b < a
• (transitivity) for any three elements a, b, and c of X, if a < b and
b < c, then a < c.
10 PERIŠIĆ

Of course we use the notation a > b (a greater than b) to mean b < a, and
a ≤ b (and b ≥ a) to mean that either a < b or a = b.
An ordered field is a field F with an order < that is well behaved with respect
to the operations of addition and multiplication, namely
• for any three elements a, b, and c of F , if a < b, then a + c < b + c;
• for any three elements a, b, and c of F with c > 0, if a < b, then
a · c < b · c.
Example 2.9. The real numbers R and the rational numbers Q with the usual
notion of < are ordered fields.
Exercise 2.10. Let F be an ordered field. Prove that if a in F and a 6= 0 then
a · a > 0. Hence deduce that 0 < 1 in an ordered field.
Example 2.11. The field Zp of integers modulo a prime p do not form an
ordered field.

Suppose that Zp is an ordered field. In Zp we have 1 + 1 + . . . 1 = 0. On the


| {z }
p
other hand by the above exercise if Zp is an ordered field then 0 < 1. Hence
by the axioms for an ordered field we have 0 + 1 < 1 + 1 i.e. 1 < 1 + 1. Now
(by the transitivity axiom) as 0 < 1 and 1 < 1 + 1 we deduce that 0 < 1 + 1.
We proceed inductively: If 0 < 1 + 1 + . . . 1, then adding 1 to both sides we have
| {z }
n
0 + 1 < 1 + 1 + . . . 1. As 0 + 1 = 1 and 0 < 1 we deduce that 0 < 1 + 1 + . . . 1
| {z } | {z }
n+1 n+1
by transitivity. Thus by induction 0 < 1 + 1 + . . . 1 for all n. On the other
| {z }
n
hand for n = p we have 0 = 1 + 1 + . . . 1. This contradicts the trichotomy
axiom (we cannot have both 0 = 1 + 1 + . . . 1 and 0 < 1 + 1 + . . . 1), hence we
deduce that Zp is not an ordered field.
Exercise 2.12. Show that every ordered field contains a copy of Z. Hence
deduce that every ordered field contains a copy of Q.
Exercise 2.13. Prove that there does not exist an order on the complex num-
bers C so that C becomes an ordered field.

In analysis we will make a lot of use of inequalities (i.e. ordering). We will now
discuss some standard inequalities involving absolute values.
The absolute value, |x|, of a real number x is defined by

x,
 if 0 < x
|x| = 0, if 0 = x

−x, if 0 > x

Exercise 2.14. Show that |x| is always greater than or equal to zero.
MATH2039 ANALYSIS 11

Let us note some consequences of the definition. First, −|x| ≤ x ≤ |x| (in fact
x always equals either |x| or −|x| (or both in the case of 0)). This implies in
particular that if |x| = 0 then 0 ≤ x ≤ 0, so x = 0.
The following are key properties of the absolute value and will be used fre-
quently.
Proposition 2.15. For F an ordered field and x, y in F we have
(1) |xy| = |x||y|;
(2) |x + y| ≤ |x| + |y| (The triangle inequality);
(3) ||x| − |y|| ≤ |x − y| (The reverse triangle inequality).

Proof. (1) This is proved by checking the possible cases. If either x = 0 or


y = 0 the statement is true since a · 0 = 0 in a field. If they are both positive,
then so is xy and so |xy| = xy = |x||y|. Whey x and y are both negative, then
|x| = −x, and |y| = −y are positive, so xy = (−x)(−y) is positive. Hence
|xy| = xy = (−x)(−y) = |x||y|.

Finally, when one of the two, say x, is negative and the other one, y is positive,
then xy < 0 so
|xy| = −(xy) = (−x)y = |x||y|

(2) The following argument is very useful. We noted that x ≤ |x| and y ≤ |y|,
hence x + y ≤ |x| + |y|. We also have that x ≥ −|x| and y ≥ −|y| so rearranging
we have −x ≤ |x| and −y ≤ |y| adding these together we get −(x+y) ≤ |x|+|y|.
Since either |x + y| = x + y or |x + y| = −(x + y) and both of these are less than
or equal to |x| + |y| we have |x + y| ≤ |x| + |y| as claimed.
(3) This is a similar argument to 2. We have
|x| = |(x − y) + y| ≤ |x − y| + |y|
by 2. Hence rearranging we have |x| − |y| ≤ |x − y|. We can argue the same
way to get
|y| ≤ |x − y| + |x|
which implies |y| − |x| ≤ |x − y|. Thus |x − y| is greater than or equal to both
|x| − |y| and |y| − |x|. Since ||x| − |y|| is equal to one of these two numbers we
conclude that
||x| − |y|| ≤ |x − y|


Proposition 2.16. Let x, y, a ∈ R. Then


(1) x < y + ε for all ε > 0 ⇐⇒ x ≤ y;
(2) x > y − ε for all ε > 0 ⇐⇒ x ≥ y;
(3) |a| < ε for all ε > 0 ⇐⇒ a = 0.
12 PERIŠIĆ

Proof. We shall only prove part (1). The other two parts are left as an exercise.
⇒: Suppose, for a contradiction, that x < y + ε for all ε > 0 yet x > y. Set
ε0 = x − y > 0. Then y + ε0 = x and hence, by trichotomy it cannot be greater
than x. This contradicts the hypothesis for ε = ε0 . Hence x ≤ y.
⇐: Suppose x ≤ y. Take an arbitrary ε > 0. By the trichotomy, we have
either x < y or x = y and we consider both cases separately. If x < y, then
x + 0 < y + 0 < y + ε by the transitive and additive properties. If x = y, we
have x + 0 = y + 0 < y + ε by the additive property. Hence, in both cases, for
all ε > 0, we have x < y + ε.

Definition 2.17. A subset A of an ordered field F has maximum s if s ∈ A
and ∀x ∈ A we have s ≥ x. A subset A of an ordered field F has minimum t
if t ∈ A and ∀x ∈ A we have t ≤ x.
Proposition 2.18. If A is a finite nonempty subset of an ordered field F then
it has a minimum and a maximum.

Proof. The proof is by induction on the number of elements in A. If A has size


n = 1 then A = {a} for some a ∈ F . The number a is both a maximum and
a minimum of A, since a ∈ A, and for any x ∈ A we have x = a so a ≥ x and
a ≤ x.
Now suppose that the result holds for all sets of size n. If A is a set of size
n + 1 then pick a in A and let B = {b ∈ A : b 6= a} of size n. By the induction
hypothesis, B has a maximum sB and a minimum tB . If a ≤ sB then let
s = sB . We have s ∈ B ⊂ A, and for all x ∈ A either x ∈ B, so s ≥ x as
this is the maximum of B, or x = a and by assumption s ≥ a. Hence s is the
maximum of A. Otherwise a > sB and we let s = a. Then s ∈ A and for all
x ∈ A either x ∈ B, so s = a > sB ≥ x, or x = a = s. Again we have shown
that s is the maximum of A.
The argument for the minimum is similar. 

Another key property of the real numbers and rational numbers is the following
fact called the Archimedean property: Given any x, y with x > 0 and y > 0
there exists n ∈ N such that n·x > y. This says that x cannot be infinitesimally
small and y cannot be infinitely large.
Proposition 2.19. The rational numbers Q satisfy the Archimedean property.

Proof. Let x, y be positive rational numbers. Putting these over a common


denominator d we have x = ad , y = db with a, b, d ∈ N. As d > 0, it suffices to
show that there exists n with na > b, since this implies that nx > y. We prove
this by induction on b. If b = 1 then n = 2 suffices as 2a > a ≥ 1 = b. Now
suppose that for a given b there exists n with na > b. Then (n + 1)a = na + a ≥
na + 1 > b + 1. Hence by induction, for all b there exists n with na > b as
required. 
MATH2039 ANALYSIS 13

3. Bounded sets

Definition 3.1. A set A of real numbers is bounded above if there exists a


real number s so that a ≤ s for all a ∈ A. A number s satisfying this definition
is called an upper bound for A . Note that upper bounds are not unique;
if s is an upper bound for A, then s + 1, s + 2, . . . , s + 100, . . . are also upper
bounds for A.
Definition 3.2. A set A of real numbers is bounded below if there exists a
real number t so that t ≤ a for all a ∈ A. A number t satisfying this definition
is called a lower bound for A. Note that lower bounds are not unique; if t is
a lower bound for A, then t − 1, t − 2, . . . , t − 100, . . . are also lower bounds for
A.
Definition 3.3. A set A of real numbers is bounded if it is both bounded
above and bounded below.
Definition 3.4. Let A be a set of real numbers. The supremum of A, denoted
sup(A), is a number s ∈ R satisfying two properties:
(1) ∀x ∈ A x ≤ s;
(2) if u ∈ R is a number such that ∀x ∈ A x ≤ u, then s ≤ u.

In words this says that s is an upper bound for A, and if u is any upper
bound for A, then s is smaller than or equal to u. The supremum is therefore
sometimes called the least upper bound of A, as it is the smallest of all
possible upper bounds for A. Any set of real numbers that is bounded above
has a supremum.
Definition 3.5. Let A be a set of real numbers. The infimum of A, denoted
inf(A), is a number t ∈ R satisfying two properties:
(1) ∀x ∈ A t ≤ x;
(2) if w ∈ R is a number such that ∀x ∈ A w ≤ x, then w ≤ t.

In words this says that t is a lower bound for A, and if w is any lower bound for
A, then t is greater than or equal to w. The infimum is therefore sometimes
called the greatest lower bound of A, as it is the largest of all possible
lower bounds for A. Any set of real numbers that is bounded below has an
infimum.
Example 3.6. Let S = [0, 1] We shall show that sup(S) = 1 : By the definition
of the interval, we have that 1 ≥ x for all x ∈ S, hence 1 is an upper bound.
Now we have to show that 1 is a least upper bound. Suppose u ≥ x for all
x ∈ S, i.e u is an upper bound of S. Since 1 ∈ S, it follows that u ≥ 1 as
required.
Remark 3.7. If a set S has an upper bound u, then it has infinitely many
upper bounds, as any number M > u is also an upper bound of S.
Exercise 3.8. Let S = [0, 1). Show that sup(S) = 1.
14 PERIŠIĆ

Lemma 3.9. If a set S has a supremum (infimum) then this supremum (infi-
mum) is unique.

Proof. We prove this statement for the supremum. Suppose s1 and s2 are
both suprema of S. Then, by the definition of supremum, they are both upper
bounds of S, and hence, by the second part of the definition applied to s1 and
s2 separately, we have s1 ≤ s2 and s2 ≤ s1 . Hence by trichotomy, we have
s1 = s2 . 

This proof illustrates a general principle. It is often easier to prove that a = b


by proving a ≤ b and b ≤ a separately.

The next result is important as it shows that a supremum of a set S can be


approximated by a point in S. An analogue results also holds for the infimum
of a set. Statement and proof are left as an exercise.
Theorem 3.10. Approximation property for suprema. Let S be a set
with a supremum sup(S). For every ε > 0 there is a point a ∈ S such that
sup(S) − ε < a ≤ sup(S).

Proof. Suppose, for a contradiction, that there is no such a ∈ S. Then, there is


an ε > 0 such that for all a ∈ S either a ≤ sup(S) − ε or a > sup(S). The case
a > sup(S) contradicts the first property of the supremum. Hence for all a ∈ S
we have a ≤ s0 = sup(S) − ε. Thus s0 is an upper bound for S. Hence, by the
second property of supremum, sup(S) ≤ s0 = sup(S) − ε implying ε = 0, which
gives a contradiction. 

We can use this approximation property to find the supremum s or the infimum
t of a set S when s ∈
/ S or t ∈
/ S as the following example illustrates. But first, let
us mention that we shall make one further assumption on the set of real numbers
R. We assume that every non-empty subset of R, which is bounded above, has
a supremum. This property is often referred to as the Completeness Axiom.
We shall use a different definition of completeness later on, but it will turn
out that the two are equivalent. We shall also see later that completeness for
suprema is equivalent to completeness for infima, i.e. a non-empty subset of R,
which is bounded below has an infimum.
Example 3.11. For the subset S = n1 | n ∈ N of R, determine whether S is


bounded above, bounded below, bounded, or neither. If S is bounded above,


determine sup(S), and decide whether or not sup(S) is an element of S. If S
is bounded below, determine inf(S), and decide whether or not inf(S) is an
element of S.

S is bounded below by 0 (since 0 < n1 for all n ∈ N), and so has an infimum.
Since 0 is a lower bound and since there are elements of S arbitrarily close to 0
(given ε > 0, we can find n so that n1 < ε by taking n to be any integer greater
than 1ε ), the infimum inf(S) = 0. In this case, inf(S) 6∈ S.
MATH2039 ANALYSIS 15

S is bounded above by 1 (since n1 ≤ 1 for all n ∈ N), and so has a supremum.


Making use of a result from Exercise 3.16, since 1 is an upper bound for S and
since 1 ∈ S, we have that 1 = sup(S). In this case, sup(S) ∈ S.

Since S is both bounded above and bounded below, it is bounded.

The approximation property is also used to show the following property of sets
of integers, which, for example doesn’t hold in Q as we shall see later.
Theorem 3.12. Let S ⊂ Z be a set of integers with a supremum s. Then
s ∈ S.

Proof. We apply the approximation property with ε0 = 1. Then there is an


x ∈ S such that s − 1 < x ≤ s. If s = x, then s ∈ S as required. Hence we
suppose s − 1 < x < s and apply the approximation property with ε1 = s − x.
Hence there is a y ∈ S such that x < y ≤ s. By the same argument as above
we can assume that x < y < s. Hence 0 < y − x < s − x. Furthermore, since
−x < 1 − s, we conclude that 0 < y − x < s + (1 − s) = 1 and y − x ∈ Z ∩ (0, 1),
which gives the desired contradiction. 
Exercise 3.13. For each subset S of R given below, determine whether S is
bounded above, bounded below, bounded, or neither. If S is bounded above,
determine sup(S), and decide whether or not sup(S) is an element of S. If S
is bounded below, determine inf(S), and decide whether or not inf(S) is an
element of S.
(1) S = n1 | n ∈ Z\{0} ;


(2) S = {2x | x ∈ Z};


(3) S = [−1, 1] ∪ {5} = {x ∈ R | − 1 ≤ x ≤ 1} ∪ {5};
(4) S = 2xy | x, y ∈ N ;


(5) S = (−1)n 1 + n1 | n ∈ N ;
 

(6) S = {x ∈ R | |x| > 2};


Lemma 3.14. For subsets A and B of R
sup(A ∪ B) = max{sup(A), sup(B)}.

Proof. Assuming, without loss of generality (wlog), that sup(A) ≥ sup(B) we


get
max{sup(A), sup(B)} = sup(A),
and hence we need to show that sup(A ∪ B) = sup(A). To show this we need
to show two things, that sup(A) is an upper bound for A ∪ B and that if u is
any upper bound for A ∪ B, then u ≥ sup(A).

If a ∈ A, then a ≤ sup(A) by definition (since sup(A) is greater than or equal


to every element of A). Similarly, if b ∈ B, then b ≤ sup(B); since sup(B) ≤
sup(A), this yields that b ≤ sup(A) for all b ∈ B. Since every element c of
16 PERIŠIĆ

A ∪ B satisfies either c ∈ A or c ∈ B (or both), we see that c ≤ sup(A), and so


sup(A) is an upper bound for A ∪ B.

Let u be any upper bound for A ∪ B. Since u ≥ c for every c ∈ A ∪ B, we also


have that u ≥ c for every c ∈ A. In particular, u is an upper bound for A, and
so by the definition of supremum, u ≥ sup(A). Therefore, sup(A) is an upper
bound for A ∪ B that is less than or equal to any other upper bound for A ∪ B.
That is, sup(A ∪ B) = sup(A). 
Lemma 3.15. Monotone Property Let A ⊆ B be non-empty subsets of R.
(1) If A and B both have a supremum, then sup(A) ≤ sup(B).
(2) If A and B both have an infimum, then inf(B) ≤ inf(A).

Proof. We only show (1), the second part is left as an exercise.


Since A ⊆ B, we have for all a ∈ A that a ≤ sup(B) and therefore sup(B) is an
upper bound of A. Now, the second property for supremum applied to sup(A)
(that is, sup(A) is the least upper bound of A), implies that sup(A) ≤ sup(B)
as required. 
Exercise 3.16. In this question, A and B are subsets of R. Show that each
of the following holds. (You can use that a subset, which is bounded above
(below) has a supremum (infimum).
(1) if A ∩ B 6= ∅, then sup(A ∩ B) ≤ min{sup(A), sup(B)};
(2) if A ∩ B 6= ∅, then inf(A ∩ B) ≥ max{inf(A), inf(B)};
(3) if t is an lower bound for A and if t ∈ A, then t = inf(A);
(4) if inf(A) exists, then inf(A) = sup{y | y is a lower bound of A};
(5) if sup(A) exists, then sup(A) = inf{y | y is a upper bound of A};
(6) sup(A) is unique if it exists;
(7) inf(A) is unique if it exists.
Exercise 3.17. In this question, A is a subset of R. Define A− = {a| −a ∈ A}.
Show that each of the following holds.
(1) if sup(A) exists, then inf(A− ) exists and inf(A− ) = − sup(A);
(2) if inf(A) exists, then sup(A− ) exists and sup(A− ) = − inf(A).
Exercise 3.18. For each of the following, either give an example of a subset S
of R satisfying the stated property, or prove that no such set exists.
(1) S has a rational lower bound and inf(S) is irrational;
(2) S has a rational lower bound and inf(S) is rational;
(3) S has an irrational lower bound and inf(S) is rational;
(4) S has an irrational lower bound and inf(S) is irrational.
MATH2039 ANALYSIS 17

4. Limits of Sequences

A sequence a0 , a1 , a2 , . . . is an infinite list of objects. To each n ∈ Z+ , it is


assigned an object an , n 7→ an . The objects of a sequence may be anything:
numbers, sets, vectors, functions, etc. We will give a formal definition of a
sequence:

Definition 4.1. Let F be a field. A sequence of elements of F is a function


f : I → F where I, called the indexing set, is a collection of consecutive
positive integers, increasing indefinitely. This can be written as
I = {m, m + 1, m + 2, . . . }, m ∈ N ∪ {0} = Z+ .

The indexing set I of a sequence is usually, but not always, taken to be N,


the set of natural numbers. The terms of a sequence are more commonly de-
noted by a1 , a2 , a3 , . . . or something similar, rather than by f (1), f (2), f (3), . . . .

A sequence a1 , a2 , a3 , . . . can be denoted by {an : n ∈ I}, {an }n∈I , {an }∞


n=1 ,
{an }, (a1 , a2 , . . . ), (an )n∈I , (an )∞ n=1 , (an ) .

We will use (an )n∈I or simply (an ).


The n-the element of a sequence (an ) is denoted by an (without brackets).
A sequence (an ) can be defined by giving a formula for its n-th term an .
Example 4.2. (a) The sequence (an ) where the n-th term is defined by
an = n, n ∈ N is the sequence
1, 2, 3, . . . .
1
(b) The sequence (bn ) where the n-th term is defined by bn = ln(n) , n≥2
is the sequence
1 1 1
, , ,....
ln(2) ln(3) ln(4)

A sequence can also be defined recursively or inductively by:


(i) defining the first term (or the first few terms),
(ii) giving a method how to compute n-th term from preceding terms.
Example 4.3. (a) The sequence of even numbers 2, 4, 6, . . . can be defined
by a0 = 2, an = an−1 + 2, n ∈ N.
(b) The sequence 2, 4, 8, . . . can be defined by
a0 = 2, an = 2an−1 , n ∈ N.
(c) The Fibonacci sequence 1, 1, 2, 3, 5, 8, . . . can be defined by
a0 = a1 = 1, an = an−1 + an−2 , n ∈ N.
Definition 4.4. A subsequence of a sequence (an )n∈I is a sequence of the
form (ank )k∈J , where for each k ∈ J we have that nk ∈ I and nk < nk+1 .
18 PERIŠIĆ

Example 4.5. For the sequence (an )n∈N , an = n the sequences


(bk )k∈N , bk = a2k = 2k, k ∈ N
and
(ck )k∈N , ck = ak+1 = k + 1, k ∈ N
are examples of subsequences.

We will be studying convergence of a sequence.


Definition 4.6. Let (an )n∈I be a sequence in F and let L ∈ F . The sequence
(an )n∈I converges to L if and only if for every ε > 0, there exists n0 ∈ I such
that
|an − L| < ε for every n ∈ I with n > n0 .

Symbolically, the sentence ”a sequence (an ) converges to L” is denoted either


by
an → L as n → ∞ or by lim an = L.
n→∞
We also say ”L is the limit of the sequence (an )n∈I .”
Example 4.7. We show that n1 → 0 as n → ∞. Let ε > 0 be arbitrary. Then
there exists an n0 ∈ N such that n0 > 1ε . (The existence of n0 follows from the
Archimedean Property for R which we will show later, and the existence of a
well defined floor-function will be seen later as well.) Hence, for all n > n0 we
have n1 < n10 < ε. Since, for all n ∈ N, we have n1 > 0, we conclude that
1 1
| | = | − 0| < ε
n n
for all n > n0 as required.
Example 4.8. To show that 2n+1 2n → 1 as n → ∞ we need to consider | 2n −
2n+1
1 1
1| = |1 + 2n − 1| = | 2n |. Let ε > 0 be arbitrary. As above, we can find an
1 1
n0 ∈ N such that n0 > 2ε . Hence, for all n > n0 we have 2n < 2n1 0 < ε. Since,
1
for all n ∈ N, we have 2n > 0, we conclude that
2n + 1 1
| − 1| = | | < ε
2n 2n
for all n > n0 as required.
Definition 4.9. A sequence that does not converge, diverges .
Example 4.10. The sequence (an ), an = (−1)n , n ∈ N diverges. Suppose
that for some a ∈ R, (−1)n → a as n → ∞. Hence, given any positive number
ε, there is an n0 ∈ N such that |(−1)n − a| < ε for all n > n0 . Now suppose
ε = 1. For n odd this gives | − 1 − a| = |1 + a| < 1 and for n even this gives
|1 − a| < 1. Furthermore, by the triangle inequality,
2 = |1 + 1| = |(1 − a) + (1 + a)| ≤ |1 − a| + |1 + a| < 1 + 1 = 2
implying 2 < 2, a contradiction.
MATH2039 ANALYSIS 19

Some properties of the limits.

Lemma 4.11. (Uniqueness) The limit of a sequence is unique if it exists.


(A sequence can have at most one limit.)

Proof. Suppose an → a and an → b as n → ∞. By definition, for any given


ε > 0, there exists an integer n1 such that |an − a| < 2ε for all n > n1 and an
integer n2 such that |an − b| < 2ε for all n > n2 . Take n0 = max{n1 , n2 }. Then,
for all n > n0 we have:
ε ε
|an − a| = |a − an | < and |an − b| <
2 2
Now we apply the triangle inequality to obtain:
ε ε
|a − b| = |(a − an ) + (an − b)| ≤ |a − an | + |an − b| < + = ε
2 2
That is, for all ε > 0, we have |a − b| < ε. Applying Theorem 2.16 yields
a = b. 
Lemma 4.12. If (an )n∈I converges to L, and (bk )k∈J , bk = ank , k ∈ J is a
subsequence of (an ), then (bk ) converges to L.

Proof. For simplicity we will take the indexing sets I, J to be N.


As (an ) converges to L, given any ε > 0 we know that there exists N0 ∈ N such
that if n > N0 then |an − L| < ε. Now suppose that k > N0 and consider ank .
Note that nk ≥ k: this is certainly true for k = 1, and inductively if nk ≥ k
then, (since nk+1 > nk ,) nk+1 > nk ≥ k +1. Hence if k > N0 then nk ≥ k > N0 .
It thus follows that for all k > N0 we have
|bk − L| = |ank − L| < ε.
This proves that (bk ) converges to L as required. 
Definition 4.13. A sequence (an ) is
(a) bounded above iff there is some number R such that an ≤ R for all
n;
(b) bounded below iff there is some number R such that R ≤ an for all
n;
(c) bounded iff it is both bounded above and bounded below or, iff there
is some number R such that |an | ≤ R for all n.
1
Example 4.14. (1) The sequence (an ), an = n, n ∈ N is bounded.
(2) The sequence (an ), an = (−1)n n2 , n ∈ N is bounded.
Theorem 4.15. Every convergent sequence (an ) is bounded.

Proof. Let  = 1 and let an → L as n → ∞. Since an → L, it exists N ∈ N


such that |an − L| < 1 for all n ∈ N and n > N. Then,
|an | = |an − L + L| ≤ |an − L| + |L| < 1 + |L| for all n > N.
20 PERIŠIĆ

Now, for n ≤ N
|an | ≤ max{|a1 |, |a2 |, . . . , |aN |}
and therefore
|an | ≤ max{|a1 |, |a2 |, . . . , |aN |, 1 + |L|}, for all n ∈ N.
Here, clearly R = max{|a1 |, |a2 |, . . . , |aN |, 1 + |L|}. 

Theorem 4.16. ( Arithmetic of sequences/limits)


Let (an )n∈I and (bn )n∈I be convergent sequences with
lim an = a, and lim bn = b.
n→∞ n→∞

Then the following holds:


(1) sum: the sequence (an + bn )n∈I converges to a + b as n → ∞;
(2) difference: the sequence (an − bn )n∈I converges to a − b as n → ∞;
(3) product: the sequence (an bn )n∈I converges to ab as n → ∞;
(4) reciprocal: if a 6= 0, then the sequence ( a1n )n∈I converges to 1
a as
n → ∞;
(5) quotient: if b 6= 0, then the sequence ( abnn )n∈I converges to a
b as n →
∞.

Proof.
(1) We need to show that for any given ε > 0, there exists n0 ∈ I so that
|(an + bn ) − (a + b)| < ε for n ∈ I, n > n0 . So, start by simplifying:
|(an + bn ) − (a + b)| = |an − a + bn − b| ≤ |an − a| + |bn − b|.
Since limn→∞ an = a, there exists n1 so that |an − a| < 12 ε for n > n1 .
Since limn→∞ bn = b, there exists n2 so that |bn − b| < 12 ε for n > n2 .
So, for n > n0 = max{n1 , n} ), we have that both |an − a| and |bn − b|
are less than 12 ε, and so
1 1
|(an + bn ) − (a + b)| ≤ |an − a| + |bn − b| < ε + ε = ε.
2 2

(2) We need to show that for any given ε > 0, there exists n0 ∈ I so that
|(an − bn ) − (a − b)| < ε for n ∈ I, n > n0 . So, start by simplifying:
|(an − bn ) − (a − b)| = |an − a − bn + b| ≤ |an − a| + | − bn + b| = |an − a| + |bn − b|.
Since limn→∞ an = a, there exists n1 so that |an − a| < 12 ε for n > n1 .
Since limn→∞ bn = b, there exists n2 so that |bn − b| < 12 ε for n > n2 .
So, for n > n0 = max{n1 , n2 }, we have that both |an − a| and |bn − b|
are less than 12 ε, and so
1 1
|(an − bn ) − (a − b)| ≤ |an − a| + |bn − b| < ε + ε = ε.
2 2
MATH2039 ANALYSIS 21

(3) We need to show that for any given ε > 0, there exists n0 so that
|an bn − a b| < ε for n > n0 . We follow the same general pattern as for
the previous two, though the algebra is a bit more intricate. As before,
we begin by simplifying |an bn − a b|:
|an bn − a b| = |an bn − an b + an b − a b| ≤ |an bn − an b| + |an b − a b|.
We take these two terms one at a time.

We start with |an bn − an b| = |an | |bn − b|. Since limn→∞ an = a,


we have control over |an |; specifically, we have that for large n, |an | is
almost equal to |a|. Specifically, apply the definition of limn→∞ an = a
with ε = 1 to see that there exists n1 so that |an − a| < 1 for n > n1 .
Rearranging, this implies that |an | < |a| + 1 for n > n1 . Using this
positive constant |a| + 1, we now apply the definition of limn→∞ bn = b
to get n2 so that |bn − b| < 21 |a|+1 1
ε for n > n2 . Thus, for n >
max{n1 , n2 }, we have that
1 1 1
|an bn − an b| = |an | |bn − b| < (|a| + 1) ε = ε.
2 |a| + 1 2

The other term to consider is |an b − a b| = |an − a| |b|. We first note


that |b| < |b| + 1. (This is to eliminate the possibility of dividing by 0,
as we’ll see in a second.) Since limn→∞ an = a, there exists n3 so that
|an − a| < 21 |b|+1
1
ε for n > n3 . Hence, for n > n3 we have that
1 1 1
|an b − a b| = |an − a| |b| < |an − a| (|b| + 1) ≤ ε(|b| + 1) = ε.
2 |b| + 1 2
Hence, for n > n0 = max{n1 , n2 , n3 }, we have that
1 1
|an bn − a b| ≤ |an bn − an b| + |an b − a b| < ε + ε = ε.
2 2

(4) we need to show that for any given ε > 0, there exists n0 so that
| a1n − a1 | < ε for n > n0 . So, as with all the others, we start by
simplifying:

1 1 |a − an |
− = .
an a |a an |
Note that since a 6= 0 by assumption and since an converges to a, we
can choose n1 so that an 6= 0 for all n > n1 , by taking ε = 12 |a| in the
definition of limn→∞ an = a.

Again taking ε = 21 |a| in the definition of limn→∞ an = a, we have not


only that an 6= 0 for n > n1 , but we also have that 32 |a| > |an | > 12 |a|
for n > n1 , since |an | lies in the interval of radius 12 |a| centred at |a|.
Hence, for n > n1 , we have that
1 2
≤ 2.
|a an | |a|
22 PERIŠIĆ

|a|2
Since limn→∞ an = a, we can choose n2 so that |an − a| < 2 ε for
n > n2 . Then, for n > n0 = max{n1 , n2 }, we have that
2 |a|2

1 1 |a − an |
− = < ε = ε,
an a |a an | |a|2 2
as desired.
(5) One way to do this would be to repeat the style of argument just given
for reciprocals, which would work but which is somewhat involved. An-
other approach, and the one we take here, is to note that since an con-
verges to a and since bn converges to b 6= 0, we have that b1n converges
to 1b , by what we just did with reciprocals, and hence abnn converges to
a
b , by taking products.


Theorem 4.17. (Squeeze rule) Let (an ), (bn ), and (cn ) be sequences such
that an ≤ bn ≤ cn for all n and (an ) and (cn ) both converge to the same limit:
limn→∞ an = limn→∞ cn = L. Then (bn ) converges and limn→∞ bn = L.

Proof. We are given that an ≤ bn ≤ cn for all n and that (an ) and (cn )
both converge with limn→∞ an = limn→∞ cn = L. We want to show that (bn )
converges and limn→∞ bn = L, which is to say, for every ε > 0, there exists n0
so that |bn − L| < ε for n > n0 .

Since limn→∞ an = limn→∞ cn = L, we have that limn→∞ (an −cn ) = L−L = 0,


and so for every µ > 0, there exists n1 so that |an − cn | < µ for n > n1 . Since
an ≤ bn ≤ cn for all n, we know that |an − bn | ≤ |an − cn | for all n, and so
|an − bn | < µ for n > n1 . We also know, since limn→∞ an = L, that for every
η > 0, there exists n2 so that |an − L| < η for n > n2 .

So, for given ε > 0, choose n1 so that |an − bn | < 21 ε for all n > n1 , and choose
n2 so that |an − L| < 21 ε for n > n2 . Then, for n > n0 = max{n1 , n2 }), we
have that
1 1
|bn − L| = |bn − an + an − L| ≤ |bn − an | + |an − L| < ε + ε = ε,
2 2
as desired. 
Definition 4.18. A sequence (an ) is
• monotone increasing (or just increasing) if an < an+1 for all n
(each term is strictly bigger than the preceding term);
• monotone non-decreasing (or just non-decreasing) if an ≤ an+1
for all n (each term is not smaller than the preceding term);
• monotone decreasing (or just decreasing) if an > an+1 for all n
(each term is strictly smaller than the preceding term);
• monotone non-increasing (or just non-increasing) if an ≥ an+1
for all n (each term is not bigger than the preceding term);
MATH2039 ANALYSIS 23

• A sequence (an ) is monotone or monotonic if it is one of these four.

Think of a non-decreasing sequence as an increasing sequence with a stutter,


and similarly of a non-increasing sequence as a decreasing sequence with a
stutter.
We will now define the real numbers. As mentioned in the previous section, we
require one more property of the real numbers, completeness. In the previous
section we mentioned that one can define completeness in terms of suprema,
but we will use the following definition. We shall see later, Theorems 5.1 and
5.3, that the two definitions are equivalent.
Definition 4.19. The real numbers, denoted R, is an ordered field satisfying
the following additional axiom:

Completeness Axiom: Every bounded monotonic sequence in R converges


to a limit in R. We say that R is a complete ordered field.
Exercise 4.20. Show that the rationales Q with their usual order < form
an ordered field but not a complete ordered field (i.e. they do not satisfy the
completeness axiom).

We can use the completeness axiom to prove convergence of sequences.


Example 4.21. Let an = 1/n, n ∈ N. Show that (an )∞
n=1 is a convergent
sequence.

Proof. First we show that (an ) is a decreasing sequence:


1 n+1 n 1
an = = > = = an+1 .
n n(n + 1) n(n + 1) n+1
Since (an ) is decreasing, it is bounded above by a1 = 1. On the other hand as
n is positive, an = 1/n is also positive, so the sequence is bounded below by 0.
Since the sequence (an ) is bounded and monotonic, it converges by the com-
pleteness axiom. 

We can use the following result to prove that in fact (1/n) converges to zero
(as we might expect it to!).
Lemma 4.22. Let (an ) be a sequence and c ∈ R. If (an ) converges to L as
n → ∞ then the sequence (bn ), bn = can converges to cL as n → ∞.

Proof. We must prove


∀ε > 0 ∃n0 such that ∀n ∈ I, n > n0 =⇒ |can − cL| < ε.
Note that |can − cL| = |c||an − L|, thus (at least if c 6= 0) we have |can − cL| < ε
if an only if |an − L| < ε/|c|. We know that an converges to L, i.e.
∀η > 0 ∃n0 such that ∀n, n > n0 =⇒ |an − L| < η.
Since this holds for all η, given any ε > 0 we can choose η to be ε/|c| if c 6= 0,
and choose η to be 1 (or anything else) if c = 0.
24 PERIŠIĆ

Then there exists n0 such that n > n0 =⇒ |an − L| < η. For c 6= 0, η = ε/|c|
and so
n > n0 =⇒ |an − L| < η =⇒ |can − cL| < ε
as required, which proves that can → cL as n → ∞. For c = 0, we always have
|can − cL| = 0 < ε, so in particular this holds for n > n0 , and so can → cL = 0
as n → ∞. 
Example 4.23. Let an = 1/n, n ∈ N. Then (an )∞
n=1 converges to 0 as n → ∞.

Proof. We know from the above example that the sequence (an ) converges to
1
some limit L. Let bn = a2n = 2n . The sequence (bn ) is a subsequence of
(an ), so it also converges to L. On the other hand bn = 12 · n1 = 12 an , so by
the previous lemma (bn ) converges to 12 L. Thus 12 L = L so L = 0. This uses
Lemma 4.11. 
Definition 4.24. Cauchy criterion A sequence (an )n∈I is a Cauchy se-
quence if for every ε > 0, there exists n0 ∈ I such that |ap − aq | < ε for all p,
q ∈ I with p, q > n0 .
Theorem 4.25. Let (an )n∈I be a sequence. If (an ) converges then (an ) is a
Cauchy sequence.

Proof. Since (an ) converges, write limn→∞ an = L, so that for every ε > 0
there exists n0 ∈ I so that |an − L| < 21 ε for n > n0 . Now, for p, q ∈ I and p,
q > n0 , consider the difference |ap − aq | :
1 1
|ap − aq | = |ap − L + L − aq | ≤ |ap − L| + |L − aq | < ε + ε = ε,
2 2
as desired. 
Remark 4.26. The Cauchy1 criterion is a very useful test for divergence. If
a sequence converges then it is a Cauchy sequence. Thus if a sequence is not
a Cauchy sequence then it cannot converge i.e. it diverges.
If (an ) is a sequence, and there exists ε > 0 so that for every n0 , there are p,
q > n0 with |ap − aq | ≥ ε, then the Cauchy criterion fails for (an ), hence (an )
diverges.
Example 4.27. Let an = (−1)n . Then for every n0 we can pick p, q > n0 such
that |ap − aq | = 2. Specifically, for any n0 if we take p = n0 + 1 and q = n0 + 2
then one of p, q is even, and the other is odd, hence ap − aq = ±2. Hence the
Cauchy criterion fails for ε = 2, so the sequence (an ) diverges.
Example 4.28. Let an = n. Then for every n0 we can pick p, q > n0 such
that |ap − aq | = 1. Specifically, for any n0 if we take p = n0 + 1 and q = n0 + 2
then |ap − aq | = |p − q| = 1. Hence the Cauchy criterion fails for ε = 1, so (an )
diverges.

We can use this argument to prove the Archimedean property for R.


1For a brief historic note on Cauchy see Appendix 0
MATH2039 ANALYSIS 25

Proposition 4.29. The real numbers R satisfy the Archimedean property:


Given any x, y ∈ R with x > 0 and y > 0 there exists n ∈ N such that
n · x > y.

This means that there are no infinitely large or infinitesimally small real num-
bers.

Proof. Suppose this is not the case, that is for some x, y > 0 we have nx ≤ y
for all n ∈ N. We define a sequence an = nx. Since x > 0 this is an increasing
sequence: an+1 = nx + x > nx = an . It is also a bounded sequence. It is
bounded below by 0, since x is positive, and by assumption it is bounded above
by y. Therefore, by the completeness axiom, the sequence converges.
On the other hand for every n0 we can pick p, q > n0 such that |ap − aq | = x.
Specifically, for any n0 if we take p = n0 + 1 and q = n0 + 2 then |ap − aq | =
|px − qx| = x. Hence the Cauchy criterion fails for ε = x, so (an ) diverges,
which is a contradiction. Thus, we conclude that there are no numbers x, y > 0
such that nx ≤ y for all n ∈ N. 

A sequence can diverge in a number of different ways. There are two particularly
useful ways that this can happen.
A sequence (an )n∈I diverges to infinity if, for every R > 0 there exists n0 ∈ I
such that an > R for every n ∈ I with n > n0 .

Symbolically, the sentence ‘an diverges to ∞’ is denoted either by an → ∞ as


n → ∞ or by limn→∞ an = ∞.

Similarly, a sequence (an )n∈I diverges to minus infinity if, for every R > 0
there exists n0 ∈ I such that an < −R for every n ∈ I with n > n0 .

Symbolically, the sentence ‘an diverges to −∞’ is denoted either by an → −∞


as n → ∞ or by limn→∞ an = −∞.
Remark 4.30. We are not saying that ∞ and −∞ are real numbers. As we
have seen there are no infinitely large or infinitely small numbers in R. The
symbols an → ∞ etc. indicate properties of the sequence of finite numbers
(elements of R), rather than describing ‘infinite numbers.’

As previously mentioned there are many different ways for a sequence


to diverge: it could diverge to ±∞, it could oscillate (for example (−1)n ) or
a combination of both (for example (−1)n · n).
Example 4.31. The sequence (an ), an = n, n ∈ N diverges to infinity.

Proof. We must show that for all R ∈ R with R > 0 there exists n0 ∈ N such
that an = n > n0 implies n > R. Given R > 0 apply the Archimedean property
to the numbers 1, R. Then there exists n0 ∈ N such that n0 · 1 > R. Now for
any n > n0 we have an = n > n0 > R. 
26 PERIŠIĆ

Definition 4.32. The integer part of a real number x is the greatest integer
n such that n ≤ x. This is denoted bxc, and is sometimes called the floor
function.
Proposition 4.33. The floor function is well defined, i.e. for every x ∈ R
there exists a greatest integer n such that n ≤ x.

Proof. If x > 0 then by the Archimedean property for the numbers 1, x, we


know that there is some natural number n1 with n1 = n1 · 1 > x. On the other
hand if x ≤ 0 then n1 = 1 > x for all x, thus for any x in R there exists n1 ∈ N
with n1 > x. Similarly there exists n0 ∈ N with n0 > −x, or equivalently
−n0 < x.
Now let S = {n ∈ Z : n ≤ x} be the set of integers n such that n ≤ x. We
note that this is nonempty, since −n0 < x. The set T of n ∈ S with n ≥ −n0
is finite – it contains at most n1 + n0 elements, since if n ∈ S then n ≤ x < n1 .
As T is finite it has a maximum m, by proposition 2.18. The number m is also
a maximum for the set S of integers n such that n ≤ x, since m ∈ T ⊂ S, and
if n ∈ S then either n ∈ T , so that n ≤ m, or n < −n0 ≤ m since −n0 ∈ T .
The set S thus has a greatest element (a maximum) as required. 
Example 4.34. The sequence an = 1/n converges to zero. (Second proof.)
To say an → 0 means that for every ε > 0 there exists n0 ∈ N such that
n > n0 =⇒ |1/n − 0| = 1/n < ε.
Note that 1/n < ε ⇐ 1/ε < n. Let n0 = b1/εc. If n ∈ N, n > n0 then n > 1/ε
so 1/n < ε as required.
3n−1
Exercise 4.35. A sequence (un ) has its nth term given by un = 4n−5 . Write
st th th th th th th
the 1 , 5 , 10 , 100 , 1000 , 10, 000 , and 100, 000 term of the sequence
in decimal form. Make a guess to the limit of this sequence as n → ∞. Using
the definition of limit, verify that the guess you’ve made is correct.
Example 4.36. Using the definition of limit, we show that n−4 1
5n → 5 for
n−4 1 1 4 1 4 4
n → ∞. We begin by considering | 5n − 5 | = | 5 − 5n − 5 | = | − 5n | = | 5n |.
4 4
Now, for arbitrary ε > 0 the positive integer n0 = b 5ε c satisfies that 5n < ε for
all n > n0 .
For example, supposing we want to find an n0 so | n−4 1
5n − 5 | < 0.0005 for all
4
n > n0 , we take n0 = b 5·0.0005 c = 1, 600.
n
Exercise 4.37. Using the definition of limit, prove that limn→∞ 1+2·10 2
5+3·10n = 3 .
1+2·10n 2 −3
For what value of n0 do we have that | 5+3·10n − 3 | < 10 for all n > n0 ?

The following method can be used to test whether a sequence (an ) is monotonic:
we know from first year calculus that a function with non-negative derivative is
increasing (that is, if f is a function with f 0 (x) ≥ 0 for all x (in its domain) and
if a < b, then f (a) < f (b)), and that a function with non-positive derivative is
decreasing (that is, if f is a function with f 0 (x) ≤ 0 for all x (in its domain)
and if a < b, then f (a) > f (b)).
MATH2039 ANALYSIS 27

So, suppose we can find a function f (x) so that an = f (n) and so that f 0 (x) ≥ 0.
Then, the sequence an is monotonically non-decreasing. (If f 0 (x) > 0, then an
is monotonically increasing.) Similarly, if f 0 (x) ≤ 0, then an is monotonically
non-increasing. (If f 0 (x) < 0, then an is monotonically decreasing.)
1 1
Exercise 4.38. Prove that n+1 < ln(n + 1) − ln(n) < n for all n ∈ N.
Pn 1 
Now, consider the sequence (an ) given by an = i=1 i − ln(n). Prove that
(an ) is a decreasing sequence and that each an is positive. Conclude that the
limit γ = limn→∞ an exists. [This number γ is known as Euler’s constant,
and little is known about it. For instance, it is not known whether γ is rational
or irrational.]
Exercise 4.39. Explain exactly what is meant by the following statements:
(1) limn→∞ 32n−1 = ∞;
(2) limn→∞ (1 − 2n) = −∞;
(3) limn→∞ e−n = 0;
Theorem 4.40. Inequalities of sequences: Let (an )n∈I be a sequence con-
verging to a and let (bn )n∈I be a sequence converging to b. If an ≤ bn for all n,
then a ≤ b.
Corollary 4.41. Let (an ) be a convergent sequence with an in the interval [a, b]
for all n ∈ I. Then limn→∞ an ∈ [a, b].

Proof of theorem 4.40. Suppose b < a, and let ε = a−b 2 . As an → a, there


exists n0 such that n > n0 implies |an − a| < ε, i.e. −ε < an − a < ε. Taking
the first inequality we see that n > n0 implies a − ε < an .
As bn → b, there exists n1 such that n > n1 implies |bn − b| < ε, i.e. −ε <
bn − b < ε. Taking the second inequaltiy we see that n > n1 implies bn < b + ε.
Now take n greater than both n0 and n1 . Then bn < b + ε and a − ε < an . But
a+b
b+ε= =a−ε
2
so we have bn < a+b
2 < an . This contradicts the assumption that an ≤ bn , so
we deduce that a ≤ b. 

Remark 4.42. Standard tricks for evaluating limits: Some of these are
facts that we will prove later in the course but which are nonetheless very useful
to know now.
(1) exponents: for c > 0, if the sequence (an ) converges to a, then the
sequence (can ) converges to ca ;
(2) trigonometric functions: if the sequence (an ) converges to a, then
the sequence (cos an ) converges to cos a, and (sin an ) converges to sin a.

We shall now derive some basic results on limits of various useful sequences.
We begin with the following fact, due to Bernoulli.
28 PERIŠIĆ

Theorem 4.43. (The Bernoulli inequality) For all a ≥ 0 and all n ∈ N:


(1 + a)n ≥ 1 + na

Proof. From the binomial theorem:


Pn n k

(1 + a)n = k=0 k a 
= 1 + na + n2 a2 + · · · + an
≥ 1 + na
The last step follows because we have assumed that a is nonnegative, so all the
terms that we throw away are greater than or equal to zero, and this makes the
right hand side smaller. 

We shall use Bernoulli’s inequality quite a lot in what follows. For now we shall
derive some simple facts about limits.
1
Example 4.44. If limn→∞ an = ±∞ then limn→∞ an = 0.

Proof. Assume that limn→∞ an = ∞. This means that for any ε > 0 there
exists an n0 such that for all n > n0 , an > 1/ε > 0. But this implies that
1
0 < 1/an < ε, for all n > n0 , i.e. |1/an | < ε and this implies that lim = 0.
n→∞ an

A similar argument works for lim an = −∞. 


n→∞

n
The converse of this statement is false. Consider the sequence an = (−1)
n .
Then lim an = 0 but the sequence 1/an = (−1)n n does not diverge to either
n→∞
∞ or −∞ (though it does diverge).
Example 4.45. For all q > 0
lim nq = ∞
n→∞

Proof. Take any R > 0. Then for n > n0 = bR/qc we have n > R/q so nq > R
and the result follows. 
Example 4.46. If a > 1, then lim an = ∞.
n→∞

Proof. As a > 1 we can write a = 1 + c for some c > 0. Then an = (1 + c)n ≥


1 + nc, by the Bernoulli inequality 4.43. But nc → ∞ by the previous example
4.45, so an → ∞. 
Example 4.47. If |q| < 1 then lim q n = 0.
n→∞

1
Proof. First we let 0 < q < 1. Then 1/q > 1 and lim ( )n = ∞ and so
n→∞ q
lim q n = 0.
n→∞

Thus for |q| < 1,


lim |q|n = 0 = lim |q n |
n→∞ n→∞
MATH2039 ANALYSIS 29

But we also have


−|q n | ≤ q n ≤ |q n |
so it follows from the squeeze rule that lim q n = 0. 
n→∞

n
Example 4.48. For all a > 0, lim a = 1.
n→∞


Proof. Assume first that a > 1. We shall show that the sequence (un ), un = n a
is decreasing. As a > 1, we have that an < an+1 . Now take the n(n + 1) root
of both sides of this inequality:

n(n+1)

n(n+1)
an < an+1
to get √ √
n+1
a< na
hence un+1 < un . Now assume that inf{un : n ∈ N} = 1 + h, where h ≥ 0.
This is a reasonable assumption as un > 1 for all n. But then 1 + h is a lower
bound of the set {un } so

1 + h ≤ n a =⇒ (1 + h)n ≤ a
Using the Bernoulli inequality again we have that
1 + nh ≤ (1 + h)n < a ∀n
If h > 0 then nh → ∞ , which is impossible, as the left hand side has to be
smaller than a fixed number a. So we must have that h = 0 and so inf{un } = 1,
hence lim un = 1.
n→∞

Now assume 0 < a ≤ 1, so 1/a ≥ 1. Then by the first part of this proof,
lim (1/a)1/n = 1, or in other words:
n→∞
1 √
n
lim = 1 =⇒ lim a=1
n→∞ a1/n n→∞

Example 4.49. If |q| < 1 then
1
lim (1 + q + q 2 + · · · + q n ) =
n→∞ 1−q

Proof. We have
1 + q + q 2 + · · · + q n = q(1 + q + · · · + q n−1 ) + 1
so
(1 + q + q 2 + · · · + q n−1 )(1 − q) = 1 − q n
which gives
1 − qn
1 + q + q 2 + · · · + q n−1 =
1−q
Now use the fact that q n → 0 as n → ∞ for |q| < 1 to get the result. 
Example 4.50. For the two sequences
√ √
(1) (an ), an = n + 5 − n
30 PERIŠIĆ



(2) (an ), an = sin 4

do the following:
• Determine whether the sequence converges or diverges;
• if the sequence converges, determine its limit;
• if the sequence diverges, determine whether the sequence diverges to
∞, diverges to −∞ or neither.
√ √
(1) an = n + 5 − n;

converges: we use the standard first method to try for evaluating


limits of differences of square roots. Write
√ √
√ √ √ √ n+5+ n 5
an = n + 5 − n = ( n + 5 − n) √ √ =√ √ .
n+5+ n n+5+ n
√ √ √ √
Then,√since n√+ 5 + n > n and since n → ∞ as n → ∞, we have
that n + 5 + n → ∞ as n → ∞, and hence (an ) converges to 0.
(2) an = sin nπ

4 ;

8kπ

diverges: for nk = 8k, k ∈ N, ank =
 sin 4 = sin(2kπ) = 0, and
for nk = 8k + 2, k ∈ N, ank = sin (8k+2)π
4 = sin 2kπ + π2 = 1.
Hence, |a8k − a8k+2 | = 1 for all k ∈ N, and so the sequence (an ) fails
the Cauchy criterion, and hence (an ) diverges.
Exercise 4.51. The sequence scavenger hunt: for each sequence (an ), do
the following:
• Determine whether the sequence converges or diverges;
• if the sequence converges, determine its limit;
• if the sequence diverges, determine whether the sequence diverges to
∞, diverges to −∞ or neither.
(1) an = (n + 2)1/n ;
n2 +3n+2
(2) an = 6n3 +5 ;

(3) an = (1 + n1 )n ;
sin(n)
(4) an = 3n ;
√ √
(5) an = 2n + 3 − n + 1;
(6) an = cos nπ

4 ;

(7) an = (1 + n1 )1/n ;
(8) an = ln(n);
(9) an = en ;
MATH2039 ANALYSIS 31

ln(n)
(10) an = √ ;
n

2 n

(11) an = 1 − n2 ;
n3
(12) an = 10n2 +1 ;

(13) an = xn , where x is a constant with |x| < 1;


c
(14) an = np , where c 6= 0 and p > 0 are constants;
2n
(15) an = 5n−3 ;

1−n2
(16) an = 2+3n2 ;

n3 −n+7
(17) an = 2n3 +n2 ;
9 n

(18) an = 1 + 10 ;
n
− 12 ;

(19) an = 2 −
(20) an = 1 + (−1)n ;
1+(−1)n
(21) an = n ;

1+(−1)n n
(22) an = n ;
( 32 )
sin2 (n)
(23) an = √
n
;
q
2+cos(n)
(24) an = n ;

(25) an = n sin(πn);
(26) an = n cos(πn);
(27) an = π − sin(n)/n ;
(28) an = 2cos(πn) ;
ln(2n)
(29) an = ln(3n) ;

ln2 (n)
(30) an = n ;
1

(31) an = n sin n ;
arctan(n)
(32) an = n ;
n3
(33) an = en/10
;
2n +1
(34) an = en ;
sinh(n)
(35) an = cosh(n) ;

(36) an = (2n + 5)1/n ;


 n
(37) an = n−1
n+1 ;
32 PERIŠIĆ

(38) an = (0.001)−1/n ;
(39) an = 2(n+1)/n ;
3/n
(40) an = n2 ;
(41) an = (−1)n (n2 + 1)1/n ;
n
( 23 )
(42) an = n 9 n;
( ) +( 10
1
2 )
Exercise 4.52. The Fibonacci sequence {an : n ≥ 0} is formed by setting
a0 = 0, a1 = 1, and an = an−1 + an−2 for n ≥ 2. Consider the derived sequence
an
qn = an−1 of quotients of consecutive terms of the Fibonacci2 sequence. Show

1+ 5
that if limn→∞ qn exists, then limn→∞ qn = 2 .

Example 4.53. Prove that if xn → −4 as n → ∞, then |xn | → 4 as n → ∞,


using the definition of limit.

We need to show that limn→∞ |xn | = 4, which is phrased mathematically as


needing to show that for each ε > 0, there is n0 so that | |xn | − 4| < ε for
n > n0 . First, note that if we take ε = 1, we have that there exists n1 so that
|xn − (−4)| < 1 for n > n1 . In particular, for n > n1 we have that xn < 0
(since for n > n1 it lies in the interval of radius 1 centred about −4, which is
the interval (−5, −3)). In particular, for n > n1 , we have that |xn | = −xn .

So, for n > n1 , we have that | |xn | − 4| = | − xn − 4| = |xn + 4|, and we have
been given that for any ε > 0, there is n2 so that | |xn | − 4| = | − xn − 4| =
|xn + 4| < ε for n > n2 . So, for any ε > 0, take n0 to be the larger of n1
(chosen so that xn < 0 for n > n1 ) and n2 (which comes from the definition
that limn→∞ xn = −4). Then, for n > n0 , we have that | |xn | − 4| < ε, and we
are done.
Exercise 4.54. Prove that each of the following statements is true, using the
definition of limit.
p
(1) if xn → −4 as n → ∞, then |xn | → 2 as n → ∞;
(2) if xn → −4 as n → ∞, then x2n → 16 as n → ∞;
xn
(3) if xn → −4 as n → ∞, then 3 → − 34 as n → ∞;
Exercise 4.55. Let (an ) be a sequence converging to a. Show that the following
holds:
√ √
(1) square roots: if a > 0, then an converges to a;
(2) |an | converges to |a|;
(3) If a 6= 0, then (−1)n an diverges;
x1 +···+xn
Exercise 4.56. Prove that if xn → x as n → ∞, then n → x as n → ∞.

2See Appendix 0 Historic notes


MATH2039 ANALYSIS 33

5. Completeness properties of the reals

We shall begin by showing that two completeness properties of the reals we


have used in the provious two sections are equivalent. Firstly we know that the
reals satisfy the following (ordered) completeness axiom.
Completeness Axiom: Every bounded monotonic sequence in R converges
to a limit in R.
Secondly, in Section 2 we have seen that one can also define completeness via
bounded set, i.e. that every set, which is bounded above has a sumpremum.
In the next theorem we shall see that this is implied by the completeness ax-
iom.
Theorem 5.1. Every non-empty subset A of R which is bounded above has
a supremum. Every non-empty subset A of R which is bounded below has an
infimum.

Proof. We will prove the first statement. The second is proved similarly (or
by noting that −A = {−a|a ∈ A} is bounded above so has a supremum, and
inf A = − sup(−A)).
The proof uses a bisection argument. As A is nonempty, there exists some
a ∈ R which is not an upper bound for A (e.g. take a = x − 1 for any x ∈ A).
As A is bounded above there exists an upper bound b for A. Clearly a < b.
Now let a0 = a and b0 = b. Let c0 = a0 +b 2 . Either c0 is an upper bound for A
0

(like b0 ) or c0 is not an upper bound for A (like a0 ). If c0 is an upper bound


then we define a1 = a0 , b1 = c0 otherwise we define a1 = c0 , b1 = b0 . We now
proceed by induction: given an , bn let cn = an +b
2
n
and if cn is an upper bound
then we define
an+1 = an , bn+1 = cn
otherwise we define
an+1 = cn , bn+1 = bn .
For each n we have the following properties:
• an ∈ [a, b], bn ∈ [a, b]; in particular an and bn are bounded sequences.
• an < cn < bn , so an ≤ an+1 = an or cn and bn ≥ bn+1 = bn or cn .
( )
cn − an = an +b
2
n
− an or bn − an
• bn+1 − an+1 = an +bn = .
bn − c n = bn − 2 2
Thus the sequences an and bn are bounded and monotonic so they converge.
Moreover as bn+1 − an+1 = bn −a
2
n
for each n, we note that bn − an → 0 as
n → ∞ so limn→∞ an = limn→∞ bn .
Let s = limn→∞ an = limn→∞ bn . For any x ∈ A we have x ≤ bn for all n
so x ≤ limn→∞ bn = s, by corollary 4.41. Thus s is an upper bound for A.
On the other hand if u is any upper bound for A then u ≥ an for all n, so
u ≥ limn→∞ an = s. Since s ≤ u for any upper upper bound u, it follows that
s is the least upper bound as required. 
34 PERIŠIĆ

Proposition 5.2. Let (an )n∈I be a convergent sequence. Then {an : n ∈ I} is


a bounded set.

Proof. Set a = limn→∞ an and apply the definition of limit of a sequence with
ε = 1. Then there exists n0 ∈ I so that |an − a| < 1 for all n > n0 . In
particular, for n > n0 , we have that an lies in the interval (a − 1, a + 1). The set
{a1 , . . . , an0 , a + 1} is finite, so it has a maximum s. We have an < a + 1 ≤ s
for all n > n0 while for n ≤ n0 , an lies in the set so an ≤ s. Thus {an : n ∈ I}
is bounded above by s.

Similarly, let t be the minimum of the finite set {a1 , . . . , an0 , a − 1}, and note
that t ≤ an for all n, so that {an : n ∈ I} is bounded below by t.

Since {an : n ∈ I} is both bounded below and bounded above, it is bounded.


(Note that the choice of ε = 1 is completely arbitrary. Any positive number
will work.) 

The following theorem shows that the alternative definition of completeness


indeed is equivalent to our axiom above.
Theorem 5.3. Let F be an ordered field where every bounded set has an infi-
mum and a supremum. Then every bounded monotone sequence in F converges
in F .

Proof. Let (an )n∈I be a bounded monotone sequence. There are two cases:
either (an ) is monotone non-increasing (which includes the case of monotone
decreasing) or is monotone non-decreasing (which includes the case of monotone
increasing). Suppose to start that (an ) is monotone non-decreasing. Let A =
{an |n ∈ I}, and let s = sup(A).

Since s = sup(A), we know that an ≤ s for all an ∈ A and also that for each
ε > 0, there exists some n0 so that |s − an0 | = s − an0 < ε. (Because, if there
is some ε0 > 0 so that s − an ≥ ε0 for all n, then s − ε0 is an upper bound for
A which is smaller than s, contradicting the choice of s = sup(A).)

However, since (an ) is monotonically non-decreasing, we have that an ≥ an0


for all n > n0 , and so we have that s ≥ an ≥ an0 for all n > n0 . In particular,
|s − an | < |s − an0 | < ε for all n > n0 , and so an satisfies the definition of
limn→∞ an = s.

The proof in the case that an is monontonically non-increasing is the same,


except that inf(A) takes the place of sup(A). 

Exercise 5.4. Give five different examples of sequences that are bounded but
not convergent.
MATH2039 ANALYSIS 35

We will give two more important completeness results. Bolzano-Weierstrass3


Theorem states that every bounded sequence of real numbers has a convergent
subsequence.
Theorem 5.5. (Bolzano-Weierstrass Theorem) Let (xn )n∈I be a sequence
of real numbers with xn ∈ [a, b] a closed bounded interval. Then there is a
subsequence (xnk ) such that (xnk ) converges to a limit in [a, b] as k → ∞.

Proof. The proof uses a bisection argument. We will bisect the interval [a, b] in
such a way that each subinterval [ak , bk ] that we construct contains infinitely
many of the terms of the original sequence xn .
Let a0 = a and let b0 = b. For each k = 1, 2, . . . , we suppose that we have
defined ak−1 , bk−1 such that [ak−1 , bk−1 ] contains xn for infinitely many val-
ues of n. Then either [ak−1 , ak−1 +b 2
k−1
] contains xn for infinitely many n, or
ak−1 +bk−1
[ 2 , bk−1 ] contains xn for infinitely many n (or both). If [ak−1 , ak−1 +b
2
k−1
]
ak−1 +bk−1
contains xn for infinitely many n then we define ak = ak−1 , bk = 2 ,
otherwise we define ak = ak−1 +b 2
k−1
, bk = bk−1 . Hence we have defined ak , bk
such that [ak , bk ] contains xn for infinitely many n.
Note that at each stage we have ak−1 ≤ ak−1 +b
2
k−1
≤ bk−1 . We define ak = ak−1
ak−1 +bk−1 ak−1 +bk−1
or ak = 2 so a k ≥ ak−1 , and we define bk = 2 or bk = bk−1
so bk ≤ bk−1 . The sequences ak , bk are bounded (as they lie in [a, b]) and
monotonic, so they converge. Moreover, at each stage bk − ak = bk−1 −a 2
k−1
,
so their limits are equal: limk→∞ ak = limk→∞ bk = L for some L. Note that
L ∈ [a, b] by inequality of sequences.
Now for each k we can pick nk such that nk > nk−1 and xnk ∈ [ak , bk ]. By
assumption there are infinitely many possible choices of nk , so to be specific,
let nk be the least integer satisfying these conditions. By construction ak ≤
xnk ≤ bk , so as (ak ), (bk ) converge to L, so does (xnk ) by the squeeze rule. We
have thus constructed a subsequence (xnk ) of (xn ), such that (xnk ) converges
to a limit L in [a, b] as required. 
Definition 5.6. A set S ⊆ R is sequentially compact if for every sequence
(xn ), xn ∈ S, there is a subsequence (xnk ) which converges to a limit in S.

The Bolzano-Weierstrass theorem states that [a, b] is sequentially compact.


Exercise 5.7. Which of the following sets is sequentially compact.
(1) S = R;
(2) S = [0, 1);
(3) S = [0, 1] ∪ [1, 2];
(4) (Harder) S = the is the middle-third Cantor set obtained by delet-
ing the middle third of [0, 1], then deleting the middle third of the
remaining intervals, and repeating ad infinitum: S = [0, 1] \ 13 , 23 \


3See Appendix 0
36 PERIŠIĆ

1 2 7 8 1 2 7 8 19 20 25 26
     
9, 9 ∪ 9, 9 \ 27 , 27 ∪ 27 , 27 ∪ 27 , 27 ∪ 27 , 27 \ ...

Hint: Let (u, v) be one of the deleted intervals. Can a sequence in S


converge to a point in (u, v)?

We now give one final completeness property of R. Recall that a sequence


(an )n∈I is a Cauchy sequence if for every ε > 0, there exists n0 ∈ I such
that
|ap − aq | < ε for all p, q ∈ I with p, q > n0 .

We know that if a sequence converges then it is Cauchy. We will now prove the
converse of this.
Theorem 5.8. (General principle of convergence) Suppose that (an )n∈I
is a Cauchy sequence in R. Then (an ) converges in R.

Proof. First we show that as (an ) is a Cauchy sequence, it is bounded. We apply


the definition of a Cauchy sequence with ε = 1. Thus there exists n1 ∈ I so that
for all p, q > n1 we have |ap −aq | < 1. Fix q > n1 . The set {a1 , a2 , . . . an1 , aq +1}
is finite, thus it has a maximum element b. For all n we have either n ≤ n1 , so
an is in the set or n > n1 whence |an − aq | < 1, so that an < aq + 1. In either
case we have an ≤ b.
Similarly, the set {a1 , a2 , . . . an1 , aq − 1} is finite so it has a minimum a, and for
all n we have an ≥ a. We deduce that as an is a Cauchy sequence there exists
an interval [a, b] such that an ∈ [a, b] for all n.
Now apply the Bolzano-Weierstrass theorem. This provides a subsequence (ank )
of (an ) with the property that (ank ) converges to L for some L ∈ [a, b]. We will
show that the original sequence (an ) converges to L. This is an ε/2 argument.
As (an ) is a Cauchy sequence, there exists n0 such that for all p, q > n0 we
have |ap − aq | < ε/2. On the other hand, as ank converges to L, there exists
k0 , such that for all k > k0 we have |ank − L| < ε/2. Pick k > k0 with nk > n0
(this holds for all k sufficiently large). Let q = nk . Then for n > n0 we have
|an − ank | = |an − aq | < ε/2. As k > k0 we also have |ank − L| < ε/2. Thus for
n > n0 we have
|an − L| ≤ |an − ank | + |ank − L| < ε/2 + ε/2 = ε
which completes the proof. 
MATH2039 ANALYSIS 37

6. Continuity

We now briefly recall the definition of the limit of a function.


Definition 6.1. (limit of a function) Let f : S → R be a function with
domain S ⊆ R. The limit of f as x → a is L , denoted limx→a f (x) = L, if
∀ε > 0 ∃δ > 0 such that if x ∈ S, 0 < |x − a| < δ then |f (x) − L| < ε.

Remark 6.2. Note that the definition of limx→a f (x) = L includes the re-
quirement that 0 < |x − a|, and so the limit does not depend on the value of
f (a). For example, limx→o sinx x = 1 but sinx x is even not defined at x = 0.

The definitions of limx→∞ f (x) = L, limx→−∞ f (x) = L, limx→a f (x) = ∞,


and limx→a f (x) = −∞ are similar:
• limx→∞ f (x) = L: for every ε > 0, there exists M > 0 so that if x ∈ S
and x > M , then |f (x) − L| < ε;
• limx→−∞ f (x) = L: for every ε > 0, there exists M < 0 so that if x ∈ S
and x < M , then |f (x) − L| < ε;
• limx→a f (x) = ∞: for every N > 0, there exists δ > 0 so that if x ∈ S
and 0 < |x − a| < δ, then f (x) > N ;
• limx→a f (x) = −∞: for every N < 0, there exists δ > 0 so that if x ∈ S
and 0 < |x − a| < δ, then f (x) < N ;
We can also define left and right-handed limits:
• right-handed limit: The limit of f as x tends to a from the right is
L, written limx→a+ f (x) = L if
∀ε > 0 ∃δ > 0 such that if x ∈ S, 0 < x − a < δ then |f (x) − L| < ε.
This depends only on values of f (x) with x − a > 0 i.e. x > a.
• left-handed limit: The limit of f as x tends to a from the left is L,
written limx→a− f (x) = L if
∀ε > 0 ∃δ > 0 such that if x ∈ S, −δ < x − a < 0 then |f (x) − L| < ε.
This depends only on values of f (x) with x − a < 0 i.e. x < a.
For completeness, the definitions of
lim f (x) = ∞, lim f (x) = −∞,
x→a+ x→a+

lim f (x) = ∞, and lim f (x) = −∞


x→a− x→a−
are similar:
• limx→a+ f (x) = ∞: for every N > 0, there exists δ > 0 so that if
0 < x − a < δ, then f (x) > N ;
• limx→a+ f (x) = −∞: for every N < 0, there exists δ > 0 so that if
0 < x − a < δ, then f (x) < N ;
38 PERIŠIĆ

• limx→a− f (x) = ∞: for every N > 0, there exists δ > 0 so that if


0 < a − x < δ, then f (x) > N ;
• limx→a− f (x) = −∞: for every N < 0, there exists δ > 0 so that if
0 < a − x < δ, then f (x) < N ;

Note that the left and right-hand limits simply restrict the domain of f to either
{x ∈ S : x < a} or {x ∈ S : x > a}.

Exercise 6.3. limx→a f (x) = L if and only if


lim f (x) = lim f (x) = L.
x→a+ x→a−

The basic properties of limits of sequences also hold for limits of functions. The
proofs are simple modifications of the corresponding results for sequences.
Theorem 6.4. Let f, g, h : S → R be functions with common domain S.
If limx→a f (x) = L and limx→a g(x) = M then

(i) sum: limx→a (f + g)(x) = limx→a f (x) + limx→a g(x) = L + M ;

(ii) difference: limx→a (f − g)(x) = limx→a f (x) − limx→a g(x) = L − M ;

(iii) product: limx→a (f · g)(x) = limx→a f (x) · limx→a g(x) = L · M ;

1 1 1
(iv) reciprocal: if L 6= 0, then limx→a f (x) = limx→a f (x) = L;

f (x) limx→a f (x) L


(v) quotient: if M 6= 0, then limx→a g(x) = limx→a g(x) = M

(vi) squeeze rule:


If f (x) ≤ g(x) ≤ h(x), ∀x ∈ S and if limx→a f (x) = limx→a h(x) = L,
then
lim g(x) = L.
x→a

Definition 6.5. Let f : S → R be a function. The function f is continuous


at a if
a ∈ S and ∀ε > 0 ∃δ > 0 such that ∀x ∈ S |x − a| < δ =⇒ |f (x) − f (a)| < ε.

Note that this definition implies the following: The function f is defined at a,
the limit limx→a f (x) exists, and limx→a f (x) = f (a).
MATH2039 ANALYSIS 39

Definition 6.6. A function f : S → R with domain S is continuous if it is


continuous at each point a in S. In other words f is continuous if
∀a ∈ S ∀ε > 0 ∃ δ > 0 such that ∀x ∈ S |x − a| < δ =⇒ |f (x) − f (a)| < ε.
Exercise 6.7. Prove, using the definition, that each of the following functions
is continuous at all points of R.
(1) hn (x) = xn , where n ∈ N;
(2) g(x) = c, where c ∈ R;
(3) f is a function on R which satisfies |f (x) − f (y)| ≤ c|x − y| for all x,
y ∈ R, where c > 0 is a constant. (Functions that satisfy this condition
are often referred to as Lipschitz functions.)

Since continuous functions are defined in terms of limits, the rules of arithmetic
for limits of functions, as given in Theorem 6.4, extend immediately to rules of
arithmetic for continuous functions.
Theorem 6.8. Let f, g : S → R be functions that are continuous at a ∈ S.
Then the following holds:
(1) the sum f +g, defined by setting (f +g)(x) = f (x)+g(x), is continuous
at a;
(2) the difference f − g, defined by setting (f − g)(x) = f (x) − g(x), is
continuous at a;
(3) the product f ·g, defined by setting (f ·g)(x) = f (x)g(x), is continuous
at a;
(4) if g(a) 6= 0, then the quotient f /g, defined by setting (f /g)(x) =
f (x)/g(x), is continuous at a.
Proposition 6.9. If f is continuous at a and if g is continuous at f (a), then
the composition (g ◦ f )(x) = g(f (x)) is continuous at a.

Proof. We need to show that for all ε > 0 there exists δ > 0 such that if
|x − a| < δ, then |g(f (x)) − g(f (a))| < ε. Since g is continuous at f (a), we
know that for each ε > 0, there exists µ > 0 so that if |z − f (a)| < µ, then
|g(z) − g(f (a))| < ε. [Here z is used as a variable so that there aren’t too many
x’s running around.]

We also know that f is continuous at a, so that for each µ > 0, there exists
δ > 0 so that if |x − a| < δ, then |f (x) − f (a)| < µ. Take the µ that is output
by the definition of continuity of g at f (a) and input it into the definition of
continuity of f at a: since |f (x) − f (a)| < µ, we can apply the definition of
continuity of g at f (a) to get that if |x − a| < δ, then |f (x) − f (a)| < µ, and so
|g(f (x)) − g(f (a))| < ε as desired. 
Proposition 6.10. Suppose that f is continuous and that the sequence (an )
converges to a. Then, the sequence (f (an )) converges to f (a).
40 PERIŠIĆ

Proof. Since f is continuous at a, for every ε > 0, there exists some δ > 0 so
that if |x − a| < δ, then |f (x) − f (a)| < ε. Since (an ) converges to a, for each
µ > 0, there exists some n0 so that if n > n0 , then |an − a| < µ. So, suppose
we are given some ε > 0, and take µ = δ, where δ comes from our choice of ε
in the definition of continuity at a and where µ is the input in the definition
of (an ) converging to a. Then, for n > n0 , we have that |an − a| < µ = δ,
and hence that |f (an ) − f (a)| < ε, which is precisely the definition of (f (an ))
converges to f (a), as desired. [This proof should convince you, if you have not
already been convinced, of the power of appropriate definition.] 
Exercise 6.11. Suppose that f is continuous and that the sequence c, f (c),
f (f (c)), f (f (f (c))), . . . converges to a. Prove that f (a) = a.
Definition 6.12. A function f : S → R is uniformly continuous if for
each ε > 0, there exists δ > 0 so that for all a, x ∈ S, if |x − a| < δ, then
|f (x) − f (a)| < ε.

Note that this definition is very similar to the definition of continuity, except in
one aspect: in the definition of continuity, the value of δ depends on both ε and
on the point at which continuity is being checked, while for uniform continuity,
the value of δ depends only on ε and not on the point at which the definition
is being checked. In symbols the definitions are as follows:
(f continuous)
∀a ∈ S ∀ε > 0 ∃δa > 0 such that ∀x ∈ S, |x − a| < δa =⇒ |f (x) − f (a)| < ε
(f uniformly continuous)
∀ε > 0 ∃δ > 0 such that ∀a, x ∈ S, |x − a| < δ =⇒ |f (x) − f (a)| < ε
where in the first formula we write δa to emphasise the fact that δa can depend
on a.
To see that the two definitions are in fact different, consider the following
example.
Example 6.13. The function f : R → R given by f (x) = x2 is NOT uniformly
continuous. Note however that since f is a polynomial, it is continuous.

To see that f is not uniformly continuous, we argue by contradiction. We start


with a bit of algebra, namely |f (x) − f (a)| = |x2 − a2 | = |x − a| |x + a|. To say
that f is NOT uniformly continuous is to say that there exists ε > 0 such that
for all δ > 0 there exist x, a in R, with |x − a| < δ but with |f (x) − f (a)| ≥ ε.

We will show this for ε = 1 (in fact it works for any ε > 0). Fix δ > 0. We
need to find x, a with |x − a| < δ but with |x2 − a2 | ≥ ε = 1. To ensure that
|x − a| < δ let us choose x to be a + 2δ . We consider what happens as a → ∞.
We have |x2 − a2 | = |x − a| |x + a| = 2δ (a + 2δ + a) ≥ aδ. For a ≥ 1/δ we then
have aδ ≥ ε = 1. Thus we see that for any fixed δ, if a is large enough, that is,
for a ≥ 1/δ we can find a value of x with |x − a| < δ, but with |x2 − a2 | ≥ 1.
Exercise 6.14. Show that the function f : [u, v] → R given by f (x) = x2 is
uniformly continuous, for any fixed u, v ∈ R and u < v.
MATH2039 ANALYSIS 41

Exercise 6.15. Show that the function f : (0, ∞) → (0, ∞) given by f (x) = x1
is not uniformly continuous. (Note that it is continuous on the given domain.)

In Example 6.13, the reason why the function is not uniformly continuous is that
the domain is unbounded. Exercise 6.14 shows that over a bounded interval
[u, v] it is uniformly continuous. In Exercise 6.15 the issue is slightly different.
Restricting 1/x to a bounded domain such as (0, 1], it is still not uniformly
continuous: the problem is that the function diverges to ∞ as we approach
zero. If however we take a closed interval [u, v] for our domain, with 0 < u < v,
then the function is uniformly continuous.

The following two theorem shows that this holds in general. It is proved as
applications of the Bolzano-Weierstrass theorem.
Theorem 6.16. (Uniform continuity) Let f be a function defined on a
closed interval [a, b]. Then f is continuous on [a, b] if and only if f is uniformly
continuous on [a, b].

Proof. Certainly if f is uniformly continuous on [a, b] then it is continuous on


[a, b]: for each ε > 0, uniform continuity provides a single value of δ such that
for all x, c in the interval, |x − c| < δ =⇒ |f (x) − f (c)| < ε.
Conversely, we must show that if f is continuous on [a, b] then it is uniformly
continuous. Fix ε > 0. We know that for each c in [a, b] there exists δc > 0
such that |x − c| < δc =⇒ |f (x) − f (c)| < ε. Our aim is to find a single δ > 0
such that |x − c| < δ =⇒ |f (x) − f (c)| < ε.
Suppose there is no such δ. Then for every δ > 0 there exists x, c in the interval
with |x − c| < δ and |f (x) − f (c)| ≥ ε. Pick xn , cn with |xn − cn | < 1/n,
and |f (xn ) − f (cn )| ≥ ε. By the Bolzano-Weierstrass theorem, there eixsts a
convergent subsequence cnk of cn . Let c = limk→∞ cnk .
Since |xn − cn | < 1/n we have |xnk − cnk | < 1/nk → 0 as k → ∞. Thus xnk
also converges to c. Since f is continuous, we conclude that f (xnk ) − f (cnk )
converges to f (c)−f (c) = 0 as k → ∞. But by assumption |f (xnk )−f (cnk )| ≥ ε
for all k, so the limit is ≥ ε by inequality of sequences. This is a contradiction,
hence we deduce the existence of δ as required. 

The above result shows that continuous functions behave particularly well on
closed intervals. We now prove some further important results about continuous
functions on closed intervals. The first of these is another application of the
Bolzano-Weierstrass theorem.
Theorem 6.17. (Maximum Value Theorem) Let f be a function that is
continuous on the closed interval [a, b]. Then f achieves its maximum on [a, b];
that is, there exists some xmax in [a, b] so that f (xmax ) ≥ f (x) for all x ∈ [a, b].
42 PERIŠIĆ

Proof. First we prove that f is bounded above, that is, there exists R in R
such that f (x) ≤ R for all x. Suppose not. Then for each n ∈ N there exists
a point xn in [a, b] such that f (xn ) > n. By the Bolzano-Weierstrass theorem
there exists a subsequence xnk of xn , with xnk → L for some L ∈ [a, b]. As f is
continuous, f (xnk ) converges to f (L) as k → ∞. However f (xnk ) > nk which
diverges to infinity. This contradiction proves that f is bounded above.
Let A = {f (x) : x ∈ [a, b]}. We have shown that this is bounded above, hence
it has a supremum s. As s is the least upper bound, given any ε > 0, we
have that s − ε is not an upper bound of A. In particular, for each n, there
exists an ∈ A with an > s − 1/n. As an ∈ A, it follows that an = f (yn ) for
some yn in [a, b]. Applying the Bolzano-Weierstrass theorem again, there is a
subsequence ynk of yn such that with ynk → xmax for some limit xmax . Now
f (ynk ) = ank > s − 1/nk , while on the other hand f (ynk ) ≤ s as s is an upper
bound. Thus by the squeeze rule f (ynk ) → s as k → ∞. As f is continuous we
deduce that f (xmax ) = s ≥ f (x) for all x in [a, b]. 
Exercise 6.18. Prove, if f is continuous and if limx→∞ (f (x + 1) − f (x)) = 0,
then limx→∞ f (x)/x = 0.
Exercise 6.19. The Minimum Value Theorem states that, if f is contin-
uous on [a, b], then f achieves its minimum on [a, b]; that is, there exists some
y0 in [a, b] so that f (y0 ) ≤ f (x) for all x ∈ [a, b]. Use the Maximum Value
Theorem to prove the Minimum Value Theorem.

Theorem 6.20. (Intermediate Value Theorem) Let f be a function that


is continuous on the closed interval [a, b], and let y be a number lying between
f (a) and f (b). Then, there exists some x0 in the open interval (a, b) so that
f (x0 ) = y. [Pictorially, this theorem says that any horizontal line whose height
lies between f (a) and f (b) must intersect the graph of f over the interval [a, b].]

Proof. We will suppose that f (a) < y < f (b), the case f (a) > y > f (b) being
similar. Define
A = {x ∈ [a, b] : f (x) ≤ y}.
Note that a ∈ A, so in particular A is non-empty, and A ⊂ [a, b] so that A is
bounded. Hence A has a supremum s in [a, b]. As in the proof of the Maximum
Value Theorem, there exists an in A with an > s − 1/n, and since an in A we
have an ≤ s. Thus by the squeeze rule, an → s as n → ∞, so as f is continuous,
f (an ) → f (s). We deduce that f (s) ≤ y, since f (an ) < y for all n.
As y < f (b), and f (s) ≤ y we must have s 6= b, so s < b. It follows that
s+1/n ∈ [a, b] for n sufficiently large. As s+1/n > s we know that s+1/n ∈/ A,
so f (s + 1/n) > y. By continuity of f and inequality of sequences, it follows
that f (s) = lim f (s + 1/n) ≥ y. Since f (s) ≤ y we conclude that in fact
n→∞
f (s) = y. 
Example 6.21. For the function f (x) which is continuous on the closed interval
[a, b] and satisfies a < f (x) < b for all x ∈ [a, b], use the Intermediate value
MATH2039 ANALYSIS 43

property for continuous functions to determine whether there is a solution to


the equation f (x) = x on the interval [a, b].

Consider the associated function g(x) = f (x) − x. Since f is continuous on


[a, b], we see that g is continuous on [a, b], being the difference of two continuous
functions. Since a < f (a) and since f (b) < b (both of these inequalities follow
from the given fact that a < f (x) < b for all x in [a, b]), we have that g(a) =
f (a) − a > 0 and g(b) = f (b) − b < 0. Applying the Intermediate value property
to g, we see that there exists c in (a, b) so that g(c) = 0, and hence so that
f (c) − c = 0. That is, f (c) = c, and so the equation f (x) = x has a solution in
[a, b], as desired.
Exercise 6.22. For each of the following functions described below, use the In-
termediate value property for continuous functions to determine whether there
is a solution to the given equation in the specified set.
(1) f (x) = x, where f (x) is continuous on the closed interval [a, b] and
satisfies f (a) < a < b < f (b) for all x ∈ [a, b];
(2) g(x) = 0, where g(x) = x2 − cos(x);
(3) f (x) = 0 on the interval [−a, a], where a is an arbitrary positive real
number and f (x) = x1995 + 7654x123 + x;
(4) tan(x) = e−x for x in [−1, 1];
(5) x3 + 2x5 + (1 + x2 )−2 = 0 for x in [−1, 1];
(6) 3 sin2 (x) = 2 cos3 (x) for x > 0;
(7) 3 + x5 − 1001x2 = 0 for x > 0;
Example 6.23. (Solving equations by the method of bisection) The
intermediate value property allows us to determine whether an equation f (x) =
0 has a solution on a closed interval [a, b], but we may also iterate this process to
find the location of a solution to an arbitrary degree of accuracy. Let’s illustrate
this by taking a specific example; the method works the same for all continuous
functions.

Consider g(x) = x2 − cos(x) from the previous exercise. We determined that


there exists a solution c1 to g(x) = 0 in the interval [0, 2], since g(0) = −1 < 0
and g(2) = 4.4161... > 0. To isolate this solution, let’s break the interval in
half, and see which half contains the solution: the value of g at 1 is g(1) =
(1)2 − cos(1) = 0.4597... > 0. Since g(0) < 0 and g(1) > 0, the intermediate
value property yields the existence of a solution to g(x) = 0 in (0, 1). [Since
g(1) > 0 and g(2) > 0, the intermediate value property yields no information
about the possible existence of solutions in [1, 2]. To answer that question, we
would need to do something else.]

Now break [0, 1] in half: the value of g at 0.5 is g(0.5) = (0.5)2 − cos(0.5) =
−0.6276..., and so there is a solution to g(x) = 0 in [0.5, 1].
44 PERIŠIĆ

Now, break [0.5, 1] in half: the value of g at 0.75 is g(0.75) = (0.75)2 −


cos(0.75) = −0.1692... < 0, and so there is a solution to g(x) = 0 in [0.75, 1].

Now, break [0.75, 1] in half: the value of g at 0.875 is g(0.875) = (0.875)2 −


cos(0.875) = 0.1246 > 0, and so there is a solution to g(x) = 0 in [0.75, 0.875].

We could continue, but life’s too short. However this is an easy method to
teach a computer, since it involves evaluating a function, comparing numbers,
and diving by 2. It is also possible to make this method a bit more intelligent:
there is no reason to divide the intervals in the middle. For instance, in the
last step done above, it would make sense to break the interval [0.75, 1] closer
to 0.75 than to 1, since the value of g at 0.75 is closer to 0 than the value of g
at 1.
MATH2039 ANALYSIS 45

7. Differentiability

Definition 7.1. For S ⊂ R, a function f : S → R is differentiable at a


point a in S if the limit
f (a + h) − f (a) f (w) − f (a)
lim = lim
h→0 h w→a w−a
df
exists. The limit is called the derivative of f at a, written f 0 (a) or (a). We
dx
say that f is differentiable if it is differentiable at every point of its domain.
Proposition 7.2. Suppose that f is differentiable at a. Then f is continuous
at a.

Proof. Recall that f is continuous at a if limx→a f (x) = f (a), or equivalently,


if limx→a (f (x) − f (a)) = 0. So,
f (x) − f (a) f (x) − f (a)
lim (f (x) − f (a)) = lim (x − a) = lim lim (x − a) = 0
x→a x→a x−a x→a x−a x→a

f (x)−f (a)
as desired, since limx→a x−a = f 0 (a) and limx→a (x − a) = 0. 
Remark 7.3. The converse is not true: f (x) = |x| is a continuous function
but it is not differentiable at x = 0.
Theorem 7.4. (Rolle’s theorem) Suppose that the function f is continuous
on the closed interval [a, b] and differentiable on the open interval (a, b), and
that f (a) = f (b). Then, there exists a number c in the interval (a, b) so that
f 0 (c) = 0.

Proof. The proof of Rolle’s theorem is a direct consequence of the maximum


value property for continuous functions on a closed interval, the definition of the
derivative, and a bit of calculation. Since f is continuous on [a, b], it achieves
its maximum at some point c in [a, b]. Assume to start that f achieves its
maximum at a point c in the open interval (a, b). Consider the derivative of f
at c, which is defined and continuous since f is assumed to be differentiable on
(a, b): f 0 (c) = limh→0 f (c+h)−f
h
(c)
exists. Since this limit exists, the two one-
f (c+h)−f (c)
sided limits limh→0+ h and limh→0− f (c+h)−f
h
(c)
exist and are both
equal to f 0 (c). Let’s examine them individually.

For limh→0+ f (c+h)−f


h
(c)
: since f achieves its maximum at c, we have that
f (c + h) ≤ f (c) for all values of h for which c + h lies in (a, b), and so f (c +
h) − f (c) ≤ 0. Hence, limh→0+ f (c+h)−f
h
(c)
≤ 0, since the numerator is negative
or 0 and the denominator is positive.

For limh→0− f (c+h)−f


h
(c)
: again since f achieves its maximum at c, we have
that f (c + h) ≤ f (c) for all values of h for which c + h lies in (a, b), and so
f (c + h) − f (c) ≤ 0. Hence, limh→0− f (c+h)−f
h
(c)
≥ 0, since the numerator is
negative or 0 and the denominator is also negative.
46 PERIŠIĆ

Since limh→0+ f (c+h)−f


h
(c)
= limh→0− f (c+h)−f
h
(c)
and since the left hand limit
is non-positive and the right hand limit is non-negative, it must be that both
are equal to 0, and hence that f 0 (c) = 0 as well.

If f does not achieve its maximum at some point in (a, b), then it must achieve
its maximum at the endpoints, and so f (x) ≤ f (a) for all x ∈ [a, b]. We
can make the same argument at the point c in (a,b) at which f achieves its
minimum, making use of the minimum value property for f , and again argue
that if f achieves its minimum at a point c in (a, b), then f 0 (c) = 0.

The only remaining alternative is that f achieves both its maximum and its
minimum at the endpoints of [a, b], in which case it must be that f is constant
on [a, b]. In this case, we can easily calculate that f 0 (c) = 0 at every c in (a, b).
This completes the proof of Rolle’s theorem. 

Theorem 7.5. (Mean Value Theorem (MVT)) Suppose that the func-
tion f is continuous on the closed interval [a, b] and differentiable on the open
interval (a, b). Then ∃c ∈ (a, b) such that
f (b) − f (a)
f 0 (c) = .
b−a

Proof. Consider the new function


 
f (b) − f (a)
g(x) = f (x) − f (a) − (x − a),
b−a
and note that g is continuous on [a, b] and differentiable on (a, b), since it
is constructed from f and a linear polynomial, and moreover we have that
g(b) = g(a) = 0. Hence, we may apply Rolle’s theorem to g to obtain a point c
in (a, b) at which g 0 (c) = 0. Calculating, we see that
f (b) − f (a)
g 0 (c) = f 0 (c) − ,
b−a
f (b)−f (a)
and so when g 0 (c) = 0, we have that f 0 (c) = b−a , which is the conclusion
of the mean value theorem. 

Proposition 7.6. Let f : R → R be a differentiable function. If f 0 (x) > 0 for


all x, then f (x) is increasing: that is, if a < b, then f (a) < f (b).

Proof. This is a straightforward application of the mean value theorem. Take


points a and b with a < b, and apply the mean value theorem to f (x) on
the interval [a, b]. So, there is some number c in (a, b) so that f (b) − f (a) =
f 0 (c)(b − a). Since f 0 (c) > 0 by assumption and since b − a > 0, we have that
f (b) − f (a) > 0, that is, that f (b) > f (a), as desired. 

Exercise 7.7. Show that f (x) = |x − 2| on the interval [1, 4] satisfies neither
the hypotheses nor the conclusion of the Mean Value Theorem.
MATH2039 ANALYSIS 47

Example 7.8. Use the mean value theorem to prove that if f 0 (x) is constant
on R, then f (x) is a linear function; that is, there exist constants a and b so
that f (x) = ax + b.

Since f 0 (x) is constant on R, there exists some a ∈ R so that f 0 (x) = a for all
x ∈ R. Consider the function g(x) = f (x) − ax. Since both f and the linear
polynomial ax are differentiable on all of R, and hence continuous on all of R,
we have that g is differentiable on all of R, and hence is also continuous on all
of R. In order to apply the mean value theorem, we need to work on closed
intervals.

So, for x0 > 0, consider the interval [0, x0 ]. Since g is continuous on [0, x0 ] and
differentiable on (0, x0 ), the mean value theorem states that there exists some
c in (0, x0 ) so that g(x0 ) − g(0) = g 0 (c)(x0 − 0). However, g 0 (c) = f 0 (c) − a =
a − a = 0, and so g(x0 ) − g(0) = 0, and so g(x0 ) = g(0) for all x0 > 0. To
show that g(x0 ) = g(0) for all x0 < 0 as well, work with the interval [x0 , 0] and
repeat the argument just given.

So, g(x) is constant, that is, there is b ∈ R so that g(x) = b for all x ∈ R.
Substituting in g(x) = f (x) − ax, this yields that f (x) − ax = b for all x ∈ R,
or that f (x) = ax + b for all x ∈ R, where a and b are constants, as desired.
Exercise 7.9. Use the mean value theorem to prove each of the following
statements.
(1) If g 0 (x) is a polynomial of degree n − 1, then g(x) is a polynomial of
degree n;
(2) x/(x + 1) < ln(1 + x) < x for −1 < x < 0 and for x > 0;
(3) sin(x) < x for x > 0;
Example 7.10. For the function g(x) = x2 − cos(x), the same as in Exercise
6.22, use Rolle’s theorem or the mean value theorem to determine whether the
solutions described in Exercise 6.22 to the equation g(x) = 0 are the only ones.

In Exercise 6.22, we saw that there exist at least two solutions c1 and c2 to
this equation, where 0 < c1 < 2 and −2 < c2 < 0. Suppose there were a third
solution c3 to g(x) = 0. Then, since there are three points c1 , c2 , and c3 at
which g(x) = 0, by Rolle’s theorem there would exist two points e1 and e2 at
which g 0 (x) = 0. (For instance, if c3 < c2 , then e1 would lie between c3 and c2 ,
and e2 would lie between c2 and c1 .) Note that there is already one point at
which g 0 (x) = 0, namely x = 0.

However, by the same sort of argument used in the solution to parts 3 and 4 of
Exercise 7.9, we have that g 0 (x) satisfies g 0 (x) = 2x + sin(x) 6= 0 for all x 6= 0.
(Specifically, we have that g 0 (0) = 0, and that g 00 (x) = 2 + cos(x) > 0 for all
x ∈ R, since −1 ≤ cos(x) ≤ 1 for all x ∈ R. Hence, g 0 (x) < 0 for all x < 0
and g 0 (x) > 0 for all x > 0.) Hence, by Rolle’s theorem, there are only the two
solutions to g(x) = 0 that we had already found.
48 PERIŠIĆ

Exercise 7.11. For each of the following functions, the same as in Exercise
6.22, use Rolle’s theorem or the mean value theorem to determine whether the
solutions described in Exercise 6.22 are the only ones.
(1) f (x) = 0 on the interval [−a, a], where a is an arbitrary positive real
number and f (x) = x1995 + 7654x123 + x;
(2) tan(x) = e−x for x in [−1, 1];
(3) 3 sin2 (x) = 2 cos3 (x) for x > 0;
(4) 3 + x5 − 1001x2 = 0 for x > 0;
Exercise 7.12. For each of the following functions described below, determine
whether there is a solution to the given equation in the specified set.
(1) g 0 (a) = 0 = g 0 (b), where a < b are real numbers and g(x) = x3 −
12πx2 + 44π 2 x − 48π 3 + cos(x) − 1;
(2) f 0 (a) = 0, where f (x) = x4 − π 3 x − sin(x) and a ∈ R;
(3) g 0 (x) = 0 for at least k − 1 distinct real numbers, where g(x) is a
differentiable function on R which vanishes at at least k distinct real
numbers.
(4) x3 + px + q = 0 has exactly one real root for p > 0;
MATH2039 ANALYSIS 49

8. The Cauchy mean value theorem and l’Hopital’s rule

Theorem 8.1. (Cauchy Mean Value Theorem) Let f and g be two func-
tions that are both continuous on [a, b] and differentiable on (a, b). Suppose
further that g 0 (x) is never zero on (a, b). Then, there exists some c in (a, b) so
that
f (b) − f (a) f 0 (c)
= 0 .
g(b) − g(a) g (c)

Proof. Consider the function


 
f (b) − f (a)
ϕ(x) = f (x) − f (a) − (g(x) − g(a)).
g(b) − g(a)
Since both f and g are continuous on [a, b] and differentiable on (a, b), the
new function ϕ(x) is as well, as it is a linear combination of f and g. Since
ϕ(a) = ϕ(b) = 0, applying the Rolle’s theorem to ϕ, there exists a point c in
(a, b) so that ϕ0 (c) = 0. That is,
 
f (b) − f (a)
ϕ0 (c) = f 0 (c) − g 0 (c) = 0.
g(b) − g(a)
 
Hence, f 0 (c) = fg(b)−g(a)
(b)−f (a)
g 0 (c). Since g 0 (c) 6= 0 no matter the value of c, this
is equivalent to
f 0 (c) f (b) − f (a)
0
= ,
g (c) g(b) − g(a)
which is the desired conclusion. 

The Cauchy mean value theorem can be thought of as a generalisation of the


mean value theorem. It reduces to the mean value theorem if g(x) = x.

The main use of the Cauchy mean value theorem for us is to give a proof of
l’Hopital’s rule.
Theorem 8.2. (l’Hopital’s rule) Suppose that f and g are differentiable on
the union I = (a − ε, a) ∪ (a, a + ε) for some ε > 0, and that g 0 (x) is non-zero
on I. Suppose also that
lim f (x) = lim g(x) = 0.
x→a x→a

Then
f (x) f 0 (x)
lim = lim 0 ,
x→a g(x) x→a g (x)

provided that the right hand side limit either exists or is ±∞.

Proof. Since limx→a f (x) = 0, we set f (a) = 0 in order to insure that f is a


continuous function on (a − ε, a + ε), and similarly we set g(a) = 0. Fix a value
of x in I, and apply the Cauchy mean value theorem to f and g on the interval
50 PERIŠIĆ

[a, x] (if x > a, or on the interval [x, a] if x < a). Hence, regardless of which
case we’re in, there exists some number z between a and x so that
f (x) f (x) − f (a) f 0 (z)
= = 0 .
g(x) g(x) − g(a) g (z)
Since z lies between a and x, it must be that z → a as x → a, and so
f 0 (z) f 0 (x)
lim 0
= lim 0 .
z→a g (z) x→a g (x)

Since
f (x) f 0 (z)
= 0 ,
g(x) g (z)
we have
f (x) f 0 (x)
lim = lim 0 ,
x→a g(x) x→a g (x)

as required. 
Definition 8.3. l’Hopital’s rule involves limits of the form
f (x)
lim , where lim f (x) = lim g(x) = 0.
x→a g(x) x→a x→a
0
We refer to such a limit as having indeterminate form 0.

Similarly, we can define what it means for a limit limx→a fg(x)


(x)
to have inde-

terminate form ∞ , namely that limx→a f (x) = limx→a g(x) = ∞. We can

convert a limit of indeterminate form ∞ into one of indeterminate form 00 very
easily, since if
f (x)
lim
x→a g(x)

has indeterminate form ∞ ∞ , then


1/g(x)
lim
x→a 1/f (x)
has indeterminate form 00 , and
1/g(x) 1/g(x) f (x) f (x)
lim = lim · = lim .
x→a 1/f (x) x→a 1/f (x) f (x) x→a g(x)

The indeterminate form ∞ can also be handled by applying the l’Hopital rule
directly. The following theorem holds.
Theorem 8.4. l’Hopital’s rule for the case ∞ ∞ : Suppose that f and g are
differentiable on the union I = (a − ε, a) ∪ (a, a + ε) for some ε > 0, and that
g 0 (x) is non-zero on I. Suppose also that
lim f (x) = lim g(x) = ∞, (−∞ or + ∞)
x→a x→a
Then
f (x) f 0 (x)
lim = lim 0 ,
x→a g(x) x→a g (x)

provided that the right hand side limit either exists or is ±∞.
MATH2039 ANALYSIS 51

Proof. For a proof see for example S.M.Nikolsky: A Course of Mathematical


Analysis, page 174. 

There are other indeterminate forms that can be handled by l’Hopital’s rule
and some algebraic manipulations:
• indeterminate form 0 · ∞: limx→a f (x)g(x), where limx→a f (x) = 0
and limx→a g(x) = ∞. In this case, rewrite
f (x)
lim f (x)g(x) = lim
x→a x→a 1/g(x)
to get indeterminate form 00 .

As an example of this indeterminate form, consider


 
1 1
lim ln .
n→∞ n n

• indeterminate form 1∞ : We are considering the case


lim f (x)g(x) , where lim f (x) = 1 and lim g(x) = ∞.
x→a x→a x→a
In this case, rewrite
lim f (x)g(x) = lim exp (ln(f (x))g(x)) ,
x→a x→a
so that the exponent has indeterminate form 0 · ∞, and then use the
previous reduction to evaluate the limit of the exponent.

As an example of this indeterminate form, consider


 n
1
lim 1 + .
n→∞ n

• indeterminate form ∞0 : Consider the case


lim f (x)g(x) where lim f (x) = ∞ and lim g(x) = 0.
x→a x→a x→a
In this case, rewrite
lim f (x)g(x) = lim exp (ln(f (x))g(x)) ,
x→a x→a
so that the exponent has indeterminate form ∞ · 0, and then use the
above reduction to evaluate the limit of the exponent.

As an example of this indeterminate form, consider


lim n1/n .
n→∞

We close by noting that l’Hopital’s rule also holds for limits of the form
limx→∞ fg(x)
(x)
which have one of these indeterminate forms. Remember
that it is necessary to check that the limit has an indeterminate form
before applying l’Hopital’s rule.
52 PERIŠIĆ

Example 8.5. Evaluate the limit


lim (x5 − 1)/(x2 − 1).
x→1

Not all indeterminate forms require l’Hopital’s rule. By factorising, we have


that
x5 − 1 (x − 1)(x4 + x3 + x2 + x + 1) x4 + x3 + x2 + x + 1 5
lim 2 = lim = lim = .
x→1 x − 1 x→1 (x − 1)(x + 1) x→1 x+1 2
(The second equality follows from the fact that when we are taking the limit
as x → 1, we do not care what is actually happening at 1 but only near 1, and
near 1 the value of x − 1 is not zero.)
Exercise 8.6. Evaluate the following limits:
(1) limx→2 (1 − cos(πx))/ sin2 (πx);
(2) limx→−1 (x7 + 1)/(x3 + 1);
(3) limx→3 (1 + cos(πx))/ tan2 (πx);
(4) limx→1 (1 − x + ln(x))/(1 + cos(πx));
(5) limx→∞ (ln(x))1/x ;
(6) limx→2 (x2 + x − 6)/(x2 − 4);
(7) limx→0 (x + sin(2x))/(x − sin(2x));
(8) limx→0 (ex − 1)/x2 ;
(9) limx→0 (ex + e−x − x2 − 2)/(sin2 (x) − x2 );
(10) limx→∞ ln(x)/x;
(11) limx→2 (x3 − x2 − x − 2)/(x3 − 3x2 + 3x − 2);
(12) limx→1 (x3 − x2 − x + 1)/(x3 − 2x2 + x).
MATH2039 ANALYSIS 53

9. Integration

In this section we will define the Riemann integral. We note that the integral
should have the following properties:
Z b Z b Z b
(1) (λf (x) + µg(x)) dx = λ f (x) dx + µ g(x) dx.
a a a

(2) If f, g are real valued functions and f (x) ≤ g(x) for all x ∈ [a, b] then
Z b Z b
f (x) dx ≤ g(x) dx.
a a

Z b
The first condition says that f 7→ f (x) dx is a linear map, so this is
a
an algebraic property. The second condition involves inequalities, so it is an
analytic property. The integral is a first example of functional analysis, that
is, analysis involving linear algebra.
Our aim is to define the Riemann integral of a function f over an interval [a, b],
Rb Rb
written as a f . When the function f is non-negative the integral a f gives the
area under the graph of f over the interval [a, b]. It is easy to calculate the area
of an rectangle. The definition of the Riemann integral is based on calculation
of areas of rectangles. In this section we will explore that idea.
We start by defining a partition π of an interval [a, b].
Definition 9.1. Let [a, b] ⊂ R be a finite, bounded interval. A partition π
of the interval [a, b] is a finite set of points {x0 , x1 , . . . , xn } such that
a = x0 < x1 < . . . < xn = b.
The points xi , are called partition points of π; i = 0, 1, . . . , n. The interval
[xi−1 , xi ] is called the i-th subinterval of π, for i = 1, . . . , n. Let ∆xi =
xi − xi−1 be the length of the i-th interval, i = 1, . . . , n. The norm of the
partition π, |π|, is the length of its longest subinterval, formally defined as
|π| = max{∆x1 , ∆x2 , . . . , ∆xn }.
Example 9.2. For example π = {0, 81 , 14 , 12 , 1} is a partition of [0, 1] with the
partition points
1 1 1
x0 = 0, x1 = , x2 = , x3 = , x4 = 1.
8 4 2
This partition consists of five points which divide the interval [0, 1] into four
subintervals. The length of subintervals are
1 1 1 1 1
∆x1 = x1 − x0 = − 0 = , ∆x2 = x2 − x1 = − = ,
8 8 4 8 8
1 1 1 1 1
∆x3 = x3 − x2 = − = , ∆x4 = x4 − x3 = 1 − = .
2 4 4 2 2
The norm of the partition π is the length of the longest subinterval, that is
1 1 1 1
|π| = max{ , , } = .
8 4 2 2
54 PERIŠIĆ

By adding points into a partition we form finer partitions. We define the notion
of a refinement of a partition.
Definition 9.3. Given a partition π of an interval [a, b], a partition π 0 is called
a refinement of π provided every partition point of π is a partition point of
π 0 , π ⊆ π 0 . If π 0 \ π contains k points, we say that π 0 is a k-point refinement
of π. Note that π is a 0-point refinement of itself.
Example 9.4. Take the partition π = {0, 18 , 14 , 12 , 1} of [0, 1] studied in Example
9.2 and define a new partition π 0 by adding points,
3 5 6 7
π 0 = π ∪ { , , , , }.
8 8 8 8
The partition π 0 : 0 < 1
8 < 1
4 <···< 7
8 < 1 is a refinement of π. The partition
points of π 0 are
i
x0i = ; i = 0, 1, 2, . . . , 8
8
and the norm |π 0 | = 18 .

Example 9.5. (i) Let π = {x0 , x1 , . . . xn } be a partition of [a, b], c ∈ [a, b]


and c 6= xi , i = 0, 1, ..., n. Then the partition π 0 = π ∪ {c} is an 1- point
refinement of π.
(ii) Let π1 and π2 be two partitions of [a, b] and let π = π1 ∪ π2 . Then π
is a partition of [a, b] consisting of all points in π1 and all points in π2 .
The partition π is a common refinement for both partitions, π1 and π2 .

Proposition 9.6. Let π and π 0 be two partitions of [a, b] and π 0 ⊇ π. Then


|π 0 | ≤ |π|.

Proof. It is enough to consider one point refinement. Let π : a = x0 < x1 . . . <


xn = b, be a partition and c ∈ [a, b]. Consider the refinement π 0 = π ∪ {c}. If
c = xi for some i = 0, 1, 2, . . . , n then π = π 0 and |π 0 | = |π|.
If c 6= xi for all i = 0, 1, . . . , n then there exists i0 ∈ {0, 1, . . . , n} such that
xi0 < c < xi0 +1 and
∆xi0 +1 = xi0 +1 − xi0 = xi0 +1 − c + c − xi0 = ∆x0i0 +1 + ∆c.
If |π| = ∆xj , j 6= i0 + 1, then again |π 0 | = |π|.
If |π| = ∆xi0 +1 then
∆xj ≤ ∆xi0 +1 , ∀j 6= i0 + 1.
Since ∆x0i0 +1 ≤ ∆xi0 +1 and ∆c ≤ ∆xi0 +1 , it follows |π 0 | ≤ |π|. 

Definition 9.7. Let f : [a, b] → R be a bounded function and π : a = x0 <


x1 < . . . < xn = b be a partition of the interval [a, b]. For each i = 1, 2, . . . , n
let
Mi = sup{f (x)|xi−1 ≤ x < xi }, and
mi = inf{f (x)|xi−1 ≤ x < xi }.
MATH2039 ANALYSIS 55

(Note that these numbers exist since f is bounded.) The upper Darboux
sum associate with the function f and the partition π is defined by
n
X
U (f, π) = Mi ∆xi ,
i=1
and the lower Darboux sum associate with the function f and the partition
π is defined by
n
X
L(f, π) = mi ∆xi .
i=1

We will calculate the lower and upper Darboux sums in a simple example.
Example 9.8. Let f (x) = x, [a, b] = [0, 1], n ∈ N. Consider the partition π
with uniform subintervals
1 2 n
π : 0 < < < ... < = 1
n n n
Hence, the partition points are xi = ni , i = 0, 1, . . . , n. For i = 0, 1, . . . , n, we
calculate
i
Mi = sup{f (x)|xi−1 ≤ x < xi } = xi = ,
n
i−1
mi = inf{f (x)|xi−1 ≤ x < xi } = xi−1 = ,
n
and the length of the subintervals
i i−1 1
∆xi = xi − xi−1 = − = .
n n n
Now we can calculate the Darboux sums
n n n
X X i 1 1 X 1 n(n + 1) 1 1
U (f, π) = Mi ∆xi = · = 2 i= 2 · = + ,
i=1 i=1
n n n i=1 n 2 2 2n
n n n
X X i−1 1 1 X 1 (n − 1)n 1 1
L(f, π) = mi ∆xi = · = 2 (i − 1) = 2 · = − .
i=1 i=1
n n n i=1 n 2 2 2n

Proposition 9.9. Let f : [a, b] → R be a function bounded above by M and


below by m, and let π : a = x0 < x1 < . . . < xn = b be a partition of [a, b].
Then
m(b − a) ≤ L(f, π) ≤ U (f, π) ≤ M (b − a).

Proof. Left as an exercise. 

Upper and lower Darboux sums have a useful monotonicity property stated in
the following lemma.
Lemma 9.10. (Refinement lemma) Let f : [a, b] → R be a bounded function.
Let π be a partition of [a, b] and let π 0 be a refinement of π. Then
L(f, π) ≤ L(f, π 0 ) ≤ U (f, π 0 ) ≤ U (f, π).
56 PERIŠIĆ

Proof. By the definition of the upper and lower Darboux sums L(f, π 0 ) ≤
U (f, π 0 ) . To prove
U (f, π 0 ) ≤ U (f, π)
it suffices to prove the inequalities for the case when π 0 is 1-point refinement
of π. We consider the refinement π 0 = π ∪ {c} where xk−1 < c < xk for some
k ∈ {0, 1, 2, . . . , n}.
Because the most intervals are unchanged, the sum U (f, π 0 ) is obtained from
U (f, π) by replacing only the term Mk ∆xk with
(1) (2)
Mk (c − xk−1 ) + Mk (xk − c)
where
(1)
Mk = sup{f (x)|xk−1 ≤ x < c}, and
(2)
Mk = sup{f (x)|c ≤ x < xk }.
(1) (2)
Since Mk , Mk ≤ Mk , it follows
(1) (2)
Mk (c−xk−1 )+Mk (xk −c) ≤ Mk (c−xk−1 )+Mk (xk −c) = Mk (xk −xk−1 ) = Mk ∆xk
and hence U (f, π 0 ) ≤ U (f, π).
Similarly we prove L(f, π) ≤ L(f, π 0 ). This completes the proof of the lemma.


The Refinement lemma states that when we add more points into a partition,
the upper sums decrease and the lower sums increase. Our next goal is to
compare L(f, π) and U (f, π 0 ) for any two partitions π and π 0 of [a, b]. We know
that for two partitions π and π 0 of [a, b], π ∪ π 0 is again a partition of [a, b]
and it is the smallest partition that is a common refinement for both partitions
π and π 0 . With this in mind, we are equipped to prove so called Comparison
lemma.
Lemma 9.11. (Comparison lemma) Let f : [a, b] → R be a bounded function
and let π, π 0 be two partitions of [a, b]. Then
L(f, π 0 ) ≤ U (f, π).

Proof. Let π, π 0 be any two partitions of [a, b]. Since the partition π 0 ∪ π is the
refinement for both partitions, π and π 0 , by the Refinement Lemma we have
L(f, π 0 ) ≤ L(f, π 0 ∪ π) ≤ U (f, π 0 ∪ π) ≤ U (f, π).


Remark Note that


sup L(f, π) and inf U (f, π)
π π
where the infimum and supremum are taken over all partitions π of [a, b], are
well defined and
sup L(f, π) ≤ inf U (f, π).
π π
The number supπ L(f, π) is called lower Darboux integral and inf π U (f, π)
is called upper Darboux integral.
MATH2039 ANALYSIS 57

We use the following notation:


I = inf U (f, π) and I = sup L(f, π).
π π

Definition 9.12. Let f : [a, b] → R be a bounded function. We say f is


Riemann integrable over [a, b] iff
I = sup L(f, π) = inf U (f, π) = I.
π π

The number
I = sup L(f, π) = inf U (f, π)
π π

is called the Riemann integral of f over [a, b] and it is denoted by


Z b
I= f (x)dx.
a

An important characterization of Riemann integrability is given in the next


theorem.
Theorem 9.13. (Riemann’s integrability criterion) A bounded function
f : [a, b] → R is Riemann integrable over [a, b] iff ∀ > 0 ∃ a partition π of [a, b]
such that
U (f, π) − L(f, π) < .

Proof. Suppose that f is Riemann integrable (that is I = I = I). For a given


 > 0, by the definitions of I and I, there exist partitions π1 and π2 of the
interval [a, b] such that
 
U (f, π1 ) − I < and I − L(f, π2 ) < .
2 2
Let π = π1 ∪ π2 . Now
 
U (f, π)−L(f, π) = U (f, π)−I+I−L(f, π) ≤ U (f, π1 )−I+I−L(f, π2 ) < + = 
2 2
Conversely, suppose that for a give  > 0 we can find a partition π such that
U (f, π) − L(f, π) < . Now, observe that
L(f, π) ≤ I ≤ I ≤ U (f, π).
Hence
0 ≤ I − I ≤ U (f, π) − L(f, π) < .
Since  > 0 was arbitrary, it follows 0 ≤ I − I < , ∀ > 0. Hence, we have
shown I − I = 0, the upper and lower Darboux integrals are equal I = I, i.e. f
is Riemann integrable over [a, b]. 

Example 9.14. We will use the Riemann’s integrability criterion to show that
the function f (x) = x is a Riemann integrable function over [0, 1] and find the
integral.
58 PERIŠIĆ

Let (πn )n∈N be a sequence of partitions of the interval [0, 1] defined by


1 n
πn : 0 < < . . . < = 1.
n n
From the previous calculations we know that the lower and upper Darboux
sums are respectively
1 1 1 1
L(f, πn ) = − and U (f, πn ) = + .
2 2n 2 2n
By the definition of the lower Darboux integral I ≥ L(f, πn ) for all n ∈ N and
limn→∞ L(f, πn ) = 21 . Hence I ≥ 12 .
Similarly, from the definition of the upper Darboux integral I ≤ U (f, πn ) for
all n ∈ N and limn→∞ U (f, πn ) = 21 . Hence I ≤ 12 . Since I ≤ I we have I = I = 12 .

In conclusion: The function f (x) = x is Riemann integrable over [0, 1] and


R1
0
xdx = 12 .

Reflecting what we have done in this example: for a sequence of partitions


(πn ) we calculated a sequence of lower Darboux sums and a sequence of upper
Darboux sums and showed that both sequences converge to the same limit.
This can be generalize to give a characterization of Riemann integrability using
sequences of partitions.

Theorem 9.15. Let f : [a, b] → R be a bounded function. Then f is Riemann


integrable over [a, b] iff there exists a sequence (πn )n∈N of partitions of [a, b]
such that limn→∞ L(f, πn ) = limn→∞ U (f, πn ). In this case
Z b
f (x)dx = lim L(f, πn ) = lim U (f, πn ).
a n→∞ n→∞

Proof. Left as an exercise. 

Example 9.16. We will show that the indicator function of the rational num-
bers χQ
(
1 x∈Q
χQ (x) =
0 x∈ /Q
is not Riemann integrable over [0, 1].

Let π : x0 = 0 < x1 < . . . < xn = 1 be a partition of [0, 1]. For each subinterval
[xk , xk+1] we have
mk = inf{χQ (x)|xk−1 ≤ x < xk } = 0 and Mk = sup{χQ (x)|xk−1 ≤ x < xk } = 1
and therefore the Darboux sums are
n
X n
X
U (χQ , π) = 1 · ∆xk = 1 and L(χQ , π) = 0 · ∆xk = 0
k=1 k=1
MATH2039 ANALYSIS 59

Hence I = 0 and I = 1
So far we have seen an example of continuous function which is Riemann inte-
grable and we have seen that the function which has discontinuity at each ratio-
nal number is not integrable over [0, 1] or any other finite interval [a, b], a < b.
A function does not need to be continuous in order to be integrable. This is
illustrated on the next example.
Example 9.17. The function f : [0, 2] → R defined by
(
1 0≤x<1
f (x) =
2 1 ≤ x <≤ 2
is a Riemann integrable function over [0, 2].
To show that the function f is Riemann integrable it is enough to find a partition
π such that U (f, π) = L(f, π) since
L(f, π) ≤ sup L(f, π) ≤ inf U (f, π) ≤ U (f, π).
π π

Consider the partition π : 0 < 1 < 2, that is the partition of the interval [0, 2]
with the partition points x0 = 0 < x1 = 1 < x2 = 2. Then m1 = M1 = 1 and
m2 = M2 = 2 giving
L(f, π) = U (f, π) = M1 · 1 + M2 · 1 = 1 + 2 = 3.

Example 9.18. Using √ the Riemann integrability criterion, we will show that
the function f (x) = x is Riemann integrable over [0, 1].

1
Proof. Let  > 0. By the Archimedean property ∃n0 ∈ N such that 0 < n0 < .
Now let π be the partition with the partition points
 2
i
xi = , i = 0, 1, . . . , n0 ,
n0
that is, the partition
 2  2  2
1 2 n0
π : 0 = x0 < < < ... < = 1.
n0 n0 n0

Observe that f (x) = x is an increasing function. Hence for i = 1, 2, . . . , n0
we have s 
2
i i
Mi = f (xi ) = = ,
n0 n0
s 2
i−1 i−1
mi = f (xi−1 ) = = .
n0 n0
Now,
n0 n0
X 1 X 1 1 1
U (f π)−L(f, π) = (Mi −mi )∆xi = ∆xi = (xn0 −x0 ) = (1−0) = <
i=1
n0 i=1 n0 n0 n0
Since  > 0 was arbitrary, we have shown that
∀  > 0 ∃ a partition π of [0, 1] such that U (f, π) − L(f, π) < .
60 PERIŠIĆ

Hence by the Riemann’s integrability criterion, f is Riemann integrable on


[a, b]. 

Theorem 9.19. Let f : [a, b] → R be a continuous function on [a, b]. Then f


is Riemann integrable over [a, b].

Proof. To prove this, for a given  > 0 we need to find a partition π of [a, b]
such that
U (f, π) − L(f, π) < .
Since f is a continuous function on [a, b], f is uniformly continuous on [a, b].
For a given  > 0, we can find δ > 0 such that

∀x, y ∈ [a, b], with |x − y| < δ =⇒ |f (x) − f (y)| < .
b−a
Now pick any partition π : a = x0 < x1 < x2 < · · · < xn = b with |π| < δ. For
∀k ∈ {1, 2, . . . , n} By the min/max value theorem
Mk = max{f (x)|x ∈ [xk−1 , xk ]}
mk = min{f (x)|x ∈ [xk−1 , xk ]}
i.e. ∃x , x ∈ [xk−1 , xk ] such that mk = f (x0 ) and Mk = f (x00 ). Now
0 00


Mk − mk = f (x00 ) − f (x0 ) < ,
b−a
and we get
n n
X  X 
U (f, π) − L(f, π) = (Mk − mk )∆xk < ∆xk = (xn − x0 ) = .
b−a b−a
k=1 k=1


Properties of the Riemann integral


Theorem 9.20. Suppose that f, g : [a, b] → R are Riemann integrable func-
tions. Then
Z b Z b Z b
(1) λf (x) + µg(x) dx = λ f (x) dx + µ g(x) dx;
a a a
Z b Z b
(2) if f (x) ≤ g(x) for all x ∈ [a, b] then f (x) dx ≤ g(x) dx.
a a

Proof. Consult references provided. 


Z
b Rb
Exercise 9.21. Show that f (x) dx ≤ a |f (x)| dx. Hence show that if

Z a


b
|f (x)| ≤ M for all x, then f (x) dx ≤ M (b − a).

a
Z b Z b
Hint: consider f (x) dx and −f (x) dx.
a a
MATH2039 ANALYSIS 61

Exercise 9.22. Suppose f is integrable on [a, b]. For [u, v] ⊆ [a, b], show that
the restriction of f to [u, v] is integrable.

We conclude this section with the Fundamental Theorem of Calculus.


Theorem 9.23. Fundamental theorem of calculus
(1) Let f be a continuous function on the closed interval [a, b], and define
Z x
F (x) = f (t) dt
a
for x ∈ [a, b]. Then F is differentiable and F 0 (x) = f (x) for every
x ∈ (a, b). In shorthand,
Z x
d
f (t) dt = f (x).
dx a

(2) If F is a differentiable function on (a, b), and f (x) = F 0 (x) is continu-


ous, then
Z b
f (x) dx = F (b) − F (a).
a
In shorthand, Z
F 0 (x) dx = F (x).

Proof. Consult references provided. 

The Riemann integral is defined for bounded functions on closed bounded in-
tervals. In order to integrate a function that is not bounded, or is defined on
an unbounded interval, we use limits. These are called improper integrals.
There are many kinds of improper integrals. We do not give a precise definition
that covers all cases.
Definition 9.24. An improper integral is an integral in which one (or more)
of the following occurs:
• the interval of integration is not a closed interval, but instead is one of
(−∞, a], [a, ∞), or (−∞, ∞), or
• the integrand has an infinite discontinuity at some point c ∈ [a, b],
namely limx→c f (x) = ±∞.

We define convergence and divergence for improper integrals essentially


by approximating what happens on closed intervals that fill out the interval of
integration.
R∞
An improper integral of the form a f (x)dx (where f is continuous on [a, ∞))
RM
converges if the limit limM →∞ a f (x)dx exists (as a finite number). If the
R∞
improper integral a f (x)dx converges, we set
Z ∞ Z M
f (x)dx = lim f (x)dx.
a M →∞ a
62 PERIŠIĆ

Similarly for improper integrals of this form where f is continuous on (−∞, a].
R∞
An improper integral of the form −∞ f (x)dx (where f is continuous on (−∞, ∞))
convergesR ∞when, for someR c(and hence any) c in (−∞, ∞), the two improper
integrals c f (x)dx and −∞ f (x)dx both converge. This is different from
RM
assuming that limM →∞ −M f (x)dx exists, as we will see later.
Rb
An improper integral of the form a
f (x)dx (where f is continuous on (a, b]
Rb
and limx→a+ f (x) = ±∞) converges if the limit limc→a+ c f (x)dx exists (as
Rb
a finite number). If the improper integral a f (x)dx exists, we set
Z b Z b
f (x)dx = lim f (x)dx.
a c→a+ c
Similarly for improper integrals of this form where f is continuous on [a, b) and
limx→b− f (x) = ±∞.
Rb
An improper integral of the form a
f (x)dx (where f is continuous on [a, b] ex-
Rc Rb
cept at a single point c in (a, b), and both the integrals a f (x)dx and c f (x)dx
Rc
are of the previous form) converges if both the improper integrals a f (x)dx
Rb
and c f (x)dx converge.

An improper integral that does not converge is said to diverge.


R∞
Example 9.25. Consider the integral 4 xdx 3/2 . This is an improper integral

because the interval of integration is [4, ∞) and hence is not a closed interval.
So, we attempt to evaluate this integral by formulating it as a limit of integrals
over closed intervals, namely
Z ∞ Z M
1 1
3/2
dx = lim 3/2
dx
4 x M →∞ 4 x
Z M
= lim x−3/2 dx
M →∞ 4
 
2 2
= lim − √ + √ = 1.
M →∞ M 4
Hence, this improper integral converges to 1.
Exercise 9.26. Determine whether the following improper integrals converge
or diverge, and evaluate those which converge.

R4 dx
(1) 0 x3/2
;

R∞ dx
(2) 1 (x+1) ;

R∞ dx
(3) 5 (x−1)3/2
;
MATH2039 ANALYSIS 63

R9 dx
(4) 0 (9−x)3/2
;

R −2 dx
(5) −∞ (x+1)3
;

R8 dx
(6) −1 x1/3
;

R∞ dx
(7) 2 (x−1)1/3
;

R∞ xdx
(8) −∞ (x2 +4)
;

R1 √
e √x dx
(9) 0 x
;

R∞ dx
(10) 1 x ln(x) .

Exercise 9.27. Show that


Z ∞
(1 + x)
dx
−∞ (1 + x2 )
diverges, but that Z t
1+x
lim dx = π.
t→∞ −t 1 + x2
64 PERIŠIĆ

10. Series

Definition
P∞10.1. A series (or infinite series) is a mathematical construct of
the form n=0 an . A series is essentially the sum of the elements in a sequence.
P∞ P∞
Consider a series n=0 an . We define what it means for n=0 an to converge
or diverge by reducing to what we have
P∞done before, namely sequences. Define
the k-th partial sum of the series n=0 an as
k
X
Sk = an
n=0

and consider the sequence (Sk )Pk=0 . The sequence (Sk ) is called the sequence

of partial sums of the series n=0 an .
P∞
Definition 10.2. The series P∞an converges if the sequence of partial
n=0
sums (Sk ) converges. The series n=0 an diverges if the sequence of Ppartial

sums (Sk ) diverges. If (Sk ) converges to S, we say that the series n=0 an
converges to S and we write

X
an = S = lim Sk .
k→∞
n=0
P∞
Example 10.3. Consider the series n=0 an , where an = 1 for all n ≥ 0. This
series diverges. To see this, consider the sequence of partial sums (Sk ) : the
k th partial sum Sk is
k
X k
X
Sk = an = 1 = k + 1,
n=0 n=0

and the sequence (Sk ), Sk = k + 1 diverges.


P∞ P∞
Fact 10.4. Let n=0 an and n=0 bn be two infinite series, and suppose there
exists P ∈ N so that an = bn for all n > P . (That
P∞is, assume the terms of the
two
P∞ series are equal after some point.) Then, n=0 an converges if and only
if n=0 bn converges. That is, the convergence or divergence of a series is not
affected by mucking about with finitely many terms of the series.

This is a powerful and highly useful fact that we’ll make repeated use of, often
without being explicit about it, so keep a sharp eye out.

P∞ n 10.5. Fix a real number r 6= 0 and consider the geometric series


Example
n=0 r . We can determine for which values of r this series converges, directly
from the definition. Namely, consider the k th partial sum Sk :
k
X 1 − rk+1
Sk = rn = 1 + r + r2 + · · · + rk = .
n=0
1−r

We now need to evaluate limk→∞ Sk . However, as k → ∞, we know how rk+1


behaves:
• if |r| < 1, then limk→∞ rk+1 = 0;
MATH2039 ANALYSIS 65

• if r = 1, then Sk = k + 1 (that is, the formula above breaks down in


this case), and so (Sk ) diverges;
• if r > 1, then limk→∞ rk+1 = ∞;
• if r ≤ −1, then limk→∞ rk+1 diverges.
Hence, limk→∞ rk+1 exists if and only if |r| < 1, and in this case
lim rk+1 = 0.
k→∞
Hence, the sequence of partial sums (Sk ) converges if and only if |r| < 1 and

X 1
rn = lim Sk = .
n=0
k→∞ 1 − r

Exercise 10.6. (1) A ball has bounce coefficient 0 < r < 1 if, when it
is dropped from height h, it bounces back to a height of rh. Suppose
that such a ball is dropped from the initial height a and subsequently
bounces infinitely many times. Determine the total up-and-down dis-
tance the ball travels.
(2) Two cars, driven by Jack and Jill, are begin driven towards each other,
with Jack driving at 25 miles per hour and Jill driving at 95 miles per
hour. When the cars are 120 miles apart, a fly leaves the front of Jack’s
car and flies to Jill’s car at 257 miles per hour; when it reaches Jill’s
car, it immediately turns around and flies back to Jack’s car, and keeps
going back and forth until it is crushed between the two cars when they
crash together. Assuming the fly loses no time in changing direction,
calculate the total distance the fly has flown in its journey between the
two cars.
Example 10.7. Though we do not yet have the technical tool we need to prove
this, the other
P∞ important series to know is to pick a real number s and consider
the series n=1 n1s . This series converges if and only if s > 1.

For s = 1, this series is called the harmonic series, and we can prove directly
that it diverges. Note that 31 + 14 > 12 , that 15 + · · · + 81 > 4 81 = 12 , and in general
that
1 1 1 1 1
+ + · · · + k > 2k−1 k = .
2k−1 + 1 2k−1 + 2 2 2 2
Hence, the (2k )th partial sum S2k satisfies S2k > 1 + k 12 . Since the terms in
the harmonic series are all positive, the sequence of partial sums is monoton-
ically increasing, and by the calculation done the sequence of partial sums is
unbounded, and so the sequence of partial sums diverges. Hence, the harmonic
series diverges.
P∞ 1
Exercise 10.8. Prove that n=1 ns diverges for s < 1, by estimating its
partial sums.
66 PERIŠIĆ

Theorem
P∞ 10.9. (Arithmetic
P∞ of series) P∞
Let
P∞ n=0 a n and n=0 bn be convergent series with n=0 an = A and
n=0 bn = B. Then
P∞ P∞ P∞
(1) sum: n=0 (an + bn ) = n=0 an + n=0 bn = A + B;
P∞ P∞ P∞
(2) difference: n=0 (an − bn ) = n=0 an − n=0 bn = A − B;

(3) P
multiplicationPby a constant: For a constant c,
∞ ∞
n=0 c an = c n=0 an = cA.

Pk Pk
Proof. Let Sk = n=0 an and Vk = n=0 bn be the partial sums of the two
series. Since both series are convergent, we have that limk→∞ Sk = A and
limk→∞ Vk = B.
(1) This follows immediately from the definition of convergence
P∞ of a series
in terms of partial sums: the partial sums of the series n=0 (an + bn )
are
Xk k
X Xk
Tk = (an + bn ) = an + bn = Sk + Vk
n=0 n=0 n=0
(since the sums are finite). Since limk→∞ Sk = A and limk→∞ Vk = B,
we have that
lim Tk = lim (Sk + Vk ) = lim Sk + lim Vk = A + B.
k→∞ k→∞ k→∞ k→∞
P∞
So, not only does the series of sums n=0 (an + bn ) converge (since
its sequence of partial sums converges), but it converges to A + B, as
expected.
(2) Much as the rule for sums just done, this follows immediately from the
definition of convergence
P∞ of a series in terms of partial sums: the partial
sums of the series n=0 (an − bn ) are
k
X k
X k
X
Wk = (an − bn ) = an − bn = Sk − Vk
n=0 n=0 n=0

(since the sums are finite). Since limk→∞ Sk = A and limk→∞ Vk = B,


we have that
lim Wk = lim (Sk − Vk ) = lim Sk − lim Vk = A − B,
k→∞ k→∞ k→∞ k→∞
P∞
So, not only does the series of differences n=0 (an −bn ) converge (since
its sequence of partial sums converges), but it converges to A − B, as
expected.
(3) Much as the rules for sums and differences just done, this follows imme-
diately from the definition of convergence
P∞ of a series in terms of partial
sums: the partial sums of the series n=0 c an are
k
X k
X
Zk = c an = c an = c S k
n=0 n=0
MATH2039 ANALYSIS 67

(since the sums are finite). Since limk→∞ Sk = A, we have that


lim Zk = lim c Sk = c lim Sk = cA,
k→∞ k→∞ k→∞
P∞
So, not only does the series of constant multiples n=0 c an converge
(since its sequence of partial sums converges), but it converges to cA,
as expected.

P∞ P∞
Exercise 10.10. (1) ShowPthat, if n=0 an converges and if n=0 bn di-

verges, then the series n=0 (an + bn ) diverges.
P∞ P∞
(2) Show that, if n=0 an diverges and if c 6= 0, then the series n=0 c an
diverges.
P∞
Example
P∞ 10.11. Construct an example of convergentP∞ series n=0 an and
n=0 b n with positive terms for which the series n=0 an bn diverges, or prove
that no such example exists.
P∞ P∞
No such example exists: since n=0 an and n=0 bn both converge, the se-
Pk Pk
quences of partial sums Sk = n=0 an and Vk = n=0 bn both converge. Note
that the k th partial sum Wk of the series of products satisfies
k k
! k !
X X X
Wk = a n bn ≤ an bn = Sk Vk .
n=0 n=0 n=0
P∞ P∞
Since Sk ≤ A = n=0 an and Vk ≤ B = n=0 bn for all k, we have that
Wk ≤ AB for all k. Since Wk is a monotonically increasing sequence (as an
and bn are positive for all n) and since Wk is bounded
P∞ (by AB), we have that
Wk converges, and hence that the series of products n=0 an bn converges.
Exercise 10.12. Unlike sequences, the convergence of series whose terms are
products and quotients of convergent series does not necessarily follow. Ex-
ploring this phenomenon is the purpose of this example. Construct examples
of each of the following, or prove that no such example exists:
P∞ P∞
(1) convergent series n=0 Pa∞
n and n=0 bn with positive terms for which
the series of products n=0 an bn converges;
P∞ P∞
(2) divergent series n=0P an and n=0 bn with positive terms for which

the series of products n=0 an bn diverges;
P∞ P∞
(3) divergent series n=0P an and n=0 bn with positive terms for which

the series of products n=0 an bn converges;
P∞ P∞
(4) convergent series n=0Pan and n=0 bn with positive terms for which
∞ an
the series of quotients n=0 bn diverges;
P∞ P∞
(5) convergent series n=0Pan and n=0 bn with positive terms for which

the series of quotients n=0 abnn converges;
P∞ P∞
(6) divergent series n=0 P an and n=0 bn with positive terms for which

the series of quotients n=0 abnn diverges;
68 PERIŠIĆ

P∞ P∞
(7) divergent series n=0 P an and n=0 bn with positive terms for which
∞ an
the series of quotients n=0 bn converges;
Fact 10.13. This is a useful
P∞ fact that we have already run across at least once.
If the terms of a series n=0 an are all positive, then the sequence of partial
sums is monotonically increasing, since
k+1
X k
X k
X
Sk+1 = an = an + ak+1 > an = Sk .
n=0 n=0 n=0

(If all the terms in the series are non-negative, then the sequence of partial
sums is monotonically non-decreasing.) This fact makes an appearance in the
proofs of several of the following tests for convergence or divergence of series.
Theorem 10.14. (Series convergence tests) Be careful when reading the
hypotheses, as not all these tests have the same hypotheses. In particular, some
only apply to series with non-negative terms, while others apply to all series.
• nth term test for divergence: If limn→∞ an 6= 0P (so that either an

diverges, or an converges to a 6= 0), then the series n=0 an diverges.
P∞ P∞
• Comparison test: If n=0 an andP n=0 bn are series with P
∞ ∞
0 ≤ an ≤ bn for all n ≥ 0, and if n=0 bn converges, then n=0 an
converges.
P∞ P∞
• Limit comparison test: If n=0 an and n=0 bn are series with
non-negative terms and if the limit limn→∞ abnn = L exists with 0 <
P∞ P∞
L < ∞, then n=0 an converges if and only if n=0 bn converges.
P∞
• Integral test: If n=0 an is a series of positive terms and if there
exists
R∞ a decreasing continuous function f (x) for which f (n)
P∞= an , then
0
f (x)dx converges (that is, is finite) if and only if n=0 an con-
verges. [This integral is an improper integral, as described earlier.]
P∞
• Ratio test: Let n=0 an be a series of Ppositive terms and suppose
an+1 ∞
limn→∞ an = L exists. If L < 1, then n=0 an converges. If L > 1,
P∞
then n=0 an diverges. If L = 1, this test gives no information.
P∞
• Root test: Let n=0 an be a series of positive terms and suppose
1/n
P∞
limn→∞ (an )P = L exists. If L < 1, then n=0 an converges. If

L > 1, then n=0 an diverges. If L = 1, this test gives no information.
P∞
• Alternating series test: Consider a series of the form n=0 (−1)n an ,
where an > 0 for all n ≥ 0. If an+1 ≤ an for all n ≥ 0 and
limn→∞ an = 0, then the series converges.

Proof. • nth term test for divergence:


P∞ we prove this by proving its
contrapositive: If the series n=0 an converges, them limn→∞ an = 0.
Pk P∞
Let Sk = n=0 an be the k th partial sum of the series n=0 an . By
definition, the sequence of partial sums (Sk ) converges. By the Cauchy
criterion, we then have that for any given ε > 0, there exists M so
that if p, q > M , then |Sp − Sq | < ε. In particular, taking any p > M
MATH2039 ANALYSIS 69

and q = p + 1, we see that |Sp − Sq | = |ap+1 | < ε. Hence, if we set


Q = M + 1, then |an | < ε for every n > Q, and so limn→∞ an = 0, as
desired.
Pk
• Comparison test: again, we use partial sums: let Sk = n=0 an and
Pk
Tk = n=0 bn be the partial sums of the two series. Since an ≤ bn for
all n, we have that Sk ≤ Tk for all k. Further, since both the series
have non-negative terms, we have that both P∞ sequences (Sk ) and (Tk )
are monotonically non-decreasing. Since n=0 bn converges, it must be
that (Tk ) is bounded, since a monotonic sequence converges if and only
if it is bounded. Hence, (Sk ) is also a bounded monotonic
P∞ sequence,
bounded by limk→∞ Tk since Sk ≤ Tk for all k, and so n=0 an is also
a convergent series. [Note that the proof of the comparison test relies
heavily on the fact that the series have non-negative terms, thus forcing
the sequences of partial sums to be monotonic.]
• Limit comparison test: since limn→∞ abnn = L > 0, we can apply
the definition of limit with ε = 12 L to get that there exists M so that
1 an 3
2 L < bn < 2 L for n > M . In particular, applying a bit of algebraic
massage, we have that an < 32 Lbn for all n > M and that bn < L2 an
Pk Pk
for n > M . Let Sk = n=0 an and Tk = n=0 bn be the partial sums
of the two series. As above, since both the series have non-negative
terms, we have that both sequences Sk and Tk are monotonically non-
decreasing. For the sake of precision, remove the first M terms of both
series, which does not affect the convergence or divergence of either.
This is done so that the two inequalities an < 32 Lbn and bn < L2 an hold
true for all n.
P∞
Suppose that n=0 bn converges, so that Tk is a bounded monotonic
sequence. Since an < 32 Lbn for all n > M , we have that Sk < 32 LTk ;
P∞
hence, the sequence Sk is bounded by 32 L limk→∞ Tk , and so n=0 an
converges.
P∞
Suppose now that n=0 an converges, so that Sk is a bounded mono-
tonic sequence. Since bn < L2 an for all n > M , we have that Tk < L2 Sk ;
P∞
hence, the sequence Tk is bounded by L2 limk→∞ Sk , and so n=0 bn
converges.
R∞
• Integral test: the definition of convergence for the integral 0 f (x)dx
RM
is that the limit limM →∞ 0 f (x)dx exists (and is finite). Recall also
RM
that 0 f (x)dx is the area under the graph of f (x) over the interval
R∞
[0, M ], and that 0 f (x)dx is the area under the graph of f (x) over
[0, ∞).
RM
Suppose that limM →∞ 0 f (x)dx exists. For each n satisfying 1 ≤ n ≤
M , consider the rectangle Rn over the interval [n − 1, n] with height
f (n) = an . Since f is decreasing, the rectangle Rn is contained entirely
70 PERIŠIĆ

under the graph of f , and the area of Rn is base · height = f (n) = an .


So, comparing areas, we see that
M
X XM Z M
(area of Rn ) = an ≤ f (x)dx.
n=1 n=1 0
RM R M +1
Since the sequence 0 f (x)dx is monotone increasing (each M f (x)dx
RM
is positive) and bounded (by limM →∞ 0 f (x)dx), we see that the
P∞
sequence of partial sums of n=1 aP n is also a bounded monotone se-

quence, hence convergent. That is, n=0 an converges.
P∞
Suppose now that n=0 an converges. For each n ≥ 1, let Wn be
the rectangle over the interval [n − 1, n] with height f (n − 1) = an−1 .
The part of the graph of f over [0, M ] is contained in the union of the
rectangles W0 ∪ · · · ∪ WM −1 , and so comparing areas, we see that
Z M XM M
X
f (x)dx ≤ (area of Wn ) = an−1 .
0 n=1 n=1
RM
As above, the sequence 0 f (x)dx is monotonically increasing and
P∞ RM
bounded (by n=0 an ), and so limM →∞ 0 f (x)dx exists (and is fi-
nite), as desired.
• Ratio test: [note: the proofs of the ratio and root tests are similar
to each other, but different from the proofs already given, in that they
don’t use partial sums, but instead use comparison to an appropriately
chosen geometric series.]

We are given that limn→∞ aan+1 n


= L exists. Suppose that L < 1.
Choose some µ so that L < µ < 1; applying the definition of limit with
ε = µ−L, there exists M so that aan+1
n
< µ for n ≥ M . (Note the change
from the usual n > M to n ≥ M , made here purely for notational
convenience.) So, aM +1 < µaM , and aM +2 < µaM +1 < µ2 aM , and in
general, we have that aM +k < µk aM for k ≥ 0. (We’re using here that
the an are all positive, so that among other things, the inequalities
don’t change direction
P∞ when we multiply through by an .) Since the
geometric series k=0 µk converges P∞(since µ < 1), the comparison test
yields that the truncated
P∞ series n=M an converges, and hence that
the original series n=0 an converges.

Suppose now that L > 1, and essentially repeat the argument. Choose
some η so that 1 < η < L; applying the definition of limit with ε = L−η,
there exists M so that aan+1
n
> η for n ≥ M . (Note the change from the
usual n > M to n ≥ M , made here purely for notational convenience.)
So, aM +1 > ηaM , and aM +2 > ηaM +1 > η 2 aM , and in general, we
have that aM +k > η k aM for k ≥ 0. (We’re using here that the an are
all positive, so that among other things, the inequalities don’t change
direction
P∞ k when we multiply through by an .) Since the geometric series
k=0 η diverges (since η > 1), the comparison test yields that the
MATH2039 ANALYSIS 71

P∞
truncated series n=M an diverges, and hence that the original series
P ∞
n=0 an diverges.

[The reason this proof does not work when L = 1 is that we cannot
find a number between L and 1, as we did in both of the parts of the
proof just given.]
• Root test: The proof here is very similar to the proof just given (and
fails when L = 1 for the same reason). When L < 1, again choose µ
satisfying L < µ < 1, and then apply the definition of limit to find M
so that (an )1/n < µ for n ≥ M . Then, taking the nth power of both
sides, we get that an < µn for all n ≥ M , and so again we canP∞use the
n
second comparison testP∞with the convergent geometric series n=M µ
to get convergence of n=0 an .

When L > 1, choose η satisfying 1 < η < L, and apply the definition
of limit to get M so that (an )1/n > η for n ≥ M , so that an > η n for
P≥
n

M . By the comparison
n
P∞ test with the divergent geometric series
n=M η , we get that n=0 an diverges.

• Alternating series test: start by considering the partial sums Sk for


k odd:
2p+1
X
S2p+1 = an = (a0 − a1 ) + (a2 − a3 ) + · · · + (a2p − a2p+1 ).
n=0
Since each term in parentheses a2s −a2s+1 is non-negative, since a2s+1 ≤
a2s by assumption, we have that the odd partial sums S2p+1 are all non-
negative, and are monotonically non-decreasing. Also, by grouping the
terms in S2p+1 differently, namely as
2p+1
X
S2p+1 = an = a0 − (a1 − a2 ) − (a3 − · · · − a2p ) − a2p+1 ,
n=0
and again using that the parenthetical terms are non-negative, we see
that S2p+1 ≤ a0 for all p, and so the odd partial sums form a bounded
monotone sequence. Let S = limp→∞ S2p+1 .

We need to show now that the even partial sums S2p converge to the
same limit. However, since S2p = S2p−1 +a2p and since limp→∞ a2p = 0,
we have that
lim S2p = lim S2p−1 + lim a2p = S + 0 = S,
p→∞ p→∞ p→∞

and soPthe sequence Sk of all partial sums converges to S. That is, the

series n=0 (−1)n an converges.

Remark
P∞ 10.15. It follows
P∞from the comparison testPthat if 0 ≤ an ≤ bn and

n=0 an diverges, thenP n=0 bn also diverges. (If n=0 bn did not diverge,

i.e. if it converged then n=0 an would also converge by the comparison test.)
72 PERIŠIĆ

P∞
Example 10.16. We use the integral test to show that the series n=1 n1s
from Example 10.7 converges for s > 1. Recall that we have already seen that
this series diverges for s ≤ 1.

So, consider the function f (x) = x1s = x−s , so that n1s = f (n). Since s > 1,
f 0 (x) = −s xs+1
1
< 0 for all x > 0, and so f (x) is decreasing. Further,
Z ∞ Z M
f (x)dx = lim x−s dx
1 M →∞ 1
1
x−s+1 M

= lim 1
M →∞ −s + 1
 
1 1 1
= lim −1 = .
M →∞ −s + 1 M s−1 s−1
Since the limit converges, the series converges, as desired.
P∞
Remark 10.17. It is known that for s an even positive integer, that n=1 n1s
is a rational multiple of π s . Moreover, there is an explicit formula for the sum
of this series.

For s an odd positive integer, we have already seen that this series
P∞ diverges for
s = 1 (as this is the harmonic series). Further, it is known that n=1 n13 is an
P∞
irrational number, but it is not known that n=1 n13 is a rational multiple of
P∞ 1
π. Nothing is known about n=1 ns for s an odd positive integer s ≥ 5, other
than it is a convergent series.

The general philosophy with convergence of series is that a series will converge
provided that the terms tend to zero fast enough. As a general principal, always
start by applying the nth term test for divergence (whether you write out the
details or just apply the test mentally). If the terms don’t go to zero then
you’re done: the series diverges. Otherwise the question is just how fast do the
terms tend to zero. Is the series similar to one that you already know about?
Then perhaps you can use the comparison, or limit comparison tests. There
are also some rules of thumb for applying the ratio and root tests. If there’s an
exponential or factorial then the ratio test is usually a good idea. If you have
something raised to the power n then the root test is useful.

In the series problems which follow, there are often many correct way to do
any problem. There are no hard and fast rules about which test to use, and
sometimes it may be easiest to use more than one: just keep applying different
tests until you get an answer, or a series that you already understand.
P∞
Example
P∞ 10.18. For a convergent series n=1 an with positive terms, prove
that n=1 ann converges.
Pk P∞
Let Sk = n=1 an be the k th partial sum of n=1 an . Consider the k th partial
P∞ an Pk an
sum Tk of the new series n=1 n , Tk = n=1 n , and compare TP k to Sk :

since ann ≤ an for all n ≥ 1, we have that Tk ≤ Sk for all n ≥ 1. Since n=1 an
is a convergent series with positive terms, its sequence of partial sums (Sk ) is a
MATH2039 ANALYSIS 73

monotonically increasing sequence that converges to S. In particular, Sk ≤ S


for all k ≥ 1. Since Tk ≤ Sk ≤ S, we see that (T ) is a bounded monotonically
Pk∞
increasing sequence, and hence converges. So, n=1 ann is a convergent series.
P∞
Exercise 10.19. In each of the following, n=1 an is a convergent series with
positive terms.
(1) Prove that,
P∞if (cn ) is a sequence of positive terms satisfying limn→∞ cn =
0, then n=1 an cn converges;
(2) Prove that, ifP(cn ) is a sequence of positive terms satisfying limn→∞ cn =

c 6= 0, then n=1 an cn converges.

In general, a series whose terms are positive is much easier to handle, particu-
larly in terms of determining convergence and divergence. One way to handle
a general series, that is one without the restriction that the terms are positive,
is to compare it to a series with positive terms.
P∞ P∞
Definition 10.20. Let n=0 an be a series. The series P∞ n=0 an converges
absolutely if the associated series of absolute values n=0 |an | converges.

Note that absolute convergence and convergence are the same for a series with
positive terms.

The connection between convergence and absolute convergence is given in the


following proposition.
P∞ P∞
Proposition
P∞ 10.21. Let n=0 an be a series. If n=0 an converges absolutely,
then n=0 an converges.

P∞ P∞
Proof. Let n=0 an be a series that converges Pabsolutely, so n=0 |an | con-

verges. By the arithmetic of series, the series n=0 2|an | then
P∞ also converges.
We wish to understand whether or not the original series n=0 an converges.
Note that 0 ≤ an + |an | ≤ 2|an |, and
P∞ so by the comparison test, the series
P ∞
n=0 (an +
P∞|an |) converges. Since
P∞ n=0 |an | converges by assumption, their
P ∞
difference n=0 (an + |an |) − n=0 |an | = n=0 an converges, by the arith-
metic of series, and we are done. 

In Theorem 10.14, we stated the ratio and root tests for series with positive
terms. Combining Theorem 10.14 with Proposition 10.21, we obtain the ratio
and root tests for series with non-zero terms, as tests to determine whether the
series converges absolutely or diverges.
P∞
Proposition 10.22. Ratio and root tests for general series: Let n=0 an
be a series with non-zero terms, so that an 6= 0 for all n.

• Ratio test: Suppose that limn→∞ aan+1 = L exists. If L < 1, then

n
P∞ P∞
a
n=0 n converges absolutely. If L > 1, then n=0 an diverges. If
L = 1, this test gives no information.
74 PERIŠIĆ

1/n
• Root
P∞ test: Suppose that limn→∞ (|an |) =LPexists. If L < 1, then

n=0 an converges absolutely. If L > 1, then n=0 an diverges. If
L = 1, this test gives no information.
Definition 10.23. Proposition 10.21 gives us that a series that converges ab-
solutely then necessarily converges. The converse however is not true: there
are series that converge but do not converge absolutely.

To give this possibility a name, say that a series converges conditionally if


it converges but does not converge absolutely.
Example 10.24. The alternating series test gives us a way to construct an
example of a seriesPthat converges conditionally. Consider the alternating
∞ n1 1 1
harmonic series n=1 (−1) n . Since n > n+1 for all n ≥ 1 and since
P∞
limn→∞ n1 = 0, the alternating series test yields that n=1 (−1)n n1 converges.
However, when we take P∞absoluten values of all the terms in this series, we get
1
P ∞ 1
the harmonic series n=1 |(−1) n | = n=1P n , which we have already seen

diverges. So, the alternating harmonic series n=1 (−1)n n1 converges but does
not converge absolutely. That is, it converges conditionally.
P∞
Example 10.25. Determine whether the series n=0 e−n converges absolutely,
converges conditionally, or diverges. If the series converges, determine its limit.

Converges absolutely: we apply the ratio test, as


e−(n+1) e−1 1
lim −n
= lim = < 1,
n→∞ e n→∞ 1 e
P∞ −n
and so n=0 e converges. (We make implicit use of the fact that for a series
of positive terms, convergence and absolute convergence are the same notion.)
Exercise 10.26. The series scavenger hunt: For each of the infinite series
given below, do the following:
• Determine whether the series converges absolutely, converges condition-
ally or diverges.
• If the series converges, determine its limit, where possible.
P∞ 2n−1
(1) n=0 3n ;
P∞ n
(2) n=0 (1.01) ;
P∞ e n
(3) n=1 ( 10 ) ;
P∞ 1
(4) n=1 n2 +n+1 ;
P∞ 1√
(5) n=1 n+ n ;
P∞ 1
(6) n=1 1+3n ;
P∞ 10n2
(7) n=2 n3 −1 ;
P∞
(8) √ 1
n=1 37n3 +3 ;
MATH2039 ANALYSIS 75

P∞ √
n
(9) n=1 n2 +n ;
P∞ 2
(10) n=2 ln(n) ;
P∞ sin2 (n)
(11) n=1 n2 +1 ;

n+2n
P∞
(12) n=1 n+3n ;
P∞ 1
(13) n=2 n2 ln(n) ;

n3 +1
P∞
(14) n=1 n4 +2 ;
P∞ 1
(15) n=1 n+n3/2 ;

10n2
P∞
(16) n=1 n4 +1 ;

n2 −n
P∞
(17) n=2 n4 +2 ;
P∞
(18) √ 1 ;
n=1 n2 +1
P∞ 1
(19) n=1 3+5n ;
P∞ 1
(20) n=2 n−ln(n) ;
P∞ cos2 (n)
(21) n=1 3n ;
P∞ 1
(22) n=1 2n +3n ;
P∞ 1√
(23) n=1 n(1+ n) ;
P∞
(24) n=1 1/(2n (n + 1));
P∞
(25) n=1n!/(n2 en );
P∞ √ n
(26) n=2 n/(3 ln(n));
P∞ 3
(27) n=2 (2n)!/(n!) ;
P∞ n 4
(28) n=1 (1 − (−1) )/n ;
P∞
(29) n=1 (2 + cos(n))/(n + ln(n));
P∞ p
(30) n=3 1/(n ln(n) ln(ln(n)));
P∞ n n
(31) n=1 n /(π n!);
P∞ n+1 n
(32) n=1 2 /n ;
P∞ n−1

(33) n=1 (−1) / n;
P∞
(34) n=1 cos(πn)/((n + 1) ln(n + 1));
P∞ n 2 2
(35) n=1 (−1) (n − 1)/(n + 1);
P∞ n n
(36) n=1 (−1) /(nπ );
P∞ n 2 3 2
(37) n=1 (−1) (20n − n − 1)/(n + n + 33);
76 PERIŠIĆ

P∞
(38) n=1 n!/(−100)n ;
P∞
(39) n=31/(n ln(n)(ln(ln(n)))2 );
P∞ n

(40) n=1 (1 + (−1) )/ n;
P∞ n 2 n
(41) n=1 e cos (n)/(1 + π );
P∞ 4
(42) n=2 n /n!;
P∞ n
(43) n=1 (2n)!6 /(3n)!;
P∞ 100 n

(44) n=1 n 2 / n!;
P∞
(45) n=3 (1 + n!)/(1 + n)!;
P∞ 2n 2
(46) n=1 2 (n!) /(2n)!;
P∞ n 2
(47) n=1 (−1) /(n + ln(n));
P∞ 2n n
(48) n=1 (−1) /2 ;
P∞ n
(49) n=1 (−2) /n!;
P∞ 2
(50) n=0 −n/(n + 1);
P∞
(51) n=1 100 cos(nπ)/(2n + 3);
P∞
(52) n=10 sin((n + 1/2)π)/ ln(ln(n));
P∞ 2n 2
(53) n=1 (2n)!/(2 (n!) );
P∞ n2
(54) n=1 (n/(n + 1)) ;
P∞
(55) n=1 1/(1 + 2 + · · · + n);
P∞ 3
(56) n=1 ln(n)/(2n − 1);
P∞ 2
(57) n=1 sin(n)/n ;
P∞ n
(58) n=1 (−1) (n − 1)/n;
P∞ n 3n n
(59) n=1 (−1) 2 /7 ;
P∞ 4
(60) n=1 cos(n)/n ;
P∞ n n n
(61) n=1 (−1) 3 /(n(2 + 1));
P∞ n−1
(62) n=1 (−1) n/(n2 + 1);
P∞
(63) n=2 (−1)
n−1
/(n ln2 (n));
P∞ n−1 n
(64) n=1 (−1) 2 /n2 ;
P∞ n
√ 3/2
(65) n=1 (−1) sin( n)/n ;
P∞ 4 −n 2
(66) n=1 n e ;
P∞
(67) n=1 sin(nπ/2)/n;
P∞ 8
(68) n=2 1/(ln(n)) ;
MATH2039 ANALYSIS 77

P∞
(69) n=13 1/(n ln(n)(ln(ln(n)))p ), where p > 0 is an arbitrary positive real
number.
P∞
Exercise 10.27. Let n=1 anPbe a convergent series of positive terms. Show

that for each s ≥ 1, the series n=1 asn is also convergent.
Example 10.28. Rearranging conditionally convergent series: There
is a rather strange fact, that illustrates the difference between an absolutely
convergent and a conditionally convergent series. First, we note that for an
absolutely convergent series, rearranging the terms does not affect the sum of
the series.

However, for a conditionally convergent series, rearranging the terms can


P∞affect
the sum of the series, and in fact, we can play a wonderful game. PLet n=0 an

be a conditionally
P∞ convergent series with non-zero terms, so that n=0 an con-
verges but n=0 |an | diverges. (The restriction to a series with non-zero terms
is not essential, but it makes the exposition aP
bit smoother.)
P∞ Choose any num-

ber S ∈ R. Then, there is a rearrangementP∞ n=0 b n of n=0 an (so the same
terms, but in a different order) so that n=0 bn converges to S.
P∞
Start by rewriting the original series n=0 an : set

an if an > 0;
pn =
0 if an ≤ 0;
and 
0 if an > 0;
qn =
an if an ≤ 0;
P∞ P∞
Note P that both n=0 pn and Pn=0 qn diverge:P∞ since an = pn + qn for all
∞ ∞
n, if n=0 pn converges, then P n=0 qn = n=0 (an − pn ) converges, by the

arithmetic of series. However, n=0 pn is a series of non-negative terms, P∞ for
which convergence and absolute convergence are the same notion, and n=0 qn
is a series of non-positive terms, for P
which convergence
P∞ and absolute convergence

are the same notion. But P∞if both n=0 pn and n=0 qn converge
P∞ absolutely,
then
P∞ so does their sum n=0 an , a contradiction. Hence, both n=0 pn and
n=0 q n diverge.

Given S, build the new series as follows: start by choosing elements b0 = p0 ,


b1 = p1 , . . . , bm = pm (ignoring all the pn that are equal to 0) from the series
P ∞ Pm Pm−1
n=0 pn until n=0 bn > S (but n=0 bn ≤ S). Then, choose elements
bm+1 = q0 , bm+2 = q1 , . . . , bm+k+1 = qk (ignoring all the qn that are equal to
Pm+k+1 Pm+k
until n=0 bn < S (but n=0 bn ≥ S). Then, choose the next elements
0) P

of n=0 pn (again ignoring the terms equalPto 0) until the sum is greater than

S, and then choose the next elements of n=0 qn (again ignoring the terms
equal to 0) until P the sum is less than S, and repeat
P∞ indefinitely. This gives

a rearrangement n=0 bn of the original series n=0 an . (Ignoring the terms
equal Pto 0 in constructing the bn means that the only terms appearing P∞ in the

series n=0 bn are the P∞ same as those
P∞ appearing in the original series n=0 an .)
The divergence of n=0 pn and n=0 qn enters into this construction, as it
ensures that we can in fact continue this process indefinitely.
78 PERIŠIĆ

P∞
It remains only to check that n=0 bn converges to S, but this follows immedi-
ately from the construction of this new series, and the fact that limn→∞ pn =
limn→∞ qn = 0.
MATH2039 ANALYSIS 79

11. Sequences and series of functions

A sequence/series of functions is a sequence/series where each term is a


function.
Definition 11.1. A sequence of functions (fn ), fn : S → R, n ∈ Z+ con-
verges pointwise to a function f : S → R if for each a ∈ S, the sequence of
real numbers (fn (a)) converges to f (a).
Definition 11.2. A sequence of functions (fn ), fn : S → R, n ∈ Z+ con-
verges uniformly on S to a function f : S → R if for each ε > 0, there exists
n0 so that if n > n0 , then |fn (a) − f (a)| < ε for each a ∈ S.

As with the difference between continuity and uniform continuity, the difference
between these two definitions is on one level small, merely the placement of a
quantifier, but it has major effects. To see this, if we rewrite the definition of
pointwise convergence, we get: A sequence of functions fn : S → R converges
pointwise to a function f : S → R if for each ε > 0 and for each a ∈ S, there
exists M so that if n > M , then |fn (a) − f (a)| < ε. Namely, the difference
is in the placement of quantifier for each a ∈ S. We demonstrate that these
two definitions are different in two steps, one on a theorem and the other on an
example.
Theorem 11.3. Suppose that (fn ) is a sequence of continuous functions fn :
S → R and suppose that (fn ) converges uniformly to f on S. Then f is
continuous on S.

Proof. We show that f is continuous at a. So, take an arbitrary ε > 0; we need


to show that there exists δ > 0 so that if |x − a| < δ, then |f (x) − f (a)| < ε.
Since (fn ) converges to f uniformly, there exists n0 so that if n > n0 , then
|fn (b)−f (b)| < 31 ε for all b ∈ S. We also know that fn0 +1 is continuous at a, and
so there exists some δ > 0 so that if |x−a| < δ, then |fn0 +1 (x)−fn0 +1 (a)| < 31 ε.
Therefore:
|f (x) − f (a)| = |f (x) − fn0 +1 (x) + fn0 +1 (x) − fn0 +1 (a) + fn0 +1 (a) − f (a)|
≤ |f (x) − fn0 +1 (x)| + |fn0 +1 (x) − fn0 +1 (a)| + |fn0 +1 (a) − f (a)|
1 1 1
< ε + ε + ε = ε,
3 3 3
and so f is continuous at a. (Here, the first and third inequalities follow from
the uniform convergence of (fn ) to f , while the middle inequality follows from
the continuity of fn0 +1 .) 
Example 11.4. For n ≥ 1, define fn : [0, 1] → [0, 1] by fn (x) = xn . Then,
(fn ) converges pointwise to the discontinuous function

0 for 0 ≤ x < 1
f (x) =
1 for x = 1
This is just a reflection of the fact that for 0 ≤ a < 1, the sequence an converges
to 0, but the rate of convergence depends on the value of a; if a is close to 0,
80 PERIŠIĆ

then the convergence is much quicker than if a is close to 1. Since the pointwise
limit of (fn ) is not continuous, we have by Theorem 11.3 that the convergence
of (fn ) to f cannot be uniform. (It is also possible to show that the convergence
of (fn ) to f cannot be uniform by direct application of the definition.)

X
Definition 11.5. For a series of functions fn , fn : S → R, let
n=0
k
X
sk (x) = fn (x), x ∈ S.
n=0
P∞
We say that the series of functions n=0 fn converges pointwise to a func-
tion s : S → R if the sequence of functions (sk ) converges pointwise to the
function s. P∞
We say that the series of functions n=0 fn converges uniformly to a func-
tion s : S → R if the sequence of functions (sk ) converges uniformly to s.
Theorem 11.6. ( Weierstrass M -test) Let (fn ), fn : S → R, n ∈ Z+ be a
sequence of functions, and suppose that there exist a sequence of real numbers
(Mn ) such that for each n we have
|fn (x)| ≤ Mn , ∀x ∈ S.
P∞
If the series of real numbers n=0 Mn converges, then the series of functions
P ∞
n=0 fn converges absolutely and uniformly on S.

P∞ First, we note that fixing any x in S we have that |fP


Proof. n (x)| ≤ Mn and

n=0 M n converges. Thus by the comparison test the series n=0 fnP
(x) con-

verges absolutely to some value. We define s(x) to be this value, thus n=0 fn
converges pointwise to s.
P∞
We now show that the convergence is uniform. Let M = n=0 Mn . Since this
converges, we know that given ε > 0 there exists k0 such that k > k0 implies
k
X
| Mn − M | < ε.
n=0
P∞ Pk
Equivalently, n=k+1 Mn < ε for all k > k0 . Let sk (x) = n=0 fn (x). By
P∞
definition of s we have s(x) − sk (x) = n=k+1 fn (x). For k > k0 we have

X ∞
X ∞
X
s(x) − sk (x) = fn (x) ≤ |fn (x)| ≤ Mn < ε
n=k+1 n=k+1 n=k+1

since we know that each sum converges, and we have fn (x) ≤ |fn (x)| ≤ Mn for
each n. Similarly

X ∞
X ∞
X
sk (x) − s(x) = −fn (x) ≤ |fn (x)| ≤ Mn < ε
n=k+1 n=k+1 n=k+1

since −fn (x) ≤ |fn (x)| ≤ Mn for each n. We deduce that for all k > k0 , and for
all x ∈ S we have |sk (x) − s(x)| < ε. Thus (sk ) converges uniformly to s. 
MATH2039 ANALYSIS 81

P∞
Corollary 11.7. Let n=0 fn be a series of continuous functions satisfying
the conditions of the Weierstrass M -test, and let s be the function this series
converges to. Then s is a continuous function.

Proof. By the Weierstrass M -test, the series converges uniformly to s. Hence


since the sequence of partial sums is a sequence of continuous functions, the
limit is continuous by Theorem 11.3. 
82 PERIŠIĆ

12. Power series

Power series are special examples of series of functions.


Definition 12.1. A real power series is a series of functions of the form

X
an (x − a)n ,
n=0

where the an are real numbers, where x is a real variable, and where a is a real
number.

The number a is called the centre of the power series .


P∞
The main question we ask about a power series n=0 an (x − a)n is, for what
values of x does this series converge?
P∞
Definition 12.2. The radius of convergence r of P a power series n=0 an (x − a)n

is the supremum of the set of all values s for which n=0 an (x − a)n converges
for all x ∈ [a − s, a + s],, i.e. the radius of convergence r is defined as
P∞
r = sup{s > 0 : n=0 an (x − a)n converges ∀x ∈ [a − s, a + s]}.

When a power series converges for all x in R then we P say that the radius of

convergence r is ∞. If r is the radius of convergence of n=0 an (x − a)n then
by definition the power series converges for x ∈ (a − r, a + r). Note that
[
[a − s, a + s] = (a − r, a + r).
s<r
P∞
Theorem 12.3. For a power series n=0 an (x − a)n with radius of conver-
gence r, the series converges for all x ∈ R such that |x − a| < r and it diverges
for all x ∈ R with |x − a| > r.

We defer the proof until later.


Note that the theorem says nothing about the points x = a ± r.
The set of points x for which a series converges is called the interval of con-
vergence.
It is possible for a power series to have a radius of convergence of 0. In this
case (by the theorem) the series converges only at x = a (where it converges to
a0 ). This case is not particularly interesting. The best possible case is when
the radius of convergence is ∞: this means that the series converges at every
point and hence the interval of convergence is all of R.
For 0 < r < ∞ the endpoints a − r and a + r are ‘critical’. Between these
points we have convergence and outside we have divergence. The behaviour at
the two endpoints must be checked by hand, as the series can either converge
or diverge at any of these points.
MATH2039 ANALYSIS 83

P∞
Example 12.4. Consider the power series n=0 xn /(n + 1), which is a power
series centred at a = 0. We always begin the same way with a power series, by
using the ratio test. The ratio test asks us to calculate
n+1
x /((n + 1) + 1) n+1
lim
n→∞ xn /(n + 1) = |x| n→∞
lim
n+2
= |x|.

By Proposition 10.22, this series converges absolutely for |x| < 1 and diverges
for |x| > 1. So, the radius of convergence is 1. Proposition 10.22 yields that the
open interval (−1, 1) lies in the interval of convergence. In order to determine
the interval of convergence, we need to check the behavior of the series at
the two endpoints of P∞this interval, namely x = 1 and x = −1. At x = 1,
the series becomes n=0 1/(n + 1), which P is the harmonic series and hence

diverges. At x = −1, this series becomes n=0 (−1)n /(n + 1), which is the
alternating harmonic series, and hence converges conditionally. So, the interval
of convergence is the half-open interval [−1, 1).
Exercise 12.5. The power series scavenger hunt: for each of the power
series given below, determine the radius and interval of convergence.
P∞ (−1)n n
(1) n=0 n! x ;

5n
P∞
(2) n=1 n2 xn ;

P∞ 1
(3) n=1 n(n+1) xn ;

P∞ (−1)n
(4) n=1

n
xn ;

P∞ (−1)n
(5) n=0 (2n+1)! x2n+1 ;

3n
P∞
(6) n=0 n! xn ;

P∞ 1
(7) n=0 1+n2 xn ;

P∞ (−1)n+1
(8) n=1 n (x + 1)n ;

3n
P∞
(9) n=0 4n (x + 5)n ;

P∞ (−1)n
(10) n=1 n2 +4 (x + 1)2n+1 ;

πn
P∞
(11) n=0 (2n+1)! (x − 1)2n ;

P∞ 1
(12) n=2 (ln n)n xn ;
84 PERIŠIĆ

P∞
(13) n=0 3 n xn ;

P∞ n!
(14) n=0 2n xn ;

P∞ (−2)n
(15) n=1 n+1 xn+1 ;

P∞ (−1)n
(16) n=1 (2n)! x2n ;

P∞ (−1)n
(17) n=1 n3/2 x3n ;

P∞ (−1)n+1
(18) n=2 n ln2 (n) xn ;

P∞ 1
(19) n=0 2n (x − 3)n ;

P∞ (−1)n
(20) n=1 (n+1)2 (x − 4)n ;

P∞ (2n+1)!
(21) n=0 n3 (x − 2)n ;

P∞ ln(n)
(22) n=1 n (x − 3)n ;

P∞ 1
(23) n=0 42n (2x − 3)n ;

P∞ 1
(24) n=2 bn (x − a)n , where b > 0 is arbitrary.

P∞ (n+p)!
(25) n=0 n!(n+q)! xn , where p, q ∈ N;

P∞ 1
(26) n=1 n3n xn−1 ;

P∞ (−1)n−1
(27) n=1 (2n−1)! x2n−1 ;

P∞
(28) n=1 n!(x − a)n , where a ∈ R is arbitrary;

P∞ n
(29) n=1 2n (3n−1) (x − 1)n .
1/n
Exercise 12.6. Prove,Pif∞ an is an sequence satisfying limn→∞1 |an | = L 6= 0,
then the power series n=0 an x has radius of convergence L .

Note that, if we use the ratio test to determine the radius of convergence of a
power series, we cannot then use the ratio test to determine whether the series
converges or diverges at the endpoints of the interval of convergence. This is
MATH2039 ANALYSIS 85

because the limit is equal to 1 at the endpoints of the interval, and when the
limit is 1 is precisely when the ratio test gives no information.
Exercise 12.7. For each of the following series, determine the values of x for
which the series converges.
P∞  n
1 x+2
(1) n=1 2n−1 x−1 ;
P∞ 1
(2) n=1 (x+n)(x+n−1) .
P∞
Exercise 12.8. Suppose n=0 an (x − a)n P is a power series with a radius of

convergence r. Show that the power series n=1 nan (x − a)n−1 also has the
radius of convergence r.
Theorem 12.9.
P∞ (Arithmetic of powerPseries) ∞
Let f (x) = n=0 an (x − a)n and g(x) = n=0 bn (x − a)n be two power series
with radii of convergence r1 and r2 respectively. Let r be the minimum of r1 , r2 ;
i.e. r = min{r1 , r2 }. Then, on the open interval (a − r, a + r) the following
holds:
(i) The sum (f + g)(x) is given by the power series

X
(f + g)(x) = (an + bn )(x − a)n .
n=0

(ii) The difference (f − g)(x) is given by the power series


X∞
(f − g)(x) = (an − bn )(x − a)n .
n=0

(iii) The product (f · g)(x) is given by the power series


X∞
(f · g)(x) = cn (x − a)n ,
n=0
Pn
where cn = k=0 ak · bn−k .

(iv) The derivative of f (x) is given by differentiating the power series term
by term:
X∞
f 0 (x) = n an (x − a)n−1 .
n=1

(v) The (indefinite) integral of f (x) is given by integrating the power


series term by term:
Z ∞
X an
f (x)dx = c + (x − a)n+1 .
n=0
n +1
86 PERIŠIĆ

13. Taylor series

We have seen that a power series centred at a with radius of convergence r


converges to a continuous and infinitely differentiable function on (a − r, a + r).
Now we want to reverse the process: starting from a function, can we find a
power series converging to it?
Suppose that
P∞ n 2 n n+1
n=0 an (x − a) = a0 +a1 (x−a)+a2 (x−a) +· · ·+an (x−a) +an+1 (x−a) +. . .
converges to f . Then
P∞
n=1 nan (x − a)
n−1
= a1 +2a2 (x−a)+· · ·+nan (x−a)n−1 +(n+1)an+1 (x−a)n +. . .
converges to f 0 .
Similarly,
2a2 + 3 · 2a3 (x − a) + · · · + n(n − 1)an (x − a)n−2 + (n + 1)nan+1 (x − a)n−1 + . . .
converges to f 00 , and in general
(n(n − 1)(n − 2) × · · · × 2)an + ((n + 1)n(n − 1) × · · · × 2)an+1 (x − a) + ((n +
2)(n + 1)n × · · · × 3)an+2 (x − a)2 + · · ·
converges to f (n) . Thus we have f (n) (a) = n!an for each n. Solving for an we
make the following definition.
Definition 13.1. For a function f that has derivatives of all orders on (a −
r, a + r), we define the Taylor series centred at a to be the power series

X f (n) (a)
(x − a)n ,
n=0
n!

where f (n) (a) denotes the nth derivative of f (x) evaluated at a.

Exercise 13.2. For each of the given functions, calculate its Taylor series about
the given point; also, determine the radius and interval of convergence of the
resulting power series wherever possible.
(1) f (x) = x3 + 6x2 + 5x − 7 about a = 6;
(2) f (x) = e3x about a = −2;
(3) f (x) = cosh(x) about a = 1.
Lemma P∞ 13.3. (UniquenessPof series representations) Consider two power

series n=0 an (x − a)n and n=0 bn (x − a)n that both converge absolutely and
uniformly on the open interval (a − ε, a + ε) for some ε > 0. If

X ∞
X
an (x − a)n = bn (x − a)n , ∀x ∈ (a − ε, a + ε),
n=0 n=0

then an = bn for all n ≥ 0.


MATH2039 ANALYSIS 87

We now present a test to determine when a function is equal to its Taylor


series, followed by an example of a function that is NOT equal to its Taylor
series.

Let f be a function which has derivatives of all orders in the interval (a−r, a+r),
for some r > 0. Then by definition, the series

X f (n) (a)
(x − a)n
n=0
n!

converges to f (x) if and only if


Rk (x) = f (x) − sk (x)
converges to zero as k → ∞.
Theorem 13.4. (Taylor’s theorem)
Suppose f is (k + 1)-times differentiable on (a − r, a + r) for some r > 0. Let
k
X f (n) (a)
sk (x) = (x − a)n .
n=0
n!

Then for each x ∈ (a − r, a + r) with x 6= a, there exists c between a and x such


that
f (k+1) (c)
f (x) = sk (x) + (x − a)k+1
(k + 1)!

The theorem says that for each x, k there exists c such that
f (k+1) (c)
Rk (x) = (x − a)k+1 .
(k + 1)!
Thus we have the following corollary.
Corollary 13.5. Let f be a function which has derivatives of all orders in the
(k+1)
interval (a−r, a+r). Suppose that x ∈ (a−r, a+r), x 6= a, and f (k+1)!(c) (x−a)k+1
converges to zero uniformly in c as k → ∞ (for x, a fixed). Then the series

X f (n) (a)
(x − a)n
n=0
n!

converges to f (x).
Example 13.6. Consider the Taylor series for the function f (x) = ex centred
at a = 0, namely
∞ ∞
X f (n) (0) n X 1 n
x = x ,
n=0
n! n=0
n!
since f (n) (0) = e0 = 1 for all n ≥ 0. In order to show that

X 1 n
ex = x
n=0
n!
88 PERIŠIĆ

for all x in R, we need to show that for each x, Rk (x) → 0. We do this by


estimating Rk (x). By Taylor’s Theorem we have
ec
Rk (x) = xk+1 → 0,
(k + 1)!
where c lies between 0 and x and depends on x and k.

First consider x > 0. Let M = ex . For all c ∈ (0, x), since ex is an increasing
1 k
function, we have that 1 < ec ≤ ex = M. We know that limk→∞ k! x = 0
M k
and limk→∞ k! x = 0, for fixed x. Now the squeeze rule for limits yields that
ec
limk→∞ (k+1)! xk+1 = 0 for each x, as desired.

In the case that x ≤ 0, we note that 0 < ex ≤ 1, and the same argument applies
here as well. Since limk→∞ Rk (x) = 0 for each x, we have that f (x) = ex is
equal to its Taylor series.
Example 13.7. WePcan pass between functions and series in the following
∞ n
way: a power series n=0 aP n (x − a) defines a function on its interval of con-

vergence I, namely f (x) = n=0 an (x − a)n for every x ∈ I (since the interval
of convergence is precisely the set of values of x for which the power series
converges).

We can then take the Taylor series for f at any point a in the interval of
convergence I. When we do this, we get back the power series we started with,
since power series representations of functions are unique.

However, when we perform the other possible composition of these operations,


namely start with a function f , construct its Taylor series which then has an
interval of convergence I, and then look at the function on I given by summing
the series, we do NOT necessarily get back the function we started with. The
easiest way to see this is via an example. So, consider the function
 2
e−1/x for x 6= 0
f (x) =
0 for x = 0
Calculating one finds that f (n) (0) = 0 for all n ≥ 0, and so the Taylor series
for this function centred at 0 is
∞ ∞
X 1 (n) X
f (0)xn = 0 xn ,
n=0
n! n=0

which is the series representation of the constant function g(x) = 0, which is


not the function f we began with.
Example 13.8. Determine a series representation for the function f (x) =
(x + 1)/(x + 2) centred at a = 0.

One way would be to calculate the Taylor series for f (x) centred at a = 0,
but this gets complicated, as the derivatives of f (x) get complicated. Another
way is to use the arithmetic of power series. We start by deriving a series
MATH2039 ANALYSIS 89

1
P∞
representation for 1/(x + 2), using the fact that 1−r = n=0 rn for |r| < 1.
Hence, for | − 12 x| < 1, we have:
1 1
=
x+2 2( 12 x + 1)
1 1
=
2 1 − (− 12 x)
∞  n
1X 1
= − x
2 n=0 2

X 1 n
= (−1)n x .
n=0
2n+1
Hence, a series representation for f (x) centred at 0 is:

x+1 X 1
f (x) = = (x + 1) (−1)n n+1 xn
x+2 n=0
2
∞ ∞
X 1 X 1
= (−1)n x n+1
+ (−1)n n+1 xn
n=0
2n+1 n=0
2
∞ ∞
X 1 n X
n+1 1
= (−1) n
x + (−1)n n+1 xn
n=1
2 n=0
2
∞  
1 X
n+1 1 n 1
= + (−1) + (−1) n+1 xn
2 n=1 2n 2

1 X 1
= + (−1)n+1 n+1 xn .
2 n=1 2
90 PERIŠIĆ

14. The exponential function

In this section we will apply what we have learned to understanding the function
ex . To begin with we need to define the function ex .
Let c be any constant greater than 1. Then we can define a function cx . If
x ∈ Z this is defined algebraically:

c · c · · · · · c(x times)
 x>0
x
c = 1 x=0
 1

c·c·····c (x times) x<0

√ algebraically for rational numbers. If x = p/q ∈ Q we define


This is also defined

cx = ( q c)p = q cp .
Exercise 14.1. Prove the following:
(1) For p, q and p0 , q 0 in Z with p/q = p0 /q 0 we have
√ √ √
0 0 √
q0
( q c)p = q cp = q c)p = cp0 .
Therefore for x rational, cx is well defined.
(2) For x, y ∈ Q, we have cx+y = cx cy .
(3) For x ∈ Q, if x > 0 then cx > 1.
(4) For x, y ∈ Q, if x < y then cx < cy . Hint: use 2 and 3.
Lemma 14.2. For any c > 1 the sequence c1/k tends to one as k → ∞, i.e.
1
lim c k = 1.
k→∞

This is example 4.48, but we’ll repeat it here.

Proof. Let ak = c1/k − 1. The result is equivalent to showing that ak → 0. For


each k the binomial formula gives us
k  
X k n k(k − 1) 2
c = (1 + ak )k = a = 1 + kak + ak + · · · + akk ≥ 1 + kak ,
n k 2
n=0

hence ak ≤ (c − 1)/k. On the other hand since 1/k > 0 we know that c1/k > 1,
so we also know that ak ≥ 0. It follows from the squeeze rule for sequences
that ak → 0, so c1/k = 1 + ak → 1 as k → ∞. 

This analysis allows us to define cx for any real x.


Definition 14.3. Given c > 1 and x ∈ R. Let xn be an increasing sequence of
rational numbers tending to x (for example truncations of the decimal expansion
of x). Then cx is defined to be the limit of cxn , i.e. cx = limn→∞ cxn .

Why does this definition make sense? We use the following lemma.
MATH2039 ANALYSIS 91

Lemma 14.4. If xn is an increasing sequence with xn ∈ Q and xn → x ∈ R,


then cxn converges. Moreover if yn is any other sequence of rational numbers
with yn → x, then cyn converges and limn→∞ cxn = limn→∞ cyn .

Proof. First we must show that cxn converges. Since xn is increasing, we know
that cxn must also be an increasing sequence. If s is any rational number greater
than x, then s > xn for all n, so cxn ≤ cs for all n. The sequence cxn is bounded
and monotonic, therefore it converges.
To justify this we will show that if we picked any other sequence of rational
numbers yn converging to x then the limit of cyn would be the same as the limit
of cxn , so cx is in fact the limit of cyn for any rational sequence tending to x.
The proof is as follows.
We must now show that for any sequence yn with yn → x as n → ∞, we
have limn→∞ cxn = limn→∞ cyn . The difference yn − xn converges to 0, so by
definition, for any k we can find M such that if n > M then |yn − xn | < 1/k.
Thus we have c−1/k < cyn −xn < c1/k for n > M . However by the previous
lemma we know that c1/k → 1 and so c−1/k = 1/c1/k also tends to 1. It
follows from the squeeze rule for sequences, that cyn /cxn = cyn −xn → 1. Hence
limn→∞ cyn = limn→∞ cxn . 

We have shown that cx is a well defined function from R to (0, ∞). We can
now carry out the following exercises.
Exercise 14.5. Prove the following:
(1) For x, y ∈ R, we have cx+y = cx cy .
(2) For x, y ∈ Q, if x < y then cx < cy .

We will do the following exercise: Show that cx is continuous.

We must show for all x0 , that given ε > 0, there exists δ > 0 such that
|x − x0 | < δ implies |cx − cx0 | < ε. Write
cx − cx0 = (cx−x0 − 1)cx0 .
We know that the sequences c1/k and c−1/k tend to 1 as k → ∞, therefore there
exists k such that |c1/k − 1| and |c−1/k − 1| are both less than cxε0 .
Take δ = 1/k. If |x − x0 | < δ, then −1/k < x − x0 < 1/k, so c−1/k < cx−x0 <
c1/k . This implies
ε ε
− x0 < c−1/k − 1 < cx−x0 − 1 < c1/k − 1 < x0 .
c c
Multiplying through by cx0 we get
−ε < (cx−x0 − 1)cx0 = cx − cx0 ε
as required.

The sequence ck tends to infinity, as k → ∞ (example 4.46), hence c−k tends to


zero as k → ∞. It follows that the function cx maps R onto (0, ∞): Given any
92 PERIŠIĆ

y > 0 we know that for some a, b (such as a = −k, b = k for k large enough)
we have ca < y < cb . Therefore by the intermediate value property there is
a solution to cx = y. Hence there is an inverse function logc x from (0, ∞) to
R.

We will now show that the function cx is differentiable. We must show that
x+h x
the limit of c h−c as h tends to 0 exists. A first simplification is to write this
h h
as cx c h−1 . It is therefore sufficient to show that c h−1 converges to some limit
L as h tends to 0, in other words that the function is differentiable at 0. Note
d
that once we have shown this we will have shown that dx (cx ) = Lcx where the
ch −1
constant L = limh→0 h is the derivative at 0.
ch −1
Consider the two sides of the limit separately limh→0+ h and

ch − 1 1 − c−|h| c−|h| (c|h| − 1)


lim− = lim− = lim− .
h→0 h h→0 −h h→0 |h|

ch −1 c|h| −1
We know that c−|h| tends to c0 = 1 as h → 0, so limh→0− h = limh→0− |h| .
h
But this is the same as limh→0+ c h−1 . In other words, if we can show that
h h
limh→0+ c h−1 exists, then so does limh→0− c h−1 , and the two are equal as re-
quired.
ch −1
Lemma 14.6. The limit limh→0+ h exists.

Proof. We have h positive, and are considering the limit as h tends to zero. We
can therefore assume that h is less than 1, in which case we can find k such
1
that k+1 ≤ h ≤ k1 . Then

ch − 1
k(c1/(k+1) − 1) ≤ ≤ (k + 1)(c1/k − 1).
h
Let ak = c1/k − 1 as above, and let bk = kak . The sequences (k + 1)(c1/k − 1)
is equal to k+1k bk , so this converges if and only if bk converges. Similarly
1/(k+1) k
k(c − 1) = k+1 bk+1 , which converges if and only if bk+1 converges, or
equivalently bk converges.

We therefore have the following problem: to show that bk = kak converges to a


limit, where ak = c1/k − 1.

We previously showed that ak converges to 0 by showing that kak is bounded


above by c − 1. We will now use the same identity
k  
k
X k
c = (1 + ak ) = ank
n
n=0

to show that kak is decreasing. As kak is bounded below by 0 it will therefore


follow that the sequence tends to a limit.
MATH2039 ANALYSIS 93

 
Pk k n Pk
Write the sum in the form n=0 a = n=0 bk,n (kak )n where the coeffi-
n k
cients bk,n are given by the formula
   n
k 1 k! k(k − 1)(k − 2) . . . (k − (n − 1))
bk,n = = =
n k n!(k − n)!k n n!k · k · k . . . k
    
1 1 2 n−1
= 1− 1− ... 1 − .
n! k k k
Note that for each n, as k increases bk,n increases, and tends to 1/n! as k → ∞.
Pk
The sum n=0 bk,n (kak )n is a constant (for each k it adds up to c), the number
of terms increases, and for each n the coefficient of (kak )n increases. Therefore,
so that the sum can remain constant, kak must be decreasing. As it is bounded
below it converges, which completes the proof. 

We conclude that for any c > 1 the function cx is well defined for all real values
x, and is differentiable with derivative Lcx where L = limk→∞ kak equals the
derivative of cx at 0. We’ll now pick a special value of c, namely c = e. This
value is suppose to have the property that ex has derivative ex . Equivalently
limk→∞ k(e1/k − 1) = 1.

We define e to be the limit of (1 + 1/k)k as k tends to ∞.

Lemma 14.7. The sequence (1 + 1/k)k converges as k tends to ∞.

Proof. Expanding (1 + 1/k)k by the binomial formula. We get


k   k
k
X k X
(1 + 1/k) = (1/k)n = bk,n
n
n=0 n=0
 
k 1 n

where bk,n = k as above.
n

(Note: Right away we see that this has something to do with the above com-
putation of derivatives of cx .)

We noted, in the proof of the previous lemma, that the terms bk,n are increasing
k
and are bounded
P∞ 1 above by 1/n!. Hence (1 + 1/k) is increasing and bounded
above by n=0 n! . This sum is a finite number: we know the series converges
by the ratio test. Therefore the sequence (1 + 1/k)k converges. 

P∞ 1
The above argument shows us more: in fact the sequence converges to n=0 n! .
P∞ 1 Pp 1
P ∞ 1
Given ε > 0, for p sufficiently large, n=0 n! − n=0 n! = n=p+1 n! < ε/2.
On the other
Pp hand for P each n, the term bk,n → 1/n! as k → ∞, so the fi-
p 1
nite sum n=0 bk,n → n=0 n! . Hence for k sufficiently large (and at least
Pp 1
Pp
p) we have n=0 n! − n=0 bk,n < ε/2. Putting all of this together we find that
94 PERIŠIĆ

∞ k ∞ p
X 1 X X 1 X
− bk,n ≤ − bk,n
n=0
n! n=0 n=0
n! n=0
∞ p p p
! !
X 1 X 1 X 1 X
= − + − bk,n < ε.
n=0
n! n=0 n! n=0
n! n=0
Pk P∞ 1
Therefore (1 + 1/k)k = n=0 bk,n converges to n=0 n! . In other words

X 1
e = lim (1 + 1/k)k = .
k→∞
n=0
n!

We will finish our discussion of ex by computing its derivative.


Lemma 14.8. The derivative of ex is ex .

d x
Proof. We have already seen that dx e must be of the form Lex where L =
1/k
limk→∞ kak and ak = e − 1. We will show that this limit is 1.
Recall we have the identity
k
X
k
e = (1 + ak ) = bk,n (kak )n
n=0
 
k 1 n

where bk,n = . On the other hand, by definition e is the limit of the
n k
Pk
increasing sequence (1 + 1/k)k = n=0 bk,n as k tends to infinity. We have
k
X k
X k
X
bk,n ((kak )n − 1) = bk,n (kak )n − bk,n = e − (1 + 1/k)k
n=0 n=0 n=0
k
and we know that e − (1 + 1/k) is a positive, decreasing, and tends to 0 as
k → ∞. Thus kak > 1 for all k, so each term of the sum is positive. Moreover
as the sum tends to zero, each individual term bk,n ((kak )n 
− 1) must also tend
k
to 0 as k → ∞. In particular setting n = 1, we have bk,1 = (1/k)1 = 1, for
1
each k, so kak − 1 = bk,1 ((kak )1 − 1) → 0.
Hence we have shown that for ak = e1/k − 1 the limit of kak as k → ∞ is 1, so
the derivative of ex is ex . 
MATH2039 ANALYSIS 95

Appendix 0 Some Historic notes

Augustin-Louis Cauchy was one of the greatest mathematicians


of all times, made pioneering contribution to many branches of mathematics
and physics, one of the first to rigorously prove theorems of calculus. Only in
mathematics there are roughly fifty concepts named after Cauchy.

Weierstrass function is a continous function that is not dif-


ferentiable at any point of a real line named by German mathematician Karl
Weierstrass.

Fibonacci numbers appear in nature suprisingly often.


Number of spirals in a sunflower or pineapple or numbers of petals in a flower.
Fibonacci numbers have also couple interesting properties, sum of two consecu-
tive, squared Fibonacci numbers is also a Fibonacci number, so called a ’Golden
Ratio’ is also associated with Fibonacci sequence.

Look TEDxTalk ’The magic of Fibonacci numbers’ by Arthur Benjamin.

This historic notes have been selected by Mikolaj Kacki, Year 2 student, aca-
demic year 2018/19.
96 PERIŠIĆ

Appendix A: Solutions to selected exercises


Solutions to Exercises, Chapter 2
Solution 2.5:
(1) Each element a ∈ F has an additive inverse −a with a + (−a) = 0.
Multiply both sides by b and apply Example 2.3 we get (a+(−a))·b = 0.
Now apply the distributive law to get a · b + (−a) · b = 0. The element
a·b has additive inverse −(a·b). Adding this to both sides and applying
associativity and commutativity of addition we get

a · b + (−(a · b)) + (−a) · b = 0 + (−(a · b)).
The left hand side equals (−a)·b while the right hand side equals −(a·b),
proving that (−a) · b = −(a · b) as required. Similarly for a · (−b).
(2) Start with 1 + (−1) = 0, and multiply on the right by a. Since 0 ·
a = 0, this becomes (1 + (−1)) · a = 0. Expanding out, this becomes
1 · a + (−1) · a = 0. Since 1 is the multiplicative identity, this becomes
a + (−1) · a = 0. Adding −a to both sides and simplifying, this becomes
(−1) · a = −a, as desired.

Solution 2.7: Write n as a product n = a · b, where 2 ≤ a, b < n, so that


a and b are not equal in Zn . Then, in Zn , the product a · b is 0, being a
multiple of n. However, if Zn were a field, then a would have a multiplicative
inverse a−1 , and we could multiply both sides of a · b = 0 on the left to obtain
a−1 · a · b = a−1 · 0, which simplifies to b = 0. This contradicts the choice of b
to satisfy 2 ≤ b < n, and so a has no multiplicative inverse, contradicting the
definition of a field.

Solution 2.10: For every a we have either 0 < a or a = 0 or a < 0. Thus


for a 6= 0 either 0 < a or a < 0. In the first case (since 0 < a) we can
multiply the inequality by a to get 0 · a < a · a. Thus (since 0 · a = 0) we have
0 < a · a as required. In the second case, a < 0, adding −a to both sides we get
a + (−a) < 0 + (−a), so 0 < −a. By the previous case we have 0 < (−a) · (−a).
Now (−a)·(−a) = −(−(a·a)) by Exercise 2.5 and −(−(a·a)) = a·a by Example
2.4. Thus we again have 0 < a · a.

Solution 2.12: We begin by showing that the natural numbers N = {0, 1, 2, 3...}
lie in our ordered field F : We have seen in the lectures that 0 < 1 and hence
1 + 1 ∈ F and also 1 = 0 + 1 < 1 + 1. Therefore we can conclude that 1 6= 1 + 1.
Also, by transitivity, 0 < 1 + 1 and hence 0 6= 1 + 1. Continuing this process
we obtain elements 1 + 1 + ... + 1 ∈ F , such that for all non-negative inte-
| {z }
n
gers n 6= m we have that 1 + 1 + ... + 1 6= 1 + 1 + ... + 1 . Now rename these
| {z } | {z }
n m
elements as follows: 1 + 1 + ... + 1 =: n and we have shown that
| {z }
n

{0, 1, 2, ...} ⊂ F.
MATH2039 ANALYSIS 97

Now, by the field axioms it follows that for each n ∈ F , also −n = −(1 + ... + 1) =
| {z }
n
(−1) + ...(−1) ∈ F and by the ordering axioms −n < 0 and an argument
| {z }
n
similar to above shows that for n 6= m also −n 6= −m. Furthermore, for all
n, m ∈ N\{0} we have −n 6= m as −n < 0 and m > 0. Hence
Z = {..., −2, −1, 0, 1, 2, ...} ⊂ F.
n
Each element in Q is of the form m with n, m ∈ Z. By the field axioms, we
have m−1 ∈ F and hence n · m−1 = m n
also lies in F . Hence we can conclude
that Q ⊂ F.

Solution 2.13: Suppose there were such an order on C, and denote it by <.
Compare 0 and i. Since 0 6= i, it must be that either 0 < i or i < 0.

Suppose that 0 < i. Multiplying both sides by i and remembering that 0 < i,
we see that 0 · i < i · i, which simplifies to 0 < −1. Adding 1 to both sides, we
see that 1 < 0. Again multiplying both sides by i and remembering that 0 < i,
we see that 1 · i < 0 · i, which simplifies to i < 0. Hence, if 0 < i, then i < 0,
contradicting the second condition in the definition of an order.

Suppose now that i < 0. Adding the additive inverse −i of i to both sides, we get
that 0 < −i. Multiplying both sides by −i, we get that 0 · (−i) < (−i) · (−i), and
so 0 < −1. Multiplying both sides by −i again, we get that 0 < (−1) · (−i) = i.
Hence, if i < 0, then then 0 > i, again contradicting the second condition in
the definition of an order.

Hence, since we have that neither 0 < i nor i < 0, we see that there cannot
exist an order on C that makes C into an ordered field.
98 PERIŠIĆ

Solutions to Exercises, Chapter 3

Solution 3.13:
(1) Bounded above by 1 (since for n ∈ Z − {0}, either n ≥ 1 in which case
1 1
n ≤ 1, or n ≤ −1, in which case n ≤ 0), and so has a supremum. Again
making use of Exercise 3.16, since 1 is an upper bound for S and since
1 ∈ S, 1 = sup(S). In this case, sup(S) ∈ S.

Bounded below by −1 (since for n ∈ Z − {0}, either n ≥ 1, in which


case 0 < n1 , or n ≤ −1, in which case n1 ≥ −1
1
= −1), and so has an
infimum. Again making use of Exercise 3.16, since −1 is a lower bound
for S and since −1 ∈ S, −1 = inf(S). In this case, inf(S) ∈ S.

Since S is both bounded above and bounded below, it is bounded.


(2) Bounded below by 0 (since 2x > 0 for all x ∈ R, we certainly have that
2x > 0 for all x ∈ Z), and so has an infimum. Given any ε > 0, we can
always find x so that 2x < ε, namely take log2 of both sides, and take
x to be any integer less than log2 (ε). Hence, there is no positive lower
bound, and so the greatest lower bound, the infimum, is inf(S) = 0.
Since there are no solutions to 2x = 0, in this case inf(S) 6∈ S.

Since 2x > x for positive integers x, given any C > 0 we can find an
x so that 2x > C, and so there is no upper bound. That is, S is not
bounded above.

Since S is not bounded above, it is not bounded.


(3) Bounded below by −1 (since [−1, 1] = {x ∈ R | − 1 ≤ x ≤ 1} and since
−1 < 5), and so has an infimum. Again making use of Exercise 3.16,
since −1 is a lower bound for S and since −1 ∈ S, −1 = inf(S). In this
case, inf(S) ∈ S.

Bounded above by 5, and so has a supremum. Again making use of


Exercise 3.16, since 5 is an upper bound for S and since 5 ∈ S, 5 =
sup(S). In this case, sup(S) ∈ S.

Since S is both bounded above and bounded below, it is bounded.


(4) Considering the subset of S in which y = 1, we have that S contains
the natural numbers N, and hence S is not bounded above.

Since x and 2y are both positive for x, y ∈ N, we have that 2xy > 0 for all
x, y ∈ N. Therefore, S is bounded below by 0, and so has an infimum.
Considering the subset of S in which x = 1, we have that S contains
1
2y for all y ∈ N. In particular, for each ε > 0, we can find y ∈ N so
that 21y < ε, namely take log2 of both sides to get −y < log2 (ε), or
equivalently y > log2 (ε). Hence, there is no positive lower bound, and
so 0 = inf(S). Since 2xy is never 0 for x, y > 0, in this case inf(S) 6∈ S.
MATH2039 ANALYSIS 99

(5) Break S up into two subsets, one of the positive terms (when n is even)
and the negative terms (when n is odd). So, S = {−2, − 34 , − 65 , . . .} ∪
{ 32 , 45 , 67 , . . .}.

The positive terms are all of the form 1 + n1 where n is even. Since n1
decreases as n increases, the largest positive term is 1 + 21 = 32 , and so
S is bounded above and hence has a supremum. Since S is bounded
above by 32 and since 32 ∈ S, sup(S) = 32 , and in this case sup(S) ∈ S.

The negative terms are all of the form 1 + n1 where n is odd. Since
1 1
n decreases as n increases, − n increases as n increases, and so the
−1−1
smallest negative term is 1 = −2, and so S is bounded below and
hence has an infimum. Since S is bounded below by −2 and since
−2 ∈ S, inf(S) = −2, and in this case inf(S) ∈ S.

Since S is both bounded above and bounded below, it is bounded.


(6) Rewrite S as S = (−∞, −2)∪(2, ∞). This set is neither bounded above
(since for each real number r, there is s ∈ S with s > r, namely the
larger of 3 and r + 1) nor bounded below (since for each real number r,
there is s ∈ S with s < r, namely the smaller of −3 and r − 1).

Since S is not bounded below, it has no infimum. Since S is not bounded


above, it has no supremum.

Since S is neither bounded above not bounded below, it is not bounded.

Solution 3.16:
(1) The easiest way to do this is to begin with an intermediate fact: if
A ⊂ B and if sup(B) exists, then sup(A) exists and sup(A) ≤ sup(B).
The proof uses the definition of supremum: since sup(B) exists, we
have that b ≤ sup(B) for all b ∈ B and that if u is an upper bound for
B, then sup(B) ≤ u. Since b ≤ sup(B) for all b ∈ B and since A ⊂ B,
we have that a ≤ sup(B) for all a ∈ A. In particular, A is bounded
above, and so sup(A) exists. To see the second statement, note that
since sup(B) is an upper bound for A, we have that sup(A) ≤ sup(B)
by definition.

So, since A ∩ B ⊂ A, we have that sup(A ∩ B) ≤ sup(A). Similarly,


A ∩ B ⊂ B, and so sup(A ∩ B) ≤ sup(B). Hence, sup(A ∩ B) ≤
min(sup(A), sup(B)).

To have an example in which sup(A ∩ B) < min(sup(A), sup(B)), take


A = {0, 1} and B = {0, 2}. Then, sup(A) = 1, sup(B) = 2, and
sup(A ∩ B) = 0 since A ∩ B = {0}.
(2) The easiest way to do this is to begin with an intermediate fact: if
A ⊂ B and if inf(B) exists, then inf(A) exists and inf(A) ≥ inf(B).
The proof uses the definition of infimum: since inf(B) exists, we have
100 PERIŠIĆ

that b ≥ inf(B) for all b ∈ B and that if t is a lower bound for B, then
inf(B) ≥ t. Since b ≥ inf(B) for all b ∈ B and since A ⊂ B, we have
that a ≥ inf(B) for all a ∈ A. In particular, A is bounded below, and
so inf(A) exists. To see the second statement, note that since inf(B) is
a lower bound for A, we have that inf(A) ≥ inf(B) by definition.

So, since A ∩ B ⊂ A, we have that inf(A ∩ B) ≥ inf(A). Similarly,


A ∩ B ⊂ B, and so inf(A ∩ B) ≥ inf(B). Hence, inf(A ∩ B) ≥
max(inf(A), inf(B)).

We note that it is possible to construct an example in which inf(A∩B) >


max(inf(A), inf(B)). Namely, take A = {−1, 0} and B = {−2, 0}.
Then, inf(A) = −1, inf(B) = −2, and inf(A∩B) = 0 since A∩B = {0}.
(3) Since t is a lower bound for A, we have that t ≤ inf(A), by the definition
of infimum. (And note that inf(A) exists since A is bounded below.)
Since t ∈ A, we also have that t ≥ inf(A). Since t ≤ inf(A) and
t ≥ inf(A), it must be that t = inf(A).
(4) Set X = {y | y is a lower bound for A}. By definition, inf(A) ∈ X,
since inf(A) is a lower bound for A. Now take any element y of X, so
that y is a lower bound for A. Again by the definition of the infimum,
y ≤ inf(A). So, inf(A) is an upper bound for X and inf(A) ∈ X, and
so inf(A) = sup(X) = sup{y | y is a lower bound for A}. (Note that
the assumption that inf(A) exists is equivalent to the assumption that
A is bounded below, which insures that X is non-empty.)
(5) Set X = {y | y is an upper bound for A}. By definition, sup(A) ∈ X,
since sup(A) is an upper bound for A. Now take any element y of X,
so that y is an upper bound for A. Again by the definition of the supre-
mum, y ≥ sup(A). So, sup(A) is a lower bound for X and sup(A) ∈ X,
and so sup(A) = inf(X) = inf{y | y is an upper bound for A}. (Note
that the assumption that sup(A) exists is equivalent to the assumption
that A is bounded above, which insures that X is non-empty.)
(6) This one we argue by contradiction. Suppose that a set A has two
suprema, and call them x1 and x2 . Both x1 and x2 are upper bounds
for A, by definition. Since x1 is a supremum for A, it is less than or
equal to all other upper bounds, and so x1 ≤ x2 . Similarly, since x2 is
a supremum for A, it is less than or equal to all other upper bounds,
and so x2 ≤ x1 . Since x1 ≤ x2 ≤ x1 , it must be that x1 = x2 , and so
the supremum of A is unique. (Note that this exercise justifies why we
call it ’the supremum’ instead of ’a supremum’.)
(7) This one we argue by contradiction. Suppose that a set A has two
infima, and call them x1 and x2 . Both x1 and x2 are lower bounds for
A, by definition. Since x1 is an infimum for A, it is greater than or
equal to all other lower bounds, and so x1 ≥ x2 . Similarly, since x2 is
an infimum for A, it is greater than or equal to all other upper bounds,
and so x2 ≥ x1 . Since x1 ≥ x2 ≥ x1 , it must be that x1 = x2 , and so
MATH2039 ANALYSIS 101

the infimum of A is unique. (Note that this exercise justifies why we


call it ’the infimum’ instead of ’an infimum’.)

Solution 3.17:
(1) Since sup(A) exists, the set A is bounded above. Let u be any upper
bound for A, so that a ≤ u for all a ∈ A. Multiplying through by
−1, this becomes −a ≥ −u for all a ∈ A. Since −a ranges over all of
A− as a ranges over A, this yields that −u is a lower bound for A− ,
and so inf(A− ) exists. In particular, taking u = sup(A), we have that
− sup(A) is a lower bound for A− .

To see that there is no lower bound for A− that is greater than − sup(A),
note that t is a lower bound for A− if and only if −t is an upper
bound for A. Therefore, a lower bound for A− greater than − sup(A)
exists if and only if an upper bound for A less than sup(A) exists,
but by the definition of supremum no such upper bound can exist.
Hence, − sup(A) is the greatest lower bound for A− , or in other words,
− sup(A) = inf(A− ), as desired.
(2) Since inf(A) exists, the set A is bounded below. Let t be any lower
bound for A, so that a ≥ t for all a ∈ A. Multiplying through by −1,
this becomes −a ≤ −t for all a ∈ A. Since −a ranges over all of A− as
a ranges over A, this yields that −t is an upper bound for A− , and so
sup(A− ) exists. In particular, taking t = inf(A), we have that − inf(A)
is an upper bound for A− .

To see that there is no upper bound for A− that is less than − inf(A),
note that u is an upper bound for A− if and only if −u is a lower bound
for A. Therefore, an upper bound for A− less than − inf(A) exists if
and only if a lower bound for A greater than inf(A) exists, but by the
definition of infimum no such lower bound can exist. Hence, − inf(A) is
the least upper bound for A− , or in other words, − inf(A) = sup(A− ),
as desired.

Solution 3.18: [Note that each of these exercises has many, many possible
solutions. And yes, it is a very silly question.]
√ √
(1) Take S = {x ∈ R | x > 2}, so that inf(S) = 2, which is irrational,
and S is also bounded below by 0, which is rational. (In fact, any set of
real numbers that is bounded below has both infinitely many rational
lower bounds and infinitely many irrational lower bounds.)
(2) Take S = (0, ∞), so that inf(S) = 0, which is rational, and S is bounded
below by −1, which is also rational.
(3) Take S = (2, 4), so that inf(S) = 2, which is rational, and S is also
bounded below by −π, which is irrational.
√ √
(4) Take S = ( 3, ∞), so that
√ inf(S) = 3, which is irrational, and S is
also bounded below by 2, which is also irrational.
102 PERIŠIĆ

Solutions to Exercises, Chapter 4


Solution 4.20: We know that 2 = 1.414213562... is irrational. Look at the
sequence a0 = 1, a1 = 1.4, a2 = 1.41, a3 = 1.414, a4 = 1.4142 etc. Each term
in the sequence lies in Q. The sequence is bounded above (by 1.5 for example)
and is increasing. However
√ it does not tend to a limit in Q. (It does tend to a
limit in R, namely 2, but this does not lie in Q.)

Solution 4.35:
3(1)−1 2
• u1 = 4(1)−5 = −1 = −2;
3(5)−1 14
• u5 = 4(5)−5 = 15 ≈ 0.9333;
3(10)−1 29
• u10 = 4(10)−5 = 35 ≈ 0.8286;
3(100)−1 299
• u100 = 4(100)−5 = 395 ≈ .7570;
3(1000)−1 2999
• u1000 = 4(1000)−5 = 3995 ≈ 0.7507;
3(10000)−1 29999
• u10000 = 4(10000)−5 = 39995 ≈ 0.7501;
3(100000)−1 299999
• u100000 = 4(100000)−5 = 399995 ≈ 0.7500;

So, it seems that a reasonable guess would be that L = limn→∞ un exists and
equals 0.75 = 43 . To verify this, we use the definition: we need to show that for
any choice of ε > 0, we can find n0 so that |un − L| < ε for all n > n0 .

Calculating, we see that



3n − 1 3 4(3n − 1) − 3(4n − 5) 11 11
|un −L| =
− =
= 4(4n − 5) = 4(4n − 5) .

4n − 5 4 4(4n − 5)
(The last equality follows since un − L is positive for n > 1.)

To find the value of n0 so that |un − L| < ε for n > n0 , we start by solving for
11
n: since 4(4n−5) < ε, we have that 11 11 5
4ε < 4n − 5, and so 16ε + 4 < n. That is,
11 5 11+20ε
for a specified value of ε, we can take n0 = b 16ε + 4 c = b 16ε c. Then, for
any choice of ε > 0, we set n0 = b 11+20ε
16ε c, and then if we take n > n0 , working
backwards we have that |un − L| < ε.

n
1+2·10 2
Solution 4.37: Set an = 5+3·10 n and L = 3 . For each choice of ε > 0, we

need to show that there exists n0 so that |an − L| < ε for all n > n0 .

Calculating, we see that


1 + 2 · 10n 2 3 + 6 · 10n − (10 + 6 · 10n )

7
|an − L| = − = = .
5 + 3 · 10n 3 n
3(5 + 3 · 10 ) n
15 + 9 · 10
MATH2039 ANALYSIS 103


7
Hence, for a given value of ε > 0, we want to find n0 so that 15+9·10n < ε for

7
n > n0 . So, we solve for n in terms of ε. First, note that 15+9·10n > 0 for all
7
positive integers n. So, we need only solve 15+9·10n < ε for n.

So, 7ε < 15+9·10n , and so −15+ 7ε < 9·10n , and so − 15 7 n


9 + 9ε < 10 . Performing
−15ε+7
a final bit of simplification, we get 9ε < 10n . If the numerator is positive,
7
that is if ε < 15 , we can solve for n by taking log10 of both sides. If on the
other hand the numerator is negative, then any positive integer will do. So,
set
7

1 if ε ≥ 15 ;
n0 = −15ε+7

blog10 9ε c otherwise

To get a specific value of n0 so that |an − L| < 10−3 for n > M , we substitute
−3
ε = 10−3 into the above equation to get that n > log10 −15·109·10−3
+7
≈ 2.8899.
So, we can take n0 = 2.

1
Solution 4.38: We start with the first part of the inequality, that n+1 <
n+1 x+1 1
 
ln(n + 1) − ln(n) = ln n . Set f (x) = ln x − x+1 and bn = f (n). We
want to show that f (x) > 0 for all x ≥ 1. Calculating, we see that f 0 (x) =
1
− x(x+1) 2 < 0 for all x > 0. This implies that f (x) is decreasing, and hence

that bn is a monotonically decreasing sequence. Since limn→∞ bn = 0, this


yields that bn > 0 for all n. (Because, if some bM < 0, then since bn is a
monotonically decreasing sequence, we would have that bn ≤ bM < 0 for all
n ≥ M , and so lim n→∞1bn would then be negative.) Since bn > 0 for all n, we
have that ln n+1
n > n+1 for all n, as desired.

To handle the other part of the inequality, consider cn = n1 − ln n+1



n and set
1 x+1 0 1
g(x) = x − ln x , so that cn = g(n). Since g (x) = − x2 (x+1) for all x > 0,
we see that cn is monotonically decreasing. Again, since limn→∞ cn = 0, we see
that cn > 0 for all n, and hence that n1 > ln n+1
n for all n, as desired.

It remains to show that an is bounded below and monotonically decreasing.


Since
P 
n+1 1 Pn 1 
an+1 − an = i=1 i − ln(n + 1) − i=1 i + ln(n)
= n+1 − ln(n + 1) + ln(n) = n+1 − ln n+1
1 1

n ,
we see that an+1 − an < 0 by the first part of the inequality. That is, an is
monotonically decreasing.

1 n+1

Since n > ln n = ln(n + 1) − ln(n) for all n, we have that
n
! n
!
X 1 X
an = − ln(n) > ln(i + 1) − ln(i) − ln(n) = ln(n + 1) − ln(n) > 0
i=1
i i=1

and so an is bounded below.


104 PERIŠIĆ

Since an is bounded above (an < a1 for all n, since it is a monotonically decreas-
ing sequence) and bounded below, it is bounded. Since it is also monotonic, we
have that an converges.

Solution 4.39: (This is an exercise in writing out the definition of the conver-
gence or divergence of a sequence for a triple of specific examples. Note that
we are not asked to determine whether the given statements are true or false,
or to prove them if they are true, but just to write them down.)
• for every ε > 0, there exists n0 so that 32n−1 > ε for all n > n0 .
• for every ε > 0, there exists n0 so that 1 − 2n < −ε for all n > n0 .
• for every ε > 0, there exists n0 so that |e−n − 0| < ε for all n > n0 .

Solution 4.51: [Please note that some solutions require knowledge from Cal-
culus, which we haven’t proved yet. So, please use with caution.]
(1) converges: whenever we are evaluating a limit in which the variable
(in this case n) appears in both the base and the exponent, we follow
the same basic procedure. First use the identity x = exp(ln(x)) to
rewrite the term. Here,
 
1/n ln(n + 2)
an = (n + 2) = exp .
n
Next, we check to see whether we are dealing with an indeterminate
form. Since the limit limn→∞ ln(n+2)
n has the indeterminate form ∞
∞,
we may use l’Hopital’s rule to evaluate
ln(n + 2) 1
lim = lim = 0.
n→∞ n n→∞ n + 2

Hence, an converges to e0 = 1.
(2) converges: there is a standard way of evaluating the limit as n → ∞
of a rational function in n (where a rational function is the quotient
of two polynomials). First, locate the highest power of n that appears
in either the numerator or the denominator, and then multiply both
numerator and denominator by its reciprocal. Here, the highest power
of n that appears is n3 , and so we calculate
1 1
n2 + 3n + 2 n2 + 3n + 2 n3 n + n32 + 2
n3
an = = · 1 = .
6n3 + 5 6n3 + 5 n3 6 + n53
We then use several properties of limits: that the limit of a quotient
is the quotient of the limits, that the limit of a sum is the sum of the
limits, and that limn→∞ n1 = 0. Here,
1
n + n32 + 2
n3 0
lim an = lim = = 0.
n→∞ n→∞ 6 + n53 6
Hence, an converges to 0.
MATH2039 ANALYSIS 105

(3) converges: as above, we first rewrite the term using x = exp(ln(x)).


Here,
n !
ln 1 + n1
   
1 1
an = 1 + = exp n ln 1 + = exp 1 .
n n n

We then concentrate on the exponent and check to see whether we are


dealing with an indeterminate form, which in this case we are, since
both limn→∞ ln(1 + n1 ) and limn→∞ n1 are equal to 0. Hence, we may
apply l’Hopital’s rule to evaluate
ln 1 + n1

1
lim 1 = lim = 1.
n→∞ n→∞ 1 + 1
n n

Hence, an converges to e1 = e.
(4) converges: here we use the squeeze law. Since −1 ≤ sin(n) ≤ 1 for
all n, we have that − 31n ≤ sin(n)
3n ≤ 31n . Since limn→∞ 31n = 0, we have
1
that limn→∞ − 3n = 0 as well, and so an converges to 0.
(5) diverges: write
√ √
√ √ 2n + 3 + n + 1 n+2
an = ( 2n + 3 − n + 1) · √ √ =√ √ .
2n + 3 + n + 1 2n + 3 + n + 1
We now massage algebraically, in order to simplify:
n + 32 + 12 n + 32
r
n+2 n+2 1 3
√ √ ≥ √ = q > q = √ n+ .
2n + 3 + n + 1 2 2n + 3 2 2(n + 3 ) 2 2(n + 3 ) 2 2 2
2 2
q
3
Since limn→∞ n+ 2 = ∞, we see that limn→∞ an = ∞, and so an
diverges.
8kπ

(6) diverges: for
 n = 8k, a8k = cos 4 = 1, while for n = 8k + 1,
(8k+1)π
a8k+1 = cos 4 = √2 . In particular, |a8k − a8k+1 | = √12 , and so
1

the sequence fails the Cauchy criterion, and so diverges.


 
1 1/n
 ln(1+ n
1
)
(7) converges: write an = 1 + n = exp n . Since

1
lim ln(1 + ) = 0,
n→∞ n
1
ln(1+ n )
we have that limn→∞ n = 0 (by the squeeze law for instance,
1
ln(1+ n ) 1
since 0 ≤ n ≤ ln(1 + n ) for n ≥ 1). Hence,
 1 
ln(1 + n)
lim exp = e0 = 1,
n→∞ n
and so an converges to 1.
(8) diverges: given ε > 0, we show that there exists M so that an > ε
for n > M . Since an = ln(n), this becomes ln(n) > ε for n > M .
Exponentiating both sides of ln(n) > ε, we get that n > eε (and vice
106 PERIŠIĆ

versa, that if n > eε , then ln(n) > ε, since ex is an increasing function),


and so we can take M = eε .
(9) diverges: very similar to the question just done. Given ε > 0, we
show that there exists M so that an > ε for n > M . Taking logs of
both sides of an = en > ε, we get that n > ln(ε). So, we make take
M = ln(ε).

(10) converges: since limn→∞ an has the indeterminate form ∞ (as both

ln(n) → ∞ and n → ∞ as n → ∞), we may apply l’Hopital’s rule to
see that
1
ln(n) 2
lim √ == lim n1 = lim √ = 0.
n→∞ n n→∞ √
2 n
n→∞ n
Hence, an converges to 0.
(11) converges: as always, we first rewrite each term as
n 2
!
ln 1 −
   
2 2 n2
an = 1 − 2 = exp n ln 1 − 2 = exp 1 .
n n n

As n → ∞, the exponent reveals itself to have the indeterminate form


0
0 , and so we may evaluate using l’Hopital’s rule:
1
ln 1 − n22

1− n22
· n43 − 1−4 2
n2
lim 1 = lim −1 = lim = 0.
n→∞
n
n→∞
n 2
n→∞ n
Hence, an converges to e0 = 1.
(12) diverges: we could use either l’Hopital’s rule (since the limit has the
indeterminate form ∞ ∞ ) or the standard trick for dealing with limits of
rational functions (multiply numerator and denominator by the recip-
rocal of the highest power of n appearing anywhere in the term), but
instead we massage algebraically:
n3 n3 n
an = > = .
10n2 + 1 10n2 + 10n2 20
n
Since 20 diverges, it follows that an diverges as well.
(13) converges: it is a reasonable guess that an = xn converges to 0,
which by definition means that given ε > 0, there exists M so that
|xn − 0| = |xn | < ε for n > M . For x = 0, this is true, since xn
becomes the constant sequence an = 0. So, we can assume that x 6= 0.
Taking ln of both sides of |xn | < ε and using that |xn | = |x|n , we
ln(ε)
get that n ln(|x|) < ln(ε), and so n > ln(|x|) . (The direction of the
inequality changes since |x| < 1 and so ln(|x|) < 0.) Hence, we may
ln(ε) ln(ε)
take M = ln(|x|) . [Then, if n > M = ln(|x|) , then n ln(|x|) < ln(ε), and
n
exponentiating we get that |x| < ε, as desired.)
(14) converges: recall that np ≥ n and that n → ∞ as n → ∞, and so
np → ∞ as n → ∞. Hence, n1p converges to 0, and therefore an = ncp
converges to c · 0 = 0.
MATH2039 ANALYSIS 107

(15) converges: using the standard trick for rational functions, write
1
2n 2n n 2
an = = · 1 = 3 .
5n − 3 5n − 3 n 5− n
1
As n → ∞, n → 0 and so an converges to 25 .
(16) converges: using the standard trick for rational functions, write
1 1
1 − n2 1 − n2 n2 n2 −1
an = = · 1 = 2 .
2 + 3n2 2 + 3n2 n2 n2 +3
1
As n → ∞, n2 → 0 and so an converges to − 13 .
(17) converges: using the standard trick for rational functions, write
1
n3 − n + 7 n3 − n + 7 n3 1 − n12 + 7
n3
an = = · 1 = .
2n3 + n2 2n3 + n2 n3 2 + n1
1 1
As n → ∞, both n2 → 0 and n → 0, and so an converges to 21 .
9 n
(18) converges: by a previous part of this exercise, we know that ( 10 )
9 9 n
converges to 0, since | 10 | < 1, and so limn→∞ (1 + ( 10 ) ) = 1 +
9 n
limn→∞ ( 10 ) = 1.
(19) converges: by a previous part of this exercise, we know that (− 21 )n
converges to 0, since | − 12 | < 1, and so limn→∞ (2 − (− 21 )n ) = 2 −
limn→∞ (− 12 )n = 2.
(20) diverges: for n even, an = 2, while for n odd, an = 0. In particular,
|an − an+1 | = 2 for all n, and so the sequence fails the Cauchy criterion
and hence diverges.
(21) converges: note that 0 ≤ 1 + (−1)n ≤ 2 for all n, and so the squeeze
law yields that since limn→∞ n2 = 0, we have that limn→∞ an = 0.
(22) converges: we begin by noting that
√ √
1 + (−1)n n 2 n
0≤ ≤ ,
( 32 )n ( 32 )n

2 n
and so we’ll concentrate on evaluating limn→∞ ( 32 )n
and hope to be able

2 n
to apply the squeeze law. Since limn→∞ ( 3 )n has the indeterminate
2
form ∞
∞ , we may use l’Hopital’s rule to evaluate
√ √1
2 n n 1
lim 3 n = lim = lim √ =0
n→∞ ( ) n→∞ ln( 3 ) exp(n ln( 3 )) n→∞ ln( 3 ) n( 3 )n
2 2 2 2 2

(where we differentiate ( 23 )n by first writing it as exp(n ln( 32 ))). Hence,


we may use the squeeze law to see that an converges to 0.
(23) converges: since 0 ≤ sin2 (n) ≤ 1 for all n and since √1n → 0 as n → ∞
√ 2
(since n → ∞ as n → ∞), the squeeze rule yields that sin√n(n) → 0 as
n → ∞. That is, an converges to 0.
108 PERIŠIĆ

p √
(24) converges: since 1 ≤ 2 + cos(n) ≤ 3 for all n and since n1 → 0 as
q
n → ∞, the squeeze law yields that 2+cos(n)
n → 0 as n → ∞. That
is, an converges to 0.
(25) converges: since sin(πn) = 0 for all integers n, this sequence is the
constant sequence an = n · 0 = 0 for all n. In particular, an converges
to 0.
(26) diverges: since cos(πn) = (−1)n , this sequence can be rewritten as
an = (−1)n n. For n ≥ 1, |an+1 − an | ≥ 2, and so the sequences fails
the Cauchy criterion, and so diverges.
(27) converges: since −1 ≤ − sin(n) ≤ 1 for all n, we have that − n1 ≤
− sin(n)
n ≤ n1 for all n, and so − sin(n)
n converges to 0. Hence, an converges
0
to π = 1.
(28) diverges: for n even, cos(πn) = 1 and for n odd, cos(πn) = −1. In
particular, |an+1 − an | = |21 − 2−1 | = 32 for all n, and so this sequences
fails the Cauchy criterion, and hence an diverges.
(29) converges: we could use l’Hopital’s rule, since limn→∞ ln(2n)
ln(3n) has the
indeterminate form ∞ ∞ , but we proceed in a more low tech way. Use
the laws of logarithms and a variant of the standard trick for rational
functions, we rewrite
1 ln(2)
ln(2n) ln(2) + ln(n) ln(2) + ln(n) ln(n) 1+ ln(n)
an = = = · 1 = ln(3)
.
ln(3n) ln(3) + ln(n) ln(3) + ln(n) ln(n) 1+ ln(n)
ln(2) ln(3)
Since ln(n) → ∞ as n → ∞, we have that both ln(n) and ln(n) go to 0
as n → ∞, and so limn→∞ an = 1.
ln2 (n) ∞
(30) converges: since limn→∞ n has the indeterminate form ∞, we can
use l’Hopital’s rule:
ln2 (n) 2 ln(n) n1 2 ln(n)
lim = lim = lim .
n→∞ n n→∞ 1 n→∞ n
This limit still has the indeterminate form ∞∞ , and we can apply l’Hopital’s
rule again to get
2
2 ln(n)
lim = lim n = 0.
n→∞ n n→∞ 1
Hence, an converges to 0.
(31) converges: write
sin( n1 )
 
1
an = n sin = 1 .
n n
Since limn→∞ an has the indeterminate form 00 , we can apply l’Hopital’s
rule to get
sin n1 cos n1 − n12
    
1
lim 1 = lim 1 = lim cos = cos(0) = 1.
n→∞
n
n→∞ − n2 n→∞ n
MATH2039 ANALYSIS 109

Hence, an converges to 1. (There is also a geometric argument for


evaluating this limit, that can be found in Adams (p. 116, Theorem
7).)
(32) converges: as n → ∞, arctan(n) → π2 , and so limn→∞ arctan(n)n =
0. (This is an application of the squeeze law, since the numerator is
bounded by 0 and π.)
n3 ∞
(33) converges: since limn→∞ en/10
has the indeterminate form ∞, we may
use l’Hopital’s rule:
n3 3n2
lim = lim .
n→∞ en/10 n→∞ 1 en/10
10

Since this latter limit still has the indeterminate form ∞ , we use l’Hopital’s
rule again:
3n2 6n
lim 1 n/10 = lim 1 n/10 .
n→∞
10 e n→∞
100 e
And as we still have the indeterminate form ∞ ∞ , we apply l’Hopital’s
rule yet again:
6n 6
lim 1 n/10 = lim 1 n/10 .
100 e 1000 e
n→∞ n→∞

The right hand limit evaluates to 0, and so an converges to 0.


(34) converges: write
 n  n
2n + 1 2n 1 2n 1n 2 1
an = n
= n + n = n + n = + .
e e e e e e e
Since both 2e < 1 and 1e < 1, we have that both ( 2e )n and ( 1e )n go to 0
as n → ∞, and so their sum goes to 0 as n → ∞. That is, an converges
to 0.
(35) converges: again there are several possible approaches, including l’Hopital’s
rule, but again we take a low tech approach, and begin by expressing
sinh(n) and cosh(n) in terms of en and e−n , to get
sinh(n) en − e−n en − e−n e−n 1 − e−2n
an = = n −n
= n −n
· −n = .
cosh(n) e +e e +e e 1 + e−2n
Since e−2n = ( e12 )n → 0 as n → ∞, we see that limn→∞ an = 1. That
is, an converges to 1.
(36) converges: as with all limits in which the variable appears in both the
base and the exponent, we begin by rewriting
 using
 the identity m =
exp(ln(m)) to get an = (2n + 5)1/n = exp ln(2n+5)
n . We may now use
ln(2n+5)
l’Hopital’s rule to evaluate the limit of the exponent limn→∞ n
(as it has the indeterminate form ∞ ∞ ) to get
2
ln(2n + 5) 2n+5
lim = lim = 0.
n→∞ n n→∞ 1
Therefore, an converges to e0 = 1.
110 PERIŠIĆ

(37) converges: as with all limits in which the variable appears in both
the base and the exponent, we begin by rewriting using the identity
m = exp(ln(m)) to get
 n  n  n   
n−1 n+1−2 2 2
an = = = 1− = exp n ln 1 − .
n+1 n+1 n+1 n+1
Since the exponent has the indeterminate form 0 · ∞ as n → ∞, we
rewrite it as
2
ln(1 − n+1 )
 
2
n ln 1 − = 1 ,
n+1 n

which as the indeterminate form 00 as n → ∞. We now apply l’Hopital’s


rule to evaluate
 
2 1 2
ln 1 − n+1 2
1− n+1
· (n+1) 2
−2n2
lim 1 = lim 1 = lim   = −2.
n→∞
n
n→∞ − n2 n→∞
1 − 2 · (n + 1)2
n+1

−2
Hence, an converges to e .
(38) converges: since − n1 → 0 as n → ∞, we see that an converges to
(0.001)0 = 1.
n+1 1
(39) converges: as n → ∞, n = 1+ n → 1, and so an converges to
21 = 2.
(40) converges: one way to evaluate this limit is to write an = ( n2 )3/n =
23/n
n3/n
and to evaluate the limits of the numerator and denominator sep-
arately. To evaluate limn→∞ 23/n , all we need note is that limn→∞ n3 =
0, and so 23/n converges to 20 = 1.

To evaluate limn→∞ n3/n , we rewrite n3/n as n3/n = exp(ln(n) n3 ) and


use l’Hopital’s rule to evaluate limn→∞ 3 ln(n)
n (since it has the indeter-
minate form ∞ ∞ ). Using l’Hopital’s rule, we get that
3
3 ln(n)
lim = lim n = 0,
n→∞ n n→∞ 1

and so n3/n converges to e0 = 1. Therefore,


23/n limn→∞ 23/n 1
lim = = = 1.
n→∞ n3/n limn→∞ n3/n 1

(41) diverges: begin by ignoring the (−1)n and worrying about what hap-
pens to the rest of the term. Using the standard trick, massage to get
2 2
(n2 + 1)1/n = exp( ln(nn+1) ). Since limn→∞ ln(nn+1) has the indetermi-
nate form ∞∞ , we may use l’Hopital’s rule to evaluate
2n
ln(n2 + 1) n2 +1
lim = lim = 0,
n→∞ n n→∞ 1
MATH2039 ANALYSIS 111

and so
ln(n2 + 1)
 
lim exp = e0 = 1.
n→∞ n
So, putting the (−1)n back into the picture, we see that an fails the
n2 +1
Cauchy criterion: specifically,
2 since
n converges to 1, for any ε > 0,
there exists M so that n − 1 < ε for n > M . Choose ε = 21 , and
n +1

note that for n > M , we get that |an − an+1 | > 1, since one of an ,
an+1 is within 12 of 1 and the other is within 12 of −1 (remember the
alternating signs). So, an diverges.
(42) converges: we perform a bit of algebraic massage: note that
2 n 2 n  n
 
3 3 20
an = 1 n 9 n
 < 9 n =
 .
2 + 10 10
27
n
Since 20
27 → 0 as n → ∞ (since 20
27 < 1), the squeeze rule yields that
an converges to 0 as well.

Solution 4.52: Suppose that qn converges and set x = limn→∞ qn . Now, note
that
an an−1 + an−2 an−2 1
qn = = =1+ =1+ .
an−1 an−1 an−1 qn−1
Hence,
 
1 1 1
x = lim qn = lim 1 + =1+ =1+ ,
n→∞ n→∞ qn−1 limn→∞ qn−1 x
since limn→∞ qn−1 = x as well. Therefore, x = 1 + x1 , and so (multiplying
2
√  x − x − 1 = 0.
through by x and simplifying) x satisfies the quadratic equation
1
By the quadratic formula, this yields that x = 2 1 ± 5 . However, since
√ 
qn ≥ 0 for all n, it must be that x ≥ 0 as well, and so x = 21 1 + 5 .

Solution 4.54: In all three of these statements, we start with the same piece of
information, namely that limn→∞ xn = −4. That is, for each ε > 0, there exists
M (which depends on ε) so that |xn − (−4)| = |xn + 4| < ε for n > M .
p
(1) we need to show that limn→∞ |xn | = 2, which is phrased mathemat-
ically
p as needing to show that for each µ > 0, therepexists P so that
| |xn | − 2| < µ for n > P . We start by rewriting | |xn | − 2|, using
the standard trick for handling differences of square roots, namely
p
p p | |xn | + 2| | |xn | − 4| | |xn | − 4|
| |xn | − 2| = | |xn | − 2| · p = p ≤ .
| |xn | + 2| | |xn | + 2| 2
p
(The last inequality follows from the fact that | |xn | + 2| ≥ 2 for all
possible values of xn .) Since for any µ > 0, there exists M so that
| |xn | − 4| < 2µ (by using the definition of limn→∞ |xn | = 4) for n > M ,
we have that
p | |xn | − 4| 2µ
| |xn | − 2| ≤ < =µ
2 2
for n > M , and so we are done.
112 PERIŠIĆ

(2) we need to show that limn→∞ x2n = 16, which is phrased mathematically
as needing to show that for each µ > 0, there exists P so that |x2n −16| <
µ for n > P . We start by rewriting |x2n − 16|, using that it is the
difference of two squares:
|x2n − 16| = |(xn − 4)(xn + 4)| = |xn − 4| |xn + 4|.
Now apply the definition of limn→∞ xn = −4 with ε = 1, so that there
exists M so that if n > M , then |xn −(−4)| < 1. In particular, if n > M ,
then −5 < xn < −3, and so |xn | < 5, and so |xn − 4| ≤ |xn | + 4 < 9.

Since xn → −4 by assumption, we know that for any ε > 0, there is


Q so that |xn − (−4)| = |xn + 4| < 19 ε for n > Q. Hence, if n > P =
max(M, Q), then
1
|x2n − 16| = |xn − 4| |xn + 4| < 9 ε = ε,
9
as desired.
(3) we need to show that limn→∞ x3n = − 34 , which is phrased mathemat-
ically as needing to show that for each µ > 0, there exists P so that
| x3n −(− 34 )| = | x3n + 43 | < µ for n > P . Note that | x3n −(− 43 )| = | x3n + 43 =
1
3 |xn +4|. We know from the definition of limn→∞ xn = −4 given above
that for any µ > 0, there exists M so that |xn − (−4)| = |xn + 4| < 3µ
for n > M . Hence, for n > M , we have that 31 |xn + 4| < 13 3µ = µ for
n > M , and so we are done.

Solution 4.55:
(1) since a > 0, we can apply the definition of limn→∞ an = a with ε = 12 a
to see that there exists P so that an > 0 for n > P (since the interval
of radius 12 a centred at a contains only positive numbers), and so for

n > P , an makes sense.
√ √
We need to get our hands on | an − a|, which we do with our usual
trick for handling differences of square roots:
√ √
√ √ √ √ | an + a| |a − a|
| an − a| = | an − a| √ √ =√ n √ .
| an + a| an + a
√ √ √
(Here we’re using
√ √ that both an√> 0 and
√ a > 0 to say that | an +
√ √
a| = an + a.) Since an + a > a for n > P , we have that
√ √ |an − a| |a − a|
| an − a| = √ √ < n√
an + a a
for n > P . Since an converges
√ to a, for every ε > 0, we can choose
M > P so that |an − a| < ε a for n > M . For this choice of M , we
have that

√ √ |an − a| |a − a| ε a
| an − a| = √ √ < n√ < √ = ε,
an + a a a
√ √
and so an converges to a.
MATH2039 ANALYSIS 113

(2) this one, we break into three cases. If a > 0, then (applying the def-
inition of limn→∞ an = a with ε = a) there exists M0 so that an > 0
for n > M0 . In this case, we have |an | = an for n > M0 and |a| = a,
and so ||an | − |a|| = |an − a|. Since there is M1 so that |an − a| < ε for
n > M1 , we have that ||an | − |a|| < ε for n > M = max(M0 , M1 ), and
so limn→∞ |an | = |a|.

If a < 0, then (applying the definition of limn→∞ an = a with ε = |a|)


there exists M0 so that an < 0 for n > M0 . In this case, we have
|an | = −an for n > M0 and |a| = −a, and so ||an | − |a|| = | − an + a| =
|an −a|. Since there is M1 so that |an −a| < ε for n > M1 , we have that
||an | − |a|| < ε for n > M = max(M0 , M1 ), and so limn→∞ |an | = |a|.

If a = 0, then the definition of limn→∞ an = a becomes: for every


ε > 0, there exists M so that |an − 0| = |an | < ε for n > M . Since
| |an | | = |an |, we have that the definition of limn→∞ |an | = 0 is satisfied
without any further work.
(3) if a 6= 0, consider the definition of limn→∞ an = a with ε = 21 |a|: there
exists M so that |an − a| < 12 |a| for n > M . That is, an lies in the
interval centred at a with radius 12 |a|, and so |an | > 21 |a|.

Now consider the sequence (−1)n an . For n > M and n even, (−1)n an =
an lies in the interval centred at a with radius 12 |a|. For n > M and
n odd, (−1)n an = −an lies in the interval centred at −a with radius
1
2 |a|. In particular, we have, regardless of whether n is odd or even,
that |(−1)n an − (−1)n+1 an+1 | > |a| for n > M , since (−1)n an and
(−1)n+1 an+1 lie on opposite sides of 0 and are both distance at least
1 n
2 |a| from the origin. Hence, (−1) an violates the Cauchy criterion (see
Theorem 4.25), and so diverges.

Solution 4.56: Since limn→∞ xn = x, we have that for each ε > 0, there exists
M so that |xn − x| < 13 ε for n > M . For any m > 0 and n > M , we now have
that
|xn+1 + · · · + xn+m − mx| = |xn+1 − x + · · · + xn+m − x|
≤ |xn+1 − x| + · · · + |xn+m − x|
1
≤ m ε.
3
Dividing by n + m, we obtain that

1 m m 1 1
n + m (xn+1 + · · · + xn+m ) − n + m x ≤ n + m 3 ε < 3 ε

m
(since n+m < 1). Viewing n as fixed for the moment, choose m so that both
| n+m x − x| < 13 ε (which we can do since limm→∞ n+m
m m
= 1 for n fixed) and
1 1
n+m |x 1 + x 2 + · · · + x n | < 3 ε (which we can do since x 1 + x2 + · · · + xn is a
114 PERIŠIĆ

constant when n is fixed). Then,



1
n + m 1
(x + · · · + x n+m ) − x


1 1 m m
= (x1 + · · · + xn ) + (xn+1 + · · · + xn+m ) − x+ x − x
n+m n+m n+m n+m

1 1 m m
≤ (x1 + · · · + xn ) + (xn+1 + · · · + xn+m ) − x + x − x
n+m n+m n+m n+m
1 1 1
≤ ε+ ε+ ε=ε
3 3 3

for all m > 0. Since this is true for all n > M and all m > 0, we have that
1
p (x1 + · · · + xp ) − x < ε for all p > M , as desired.

MATH2039 ANALYSIS 115

Solutions to Exercises, Chapter 5

Solution 5.4:
(1) an = (−1)n , bounded above by 1 and bounded below by −1, hence
bounded. This sequence fails the Cauchy criterion, since |an −an+1 | = 2
for all n, and so diverges.
(2) sin(n), bounded above by 1 and bounded below by −1, hence bounded.
Though it seems fairly clear why this sequence diverges, the actual
proof is a bit subtle, and we do not give it here. If you are intrigued,
ask me after class, or come to my office hours.
(3) 0, 1, 0, 0, 1, 0, 0, 0, 1, 0, 0, 0, 0, 1, . . ., bounded above by 1 and bounded
below by 0, hence bounded. Arbitrarily far out in the sequence, there
are consecutive terms taking the values 0 and 1, so the sequence fails
the Cauchy criterion and hence diverges.
(4) an = the nth digit of π, bounded above by 9 and bounded below by
0, hence bounded. Does not converge, because the only way for a
sequence of integers to converge is for it to be eventually constant,
that is, constant past some index, which in this case would then imply
that π is a repeating decimal, hence a rational number, which it isn’t.
(In fact, fixing an irrational number x and taking an to be the nth digit
of the decimal expansion of x gives a sequence that is bounded but not
convergent, by the same argument.)
(5) an = the nth digit of the rational number 17 = .142857, using the same
argument as above (which works for rational numbers, as long as the
length of the repeating section in the decimal expansion is longer than
one digit).
116 PERIŠIĆ

Solutions to Exercises, Chapter 6

Solution 6.7:
(1) To show that hn (x) is continuous at a ∈ R, we need to show that for
each ε > 0, there exists δ > 0 so that if |x−a| < δ, then |hn (x)−hn (a)| <
ε. Since hn (x) = xn , this is the same as showing that for each ε > 0,
there exists δ > 0 so that if |x − a| < δ, then |xn − an | < ε. Let’s break
the proof into cases.

If n = 1, then all we need to do to satisfy the definition is take δ = ε.


So, we can assume that n ≥ 2. If in addition we have that a = 0, then
by the definition of limit, we need to show that for each ε > 0, there is
δ > 0 so that if |x| < δ, then |xn | = |x|n < ε. So, taking δ = ε1/n , we
are done in this case as well.

Consider now the case that n ≥ 2 and a > 0, and factor |xn − an | to get
|xn − an | = |(x − a)(xn−1 + axn−2 + · · · + an−2 x + an−1 )|. Recall that
we have a great deal of choice in how we choose δ, so we may restrict
our attention to the interval |x − a| < 21 a, so that 12 a < x < 32 a, by
requiring that δ < 21 a (which makes sense, since a > 0). Calculating,
we see that
|xn − an | = |(x − a)(xn−1 + axn−2 + · · · + an−2 x + an−1 )|
≤ |x − a|(xn−1 + axn−2 + · · · + an−2 x + an−1 )
 n−1  n−2 !
3 3 n−2 3 n−1
< |x − a| a +a a + ··· + a a+a
2 2 2
n−1
X  3 k
= |x − a|an−1
2
k=0
1 − (3/2)n
= |x − a|an−1 = C|x − a|,
1 − (3/2)
n
where C = an−1 1−(3/2)
1−(3/2) > 0 depends on both a > 0 and n ≥ 2. So,
take δ to be the smaller of C1 ε and 12 a. Then, for |x − a| < δ, we have
that |xn − an | < C|x − a| ≤ ε as desired. (The first inequality follows
from the calculation above and the fact that |x − a| < δ < 12 a, while
the second inequality follows from δ < C1 ε.)

A similar argument, with appropriate placements of absolute values,


holds for a < 0. (Note that for a given ε > 0, the choice of δ depends
on ε, on a, and on n.)
(2) To show that g(x) is continuous at a ∈ R, we need to show that for each
ε > 0, there exists δ > 0 so that if |x − a| < δ, then |g(x) − g(a)| < ε.
Since g(x) = c for all x, this is the same as showing that for each ε > 0,
there exists δ > 0 so that if |x − a| < δ, then |c − c| = 0 < ε. So,
MATH2039 ANALYSIS 117

regardless of the value of ε, taking δ = 1 (or whatever your favorite


positive number happens to be today) satisfies the definition.
(3) :( Sorry

Solution 6.11: First, for the sake of notational clarity, define the n-fold compo-
sition of f with itself by f ◦n , so that f ◦n = f ◦f ◦(n−1) . The hypothesis can then
be restated as saying that the sequence {f ◦n (c)} converges to a. Now, apply f
to both sides. Since f is continuous, the sequence {f (f ◦n (c))} converges to f (a),
by Proposition 6.10. However, since f (f ◦n (c)) = f ◦ f ◦n (c) = f ◦(n+1) (c), the
sequence {f (f ◦n (c))} is the same as the sequence {f ◦n (c)} with the first term
removed, and so {f (f ◦n (c))} converges to a as well. Hence, since {f (f ◦n (c))}
converges to both a and f (a), we have that a = f (a).

Solution 6.18: First, since limx→∞ (f (x + 1) − f (x)) = 0, for any ε > 0, there
exists x0 (which we can take to be positive) so that |f (x + 1) − f (x)| < 21 ε for
x > x0 . Now, using the maximum value property, Theorem 6.17, there exists a
maximum value M of |f (x)| on the interval [x0 , x0 + 1].

The first claim is that for any k ≥ 0, we have that |f (x)| ≤ 12 kε + M for x in
the interval [x0 + k, x0 + k + 1]. To see this, let K be the maximum value of
|f (x)| on [x0 + k, x0 + k + 1], occurring at y. Then, x0 + k ≤ y ≤ x0 + k + 1, and
so x0 ≤ y − k ≤ x0 + 1. We now engage in some algebraic manipulation:
|f (y)| = |f (y) − f (y − k) + f (y − k)|
≤ |f (y) − f (y − k)| + |f (y − k)|
≤ |f (y) ± f (y − 1) ± · · · ± f (y − k + 1) − f (y − k)| + |f (y − k)|
1 1 1
≤ ε + ε + ··· + ε + M
2 2 2
k
≤ ε + M.
2

In particular, this tells us that


k k k k
|f (y)| 2ε +M ε M ε M ε M 1 M
≤ ≤ 2 + ≤ 2 + < 2 + < ε+
y y y y x0 + k y k y 2 y
for all y in the interval [x0 + k, x0 + k + 1].

Now, choose x1 > x0 so that xM1 < 12 ε. Then, for all y > x1 , we have that

f (y) |f (y)| 1 M 1 1
y = y < 2ε + y < 2ε + 2ε

f (x)
for all y > x1 . In particular, we have that the definition of limx→∞ x = 0 is
satisfied, as desired.

Solution 6.19: Since f is continuous on [a, b], so is g(x) = −f (x). Since g is


continuous on the closed interval [a, b], the maximum value property applied to
g yields that there exists some x0 in [a, b] so that g(x0 ) ≥ g(x) for all x in [a, b].
118 PERIŠIĆ

Hence, −f (x0 ) ≥ −f (x) for all x in [a, b], and so f (x0 ) ≤ f (x) for all x in [a, b].
That is, f satisfies the minimum value property.

Solution 6.22:
(1) Missing
(2) Again missing :(
(3) for f (x) = x1995 + 7654x123 + x on the closed interval [−a, a], start
by verifying continuity; actually, f is continuous on all of R being a
polynomial, and hence is continuous on [−a, a]. Now, check the sign of
f on the endpoints of the given interval: f (a) = a1995 +7654a123 +a > 0
(since a > 0) and f (−a) = (−a)1995 +7654(−a)123 +(−a) = −f (a) < 0,
and so the intermediate value property implies that there exists some c
in (−a, a) with f (c) = 0. (And actually, casual inspection reveals that
f (0) = 0.)
(4) for tan(x) = e−x for x in [−1, 1], start by defining g(x) = tan(x) − e−x ,
so that tan(c) = e−c if and only if g(c) = 0, as was done above. Note
that g is continuous on [−1, 1], since e−x is continuous on all of R
and tan(x) is continuous as long as its denominator cos(x) is non-zero,
which holds true on [−1, 1]. Since we are working on the closed interval
[−1, 1], check the values of g on the endpoints: g(1) = tan(1) − e−1 =
1.1895... > 0 and g(−1) = −4.2757... < 0, and so there exists some c in
(−1, 1) with g(c) = 0, and hence with tan(c) = e−c .
(5) as above, f (x) = x3 + 2x5 + (1 + x2 )−2 is continuous on [−1, 1], as it
is the sum of a polynomial and a rational function whose denominator
is non-zero on [−1, 1]. As always, check the endpoints of the interval
first: f (1) = 13 11
4 and f (−1) = − 4 , and so by the intermediate value
property, there is some c in (−1, 1) at which f (c) = 0.
(6) consider f (x) = 3 sin2 (x) − 2 cos3 (x). Since both sin(x) and cos(x) are
continuous on all of R, we have that f is continuous on all of R. Since no
specific closed interval is given, we need to find an appropriate interval
on which to apply the intermediate value property for f , if in fact such
an interval exists. Fortunately, we remember that sin(kπ) = 0 for all
integers k, and so we may consider the interval [kπ, (k + 1)π] for any
integer k ≥ 1, so that the interval lies in (0, ∞). At the endpoints of
this interval, f (kπ) = −2 cos3 (kπ) and f ((k + 1)π) = −2 cos3 ((k + 1)π).
Since cos(kπ) and cos((k+1)π) are equal to ±1 and have opposite signs,
f (kπ) and f ((k+1)π) are both non-zero and have opposite signs, and so
by the intermediate value property, there is a point ck in (kπ, (k + 1)π)
at which f (ck ) = 0, that is, at which 3 sin2 (ck ) = 2 cos3 (ck ), as desired.
(7) first, note that f (x) = 3 + x5 − 1001x2 is a polynomial and so is contin-
uous on all of R, and in particular is continuous for x > 0. As above,
we need to choose a closed interval on which to apply the intermedi-
ate value property. Let’s start by evaluating f at some of the natural
numbers: f (1) = −997; f (2) = −3969; f (10) = −90097; f (11) = 880.
MATH2039 ANALYSIS 119

Hence, the intermediate value property implies that there is a number


c in the open interval (10, 11) at which f (c) = 0.
120 PERIŠIĆ

Solutions to Exercises, Chapter 7

Solution 7.7: First, note that f is continuous on [1, 4], as it is the composi-
tion of two continuous functions, namely absolute value and a linear polyno-
mial. However, f is not differentiable at x = 2 (since absolute value is not
differentiable at 0), and so the hypotheses of the mean value theorem are not
satisfied.
To see that f does not satisfy the conclusion of the mean value theorem, we
calculate: f (4) − f (1) = |4 − 2| − |1 − 2| = 2 − 1 = 1 and 4 − 1 = 3. However,
for x > 2, we have that f 0 (x) = 1 and for x < 2 we have that f 0 (x) = −1, and
so there cannot be a point c in (1, 4) at which f 0 (c) = (f (4) − f (1))/(4 − 1) =
1/3.

Solution 7.9:
(1) This proof follows the same general outline as the proof just given.
Suppose that g 0 (x) = an−1 xn−1 +an−2 xn−2 +· · ·+a1 x+a0 , and consider
the new function h(x) = n1 an−1 xn + n−1
1
an−2 xn−1 +· · ·+ 12 a1 x2 +a0 x−
g(x). Note that since g and polynomials are differentiable, and hence
continuous, on all of R, we have that h is differentiable, and hence
continuous, on all of R. Also, h0 (x) = an−1 xn−1 + an−2 xn−2 + · · · +
a1 x + a0 − g 0 (x) = 0 for all x ∈ R.

For x0 > 0, apply the mean value theorem to h on the interval [0, x0 ].
Since h is continuous on [0, x0 ] and differentiable on (0, x0 ), the mean
value theorem yields that there exists some c in (0, x0 ) so that h(x0 ) −
h(0) = h0 (c)(x0 − 0) = 0, since h0 (c) = 0. That is, h(x0 ) = h(0) for
all x0 > 0. As above, we also get that h(x0 ) = h(0) for all x0 < 0 by
applying the mean value theorem to h on the interval [x0 , 0].

Hence, setting b = h(0), we have that h(x) = b for all x ∈ R. Substitut-


ing in the definition of h, this yields that n1 an−1 xn + n−1 1
an−2 xn−1 +
· · · + 2 a1 x2 + a0 x − g(x) = b for all x ∈ R, that is, g(x) = n1 an−1 xn +
1
1
n−1 an−2 x
n−1
+ · · · + 12 a1 x2 + a0 x − b for all x ∈ R, and so g is a
polynomial of degree n.
(2) This is a slightly different sort of argument, and we break it into two
pieces, corresponding to the two inequalities.

Set h(x) = x − ln(x + 1), and note that h is differentiable, and hence
continuous, on (−1, ∞). The two cases, of −1 < x < 0 and of x > 0, are
handled in the same fashion, and we write out the details only for the
case x > 0. Apply the mean value theorem to h on any closed interval
in [0, ∞). Note that h(0) = 0 − ln(1) = 0. If there were another point
x0 > 0 at which h(x0 ) = 0, then by applying either Rolle’s theorem or
the mean value theorem to h on the interval [0, x0 ], there would exist a
point c in (0, x0 ) at which h0 (c) = 0. However, h0 (c) = 1 − c+1
1
, which
is non-zero for c 6= 0. Hence, h(x) 6= 0 for all x ∈ (0, ∞). By the
MATH2039 ANALYSIS 121

intermediate value theorem, this forces either h(x) > 0 for all x > 0 or
h(x) < 0 for all x > 0 (because if there are points a and b in (0, ∞) at
which h(a) > 0 and h(b) < 0, then there is a point c between a and b at
which h(c) = 0). Since h(1) = 1 − ln(2) = 0.3069... > 0, we have that
h(x) > 0 on (0, ∞), that is, that x > ln(x + 1) for all x > 0, as desired.
(As noted above, the argument to show that h(x) > 0 for −1 < x < 0,
or equivalently that x > ln(x + 1) for −1 < x < 0, is similar, and is left
for you to write out.)
x
For the other inequality, set g(x) = ln(x + 1) − x+1 , and note that g
is differentiable, and hence continuous, for x > −1. (As above, we give
the details in the case that x > 0, and leave the case of −1 < x < 0 to
you the reader.) Note that g 0 (x) = (x+1) x
2 > 0 for x > 0. In particular,

applying the mean value theorem to g on the interval [0, x0 ], we see


that there is c in (0, x0 ) so that g(x0 ) − g(0) = g 0 (c)(x0 − 0) > 0, since
both g 0 (c) > 0 and x0 > 0. Hence, g(x0 ) > g(0) = 0 for all x > 0. That
x
is, ln(x + 1) > x+1 for all x > 0.
(3) Oops...

Solution 7.11:
(1) we know that there is one solution to f (x) = 0 in [−a, a], namely
x = 0 (which can be found with using the intermediate value theorem
or by inspection). To see that there are no others, we again use Rolle’s
theorem: if there were b in [−a, a], b 6= 0, with f (b) = 0, then there
would exist some point c between b and 0 with f 0 (c) = 0. However,
f 0 (x) = 1995x1994 + 941442x122 + 1 and so f 0 (c) ≥ 1 > 0 for all c ∈ R.
Hence, by Rolle’s theorem, there is no second solution to f (x) = 0.
(2) again working with g(x) = tan(x) − e−x , we saw earlier that there is
a solution to g(x) = 0 in the interval [−1, 1]. However, since g 0 (x) =
sec2 (x) + e−x > 0 for all x ∈ (−1, 1), Rolle’s theorem implies that there
can be no second solution to g(x) = 0 in the interval [−1, 1]. (It is the
same reasoning as before: if there were two solutions to g(x) = 0, then
there would exist a point c between them at which g 0 (c) = 0; however,
the calculation above shows that g 0 (c) 6= 0 for all c in (−1, 1))
(3) we don’t have enough information to decide whether we’ve found all the
solutions to f (x) = 0. With f (x) = 3 sin2 (x) − 2 cos3 (x), we have that
f 0 (x) = 6 sin(x) cos(x) + 6 cos2 (x) sin(x) = 6 sin(x) cos(x)(1 + cos(x)) =
0 when x = kπ for k ∈ N (since sin(kπ) = 0) and when x = (k + 12 )π
(since cos((k + 21 )π) = 0 for k ∈ N). Note that f (kπ) = −2 cos3 (kπ) =
(−1)k+1 2 6= 0 and that f ((k + 21 )π) = 3 sin2 ((k + 21 )π) = 3 6= 0. So, for
any m ∈ N, consider the interval (mπ, (m + 2)π).

So, there exist three points in this interval at which f 0 (x) = 0, namely
at (m + 12 )π, (m + 1)π, and (m + 32 )π, and our earlier analysis using
the intermediate value theorem found only two points in this interval at
which f (x) = 0. However, while Rolle’s theorem yields that two points
122 PERIŠIĆ

at which f (x) = 0 yields one point at which f 0 (x) = 0, we are unable to


argue the other way: there may be many points at which f 0 (x) = 0 and
still no points at which f (x) = 0. This example shows the limitations
of this sort of analysis.
(4) for f (x) = 3 + x5 − 1001x2 on x > 0, again differentiate: f 0 (x) =
5x4 − 2002x = x(5x3 − 2002), and so there is only one point in (0, ∞)
at which f 0 (x) = 0, namely the solution c of 5c3 − 2002 = 0. By
calculation, we have that c = 7.3705..., and so if there is a second
solution to f (x) = 0 in (0, ∞), it must lie in the interval (0, c) (since
by Rolle’s theorem, if there are two solutions to f (x) = 0, then there
exists at least one solution to f 0 (x) = 0 between them).

Since f (0) = 3 and since f (c) = −32624.3179..., the intermediate value


property implies that that there is a solution to f (x) = 0 in the interval
(0, c). Since the only solution to f 0 (x) = 0 on (0, ∞) occurs at c, Rolle’s
theorem implies that there can be at most two solutions to f (x) = 0 in
(0, ∞), and we have found them both.
Solution 7.12: (In these problems, I’ve stopped explicitly checking the conti-
nuity and differentiability hypotheses of the intermediate value property and of
Rolle’s theorem and the mean value theorem, because they have been checked
so many times already and since they hold true for all the functions in this
exercise.)
(1) using the general mantra that two solutions to g(x) = 0 yield one
solution to g 0 (x) = 0 via Rolle’s theorem, let’s see if we can find three
solutions to g(x) = 0 for g(x) = x3 − 12πx2 + 44π 2 x − 48π 3 + cos(x) − 1.
Factoring, we see that g(x) = (x − 2π)(x − 4π)(x − 6π) + cos(x) − 1, and
so g(2π) = g(4π) = g(6π) = 0. By Rolle’s theorem, there then exists
a in (2π, 4π) and b in (4π, 6π) so that g 0 (a) = g 0 (b) = 0, as desired.
(Also, note that the mixture of polynomial and trigonometric functions
makes it unlikely that we would find solutions to g 0 (x) = 0 by direct
calculation.)
(2) a still slightly different method: calculating, we see that f 0 (x) = 4x3 −
π 3 − cos(x), and that f 0 (−10) = −4000 − π 3 − cos(−1000) < 0 and that
f 0 (10) = 4000−π 3 −cos(1000) > 0. Since f is continuous on R, it is cer-
tainly continuous on the interval [−10, 10], and so by the intermediate
value property, there is some a in (−10, 10) at which f 0 (a) = 0.
(3) label the points at which g vanishes as a1 < a2 < · · · < an . For each
consecutive pair ak , ak+1 , Rolle’s theorem yields that there exists a
point bk between ak and ak+1 at which g 0 (bk ) = 0. This yields k − 1
points b1 , . . . , bk−1 at which the derivative g 0 (x) vanishes, as desired.
(4) let h(x) = x3 + px + q. Suppose that h has two real roots; by Rolle’s
theorem, there is then a number c between these roots at which h0 (c) =
0. However, calculating directly we see that h0 (x) = 3x2 + p ≥ p > 0
for all x ∈ R, and so there are no solutions to h0 (x) = 0. Hence, there
can be at most one root of h.
MATH2039 ANALYSIS 123

To see that there is a root, we note that since h has odd degree (and
since the coefficient of the highest degree term is positive), we have
that limx→∞ h(x) = ∞ and limx→−∞ h(x) = −∞. Hence, we can find
a point a at which h(a) > 0 and a point b at which h(b) < 0, and the
intermediate value property then implies that there is a point between
a and b at which h(x) = 0.
124 PERIŠIĆ

Solutions to Exercises, Chapter 8

Solution 8.6: [Note that for some of these limits, we do not need to use
as heavy a piece of machinery as l’Hopital’s rule, just some clever simplify-
ing.]
(1) since this limit has the indeterminate form 00 (since both limx→2 (1 −
cos(πx)) = 0 and limx→2 sin2 (πx) = 0), we may use l’Hopital’s rule:
1 − cos(πx) π sin(πx) 1 1
lim 2 = lim = lim = .
x→2 sin (πx) x→2 2π sin(πx) cos(πx) x→2 2 cos(πx) 2

(Note that we may also evaluate this limit without l’Hopital’s rule,
using the trigonometric identity sin2 (θ) + cos2 (θ) = 1, as follows:
1 − cos(πx) 1 − cos(πx) 1 1
lim 2 = lim 2
= lim = .)
x→2 sin (πx) x→2 1 − cos (πx) x→2 1 + cos(πx) 2

(2) again, here we have the choice of factoring or using l’Hopital’s rule. I
feel like factoring:
x7 + 1 (x + 1)(x6 − x5 + x4 − x3 + x2 − x + 1)
lim = lim
x→−1 x3 + 1 x→−1 (x + 1)(x2 − x + 1)
x6 − x5 + x4 − x3 + x2 − x + 1 7
= lim = .
x→−1 x2 − x + 1 3

(3) write tan(z) = sin(z)/ cos(z) and simplify:


1 + cos(πx) (1 + cos(πx)) cos2 (πx)
lim = lim
x→3 tan2 (πx) x→3 sin2 (πx)
(1 + cos(πx)) cos2 (πx) cos2 (πx) 1
= lim = lim = .
x→3 1 − cos2 (πx) x→3 1 − cos(πx) 2

(4) as this has the indeterminate form 00 , and since there seems to be no
easy simplification possible, we use l’Hopital’s rule:
1 − x + ln(x) −1 + x1
lim = lim .
x→1 1 + cos(πx) x→1 −π sin(πx)

Since this limit still has the indeterminate form 00 , we may use l’Hopital’s
rule again:
−1 + x1 − x12 1
lim = lim = − 2.
x→1 −π sin(πx) x→1 −π 2 cos(πx) π

(5) this has the indeterminate form ∞0 , and so we rewrite it:


 1/x
lim (ln(x))1/x = lim eln(ln(x)) = elimx→∞ ln(ln(x))/x .
x→∞ x→∞
MATH2039 ANALYSIS 125


The exponent has the indeterminate form ∞, and so we may use
l’Hopital’s rule:
1 1
ln(ln(x)) ln(x) · x
lim = lim = 0.
x→∞ x x→∞ 1
Hence, we see that
lim (ln(x))1/x = elimx→∞ ln(ln(x))/x = e0 = 1.
x→∞

(6) factoring, we see that


x2 + x − 6 (x − 2)(x + 3) x+3 5
lim 2
= lim = lim = .
x→2 x −4 x→2 (x − 2)(x + 2) x→2 x + 2 4

(7) as this limit has the indeterminate form 00 , we may use l’Hopital’s rule:
x + sin(2x) 1 + 2 cos(2x) 1+2
lim = lim = = −3.
x→0 x − sin(2x) x→0 1 − 2 cos(2x) 1−2

(8) since this limit has the indeterminate form 00 , we may apply l’Hopital’s
rule:
ex − 1 ex
lim = lim = ∞.
x→0 x2 x→0 2x

(9) in this limit, though we need to check at each stage, we will apply
l’Hopital’s rule four times, as the original limit has the indeterminate
form 00 , and each of the first three applications of l’Hopital’s rule results
in a limit still in the indeterminate form 00 .
ex + e−x − x2 − 2 ex − e−x − 2x ex − e−x − 2x
lim 2 = lim = lim
x→0 sin (x) − x2 x→0 2 sin(x) cos(x) − 2x x→0 sin(2x) − 2x

ex + e−x − 2
= lim
x→0 2 cos(2x) − 2

ex − e−x
= lim
x→0 −4 sin(2x)

ex + e−x 1
= lim =− .
x→0 −8 cos(2x) 4

(10) this limit has the indeterminate form ∞∞ , and so we apply l’Hopital’s
rule:
1
ln(x)
lim = lim x = 0.
x→∞ x x→∞ 1

(11) here, we first attempt to evaluate the limit by factoring, a sensible first
step for limits of rational functions:
x3 − x2 − x − 2 (x − 2)(x2 + x + 1) x2 + x + 1 7
lim 3 2
= lim 2
= lim 2
= .
x→2 x − 3x + 3x − 2 x→2 (x − 2)(x − x + 1) x→2 x − x + 1 3
126 PERIŠIĆ

(12) again, we first attempt to evaluate the limit by factoring:


x3 − x2 − x + 1 (x − 1)(x2 − 1) x+1
lim 3 2
= lim = lim = 2.
x→1 x − 2x + x x→1 x(x − 1)2 x→1 x
MATH2039 ANALYSIS 127

Solutions to exercises, Chapter 9

Solution 9.26:
(1) this is an improper integral because 1/x3/2 is continuous on (0, 4] and
limx→0+ 1/x3/2 = ∞. So, we evaluate:
Z 4 Z 4
1 1
3/2
dx = lim 3/2
dx
0 x c→0+ c x
Z 4
= lim x−3/2 dx
c→0+ c
 
2 2
= lim − √ + √
c→0+ 4 c
1
= −1 + 2 lim √ = ∞,
c→0+ c
and so this improper integral diverges.
(2) this is an improper integral because the interval of integration is [1, ∞),
which is not a closed interval. So, we evaluate:
Z ∞ Z M
1 1
dx = lim dx
1 x+1 M →∞ 1 x+1
  
1
= lim ln(M + 1) − ln = ∞,
M →∞ 2
and so this improper integral diverges.
(3) this is an improper integral, as the interval of integration is [5, ∞),
which is not a closed interval. So, we evaluate:
Z ∞ Z M
1 1
3/2
dx = lim dx
5 (x − 1) M →∞ 5 (x − 1)3/2
Z M
= lim (x − 1)−3/2 dx
M →∞ 5
 
2
= lim − √ + 1 = 1,
M →∞ M −1
and so this improper integral converges to 1.
(4) this is an improper integral because 1/(9 − x)3/2 is continuous on [0, 9)
and limx→9− 1/(9 − x)3/2 = ∞. So, we evaluate:
Z 9 Z c
1 1
3/2
dx = lim dx
0 (9 − x) c→9− (9 − x)3/2
Z0 c
= lim (9 − x)−3/2 dx
c→9− 0
 
2 2
= lim − + √ = ∞,
c→9− 3 9−c
and so this improper integral diverges.
128 PERIŠIĆ

(5) this is an improper integral, since the interval of integration is (−∞, −2]
and so is not a closed interval. So, we evaluate:
Z −2 Z −2
1 1
3
dx = lim dx
−∞ (x + 1) M →−∞ M (x + 1)3
 
1 1 1 1 1
= lim − + =− ,
M →−∞ 2 (−2 + 1)2 2 (M + 1)2 2
and so this improper integral converges to − 12 .
(6) this is an improper integral, since the integrand is not continuous on
[−1, 8] as it has a discontinuity at 0. Hence, we can break it up as the
sum of two improper integrals:
Z 8 Z 0 Z 8
1/3 1/3
dx/x = dx/x + dx/x1/3 ,
−1 −1 0
R8 R0 R8
and we have that −1 dx/x1/3 converges if both −1 dx/x1/3 and 0 dx/x1/3
converge. So, we evaluate:
Z 0 Z c
1 1
1/3
dx = lim dx
−1 x c→0− −1 x1/3
Z c
= lim x−1/3 dx
c→0− −1
 
3 2/3 3 3
= lim c − =− ,
c→0− 2 2 2
and
Z 8 Z 8
1 1
1/3
dx = lim dx
0 x c→0+ c x1/3
Z 8
= lim x−1/3 dx
c→0+
c 
3 2/3 3 2/3
= lim 8 − c = 6.
c→0+ 2 2
Since both these improper
R8 integrals converge, we see that the original
improper integral −1 dx/x 1/3
converges to 92 .

(7) this is an improper integral, since the interval of integration is [2, ∞)


and hence is not a closed interval. So, we evaluate:
Z ∞ Z M
1 1
1/3
dx = lim dx
2 (x − 1) M →∞ 2 (x − 1)1/3
Z M
= lim (x − 1)−1/3 dx
M →∞ 2
 
3 3
= lim (M − 1)2/3 − = ∞,
M →∞ 2 2
and so this improper integral diverges.
MATH2039 ANALYSIS 129

(8) this is an improper integral since the interval of integration is (−∞, ∞)


and hence is not a closed interval. We evaluate this improper
R∞ integral by
breaking it up as the sum of two improper integrals −∞ xdx/(x2 +4) =
R0 R∞
−∞
xdx/(x2 + 4) + 0 xdx/(x2 + 4), and evaluating the two resulting
improper integrals separately. So,
Z 0 Z 0
x x
2+4
dx = lim 2+4
dx
−∞ x M →−∞ M x
 
1 2 1
= lim ln(M + 4) − ln(4) = ∞.
M →−∞ 2 2
Since one of these two improper integrals diverges, weR don’t need to
0
evaluate the other one, as the original improper integral −∞ xdx/(x2 +
4) necessarily diverges.
(9) this is an improper
√ √
integral, as the integrand is continuous on (0, 1] and
limx→0+ e x / x = ∞. So, we evaluate:
√ Z 1 √x
1 Z 1
e x √
Z
e
√ dx = lim √ dx = lim √ 2eu du = lim 2(e − e c )
0 x c→0+ c x c→0+ c c→0+

and so this improper integral converges to 2(e − 1).


(10) this is an improper integral, as the interval of integration is [1, ∞) and
so is not a closed interval. Moreover, the integrand is not continuous
at 0 but limx→1+ 1/x ln(x) = ∞, and so we need toR break this im-

proper integral into the sum of two improper integrals 1 dx/x ln(x) =
R2 R∞
1
dx/x ln(x)+ 2 dx/x ln(x), and evaluate the two resulting improper
integrals separately. So,
Z 2 Z 2
1 1
dx = lim dx
1 x ln(x) c→1+ c x ln(x)

= lim (ln(ln(2)) − ln(ln(c))) = ∞,


c→1+

and so this
R ∞ improper integral diverges, and so the original improper
integral 1 dx/x ln(x) necessarily diverges.
R∞
Solution 9.27: We first need to write −∞ (1 + x)dx/(1 + x2 ) as the sum of
two improper integrals, for instance
Z ∞ Z 0 Z ∞
1+x 1+x 1+x
2
dx = 2
dx + dx,
−∞ 1 + x −∞ 1 + x 0 1 + x2
and then evaluate the two resulting improper integrals separately. So,
Z ∞ Z M
1+x 1+x
2
dx = lim dx
0 1 + x M →∞ 1 + x2
"0Z #
M Z M
1 x
= lim dx + dx
M →∞ 0 1 + x2 0 1 + x2
  
1 1
= lim (arctan(M ) − arctan(0)) + ln(1 + M 2 ) − = ∞,
M →∞ 2 2
130 PERIŠIĆ

R∞
since limM →∞ ln(1 + M 2 ) = ∞, and so the original improper integral −∞
(1 +
x)dx/(1 + x2 ) diverges.
Rt
However, when we evaluate limt→∞ −t (1 + x)dx/(1 + x2 ), we get
Z t Z t Z t 
1+x 1 x
lim dx = lim dx + dx
t→∞ −t 1 + x2 t→∞ −t 1 + x
2
−t 1 + x
2

π
= lim 2 arctan(t) = 2 = π,
t→∞ 2

since, using arctan(−t) = − arctan(t),


Z t
1 π
lim dx = lim (arctan(t) − arctan(−t)) = 2 = π
t→∞ −t 1 + x2 t→∞ 2
and
Z t
x 1
lim 2
dx = ln(1 + t2 ) − ln(1 + (−t)2 ) = 0
t→∞ −t 1+x 2
Rt
and so limt→∞ −t
(1 + x)dx/(1 + x2 ) converges.
MATH2039 ANALYSIS 131

Solutions to Exercises, Chapter 10

Solution 10.6:
(1) Before hitting the ground the first time, the ball travels distance a.
Between hitting the ground the first and second times, the ball travels
distance 2ra (distance ra up from the ground, and then distance ra
back to down to earth again). Between hitting the ground the second
and third times, the ball travels distance 2r2 a (distance r2 a up from the
ground, and then distance r2 a back to down to earth again). Between
hitting the ground the nth and the (n + 1)st times, the ball travels
distance 2rn a (distance rn a up from the ground, and then distance rn a
back to down to earth again). Hence, the total distance traveled is

X ∞
X
a + 2ra + 2r2 a + . . . = a+ 2rn a = a + 2ra rn−1
n=1 n=1

X 2ra a + ra
= a + 2ra rk = a + = .
1−r 1−r
k=0

(2) One way to do this problem is to actually write out the appropriate
geometric series and summing it. The easier way is to note that the
cars will crash exactly one hour after the fly leaves the front of Jack’s
car, and in that hour (given the assumption that the fly loses no time
in changing direction) the fly flies exactly 257 miles.

Solution 10.8: Note that for s < 1, we have that ns < n (even for s = 0
or s negative), and hence that n1s > n1 . Hence, if we let Sk be the k th partial
P∞
sum of the harmonic series n=1 n1 , and Tk be the k th partial sum of the series
P∞ 1 1
n=1 ns under consideration, then Tk > Sk . Since ns > 0 for all n, we have
that Tk is an unbounded monotonically increasing sequence, unbounded since
Sk is unbounded byP∞ the argument given in Example 10.7, and so Tk diverges.
So, by definition, n=1 n1s diverges.

Solution 10.10:
P∞
(1) we argue
P∞ by contradiction: suppose that n=0 (an + bn ) converges.
Since n=0 an converges by assumption, the arithmetic of series, The-
orem 10.9, yields
P∞ that their difference
P∞ also converges. However, their
difference is n=0 (an +bn −an ) = n=0 bn , which diverges by assump-
tion, yielding the desired contradiction.
(2) again we argue by contradiction: suppose that the series of multiples
P∞ Pk
n=0 can converges. Then, the sequence P Tk = n=0 can of partial
k Pk
sums converges. Note though that Tk = n=0 Pcan = c n=0 an =

cSk , where Sk is the k th partial sum of the series n=0 an . Since Tk
converges, the sequence 1c Tk = Sk also converges, by the arithmetic of
sequences (since the constant sequence 1c converges), and so the original
series converges, a contradiction.
132 PERIŠIĆ

Solution 10.12:

(1) by what has just been done, all we need are two convergent series.
For instance,
P∞take an =P (0.5)n and bn = (0.3)n for all n ≥ 0. Then,
P ∞ ∞
n=0 an , n=0 bn , and n=0 an bn are all convergent geometric series.
P∞
(2) takeP an = 1 for all n ≥ 0 and bn = 1 for all n ≥ 0. Then, both n=0 an
∞ P ∞
and n=0 bn are both divergent geometric series, as is n=0 an bn (since
an bn = 1 for all n ≥ 0).
1
P∞
P∞one, let’s take an = bn = n for all n ≥ 1. Then, both n=1 an
(3) for this
and n=1 bn are the harmonic
P∞ series, P
and hence divergent. However,
∞ 1
the series of products a
n=1 n nb = n=1 n2 is convergent, by the
discussion in Example 10.7.
P∞
(4) take any convergent series, for example n=0 P∞ (0.5)n , andPset an = bn =
an ∞
(0, 5)n . Then, the series of quotients is n=0 bn = n=0 1, which
diverges.

(5) here, we can take an = n12 and bn = n14 for n ≥ 1. Then, both
P∞ P∞ 1 P∞ P∞ 1
n=1 an = n=1 n2 and n=1 bn = n=1 n4 converge by Exercise
10.7, as does the series of quotients, as abnn = n12 .
P∞ P∞ n
(6) let’s
P∞ use geometric
P∞ nseries again: both of n=0 an = n=0 6 and
n=0 bn =
P∞ n=0 2 are
P∞ divergent geometric series, and the series of
quotients n=0 abnn = n=0 3n is also a divergent geometric series.
P∞ P∞ P∞ P∞ 2
(7) n=1 an = n=1 1 and n=1 bn =P n=1 n both P∞ diverge, but the
∞ an
corresponding sequence of quotients n=1 bn = n=1 n12 converges.

Pk P∞
Solution 10.19: Let Sk = n=1 an be the k th partial sum of n=1 an .

(1) Since limn→∞ cn = 0 and cn > 0 for all n, there exists M > 0 so that
0 < cn < 1 for n > M . Let M = max(1, c1 , c2 , . . . , cM ), and note that
th
n ≤ M for all n ≥ 1. In particular, the k
cP partial sum of the series

n=1 an cn satisfies

k
X k
X
an cn ≤ an M = M Sk .
n=1 n=1
P∞
Hence, the sequence of partial sums of the series n=1 an cn forms
a monotonic
P∞ (since all the a n and cn are positive), bounded (above
by
P∞ M a
n=1 n , below by 0) sequence, and so converges. That is,
a c
n=1 n n converges.

(2) The proof in the case that limn→∞ cn = c 6= 0 is very similar to the
proof in the case that limn→∞ cn = 0. Since limn→∞ cn = c 6= 0, there
exists M > 0 so that cn < c + 1 for n > M . Let M = max(c +
1, c1 , c2 , . . . , cM ), and note that cn ≤ M for all n ≥ 1. In particular,
MATH2039 ANALYSIS 133

P∞
the k th partial sum of the series n=1 an cn satisfies
k
X k
X
an cn ≤ an M = M Sk .
n=1 n=1
P∞
Hence, the sequence of partial sums of the series n=1 an cn forms
a monotonic
P∞ (since all the a n and c n are positive), bounded (above
by
P∞ M n=1 an , below by 0) sequence, and so converges. That is,
n=1 an cn converges.

Solution 10.26: we make implicit use of the fact that convergence and absolute
convergence are the same for series with positive terms.
(1) converges absolutely: we could apply the ratio test, but we do not
need to use such heavy machinery. Instead, we note that
∞ ∞ ∞  n
X 2n−1 1 X 2n 1X 2 1 1 3
n
= n
= = = ,
n=0
3 2 n=0 3 2 n=0 3 2 (1 − 2/3) 2
P∞ 2n
since n=0 3n is a convergent geometric series.
(2) diverges: this is a geometric series, and since 1.01 > 1, it is a divergent
geometric series.
(3) converges absolutely: this is a convergent geometric series, since
e
10 < 1, and it converges to
∞  ∞
X e n X  e n 1 10 10 − e e
= −1= −1= − = .
n=1
10 n=0
10 1 − e/10 10 − e 10 − e 10 −e

(4) converges absolutely: we use the comparison test: since n2 + Pn∞+ 1 >
1
n2 for all n ≥ 1, we have that n2 +n+1 < n12 for all n ≥ 1. Since n=1 n12
P∞ 1
converges, we have that n=1 n2 +n+1 converges.
√ √
(5) diverges: note that for n ≥ 1, we have that n ≥ n, and so n P + n≤

2n. Therefore, n+ n ≥ 2n for n ≥ 1. Since the harmonic series n=1 n1
1√ 1
P∞ 1
diverges, its multiple
P∞ 2n diverges, and hence by the comparison
n=1 √
test the series n=1 1/(n + n) diverges.
(6) converges absolutely: since 1 + 3n > 3n for all n ≥ 1, we have that
1 n
1 1
P∞ 1 P∞
1+3n < 3n for all n ≥ 1. Since n=1
P∞ 3n = n=1 3 converges, the
second convergence test yields that n=1 1/(1 + 3n ) converges.
(7) diverges: we’ll use the limit comparison test: for large values of n, it
2
seems that n10n 1
3 −1 behaves like a constant multiple of n , and in fact

10n2 /(n3 − 1) 10n3


lim = lim 3 = 10 = L.
n→∞ 1/n n→∞ n − 1
P∞
Since the limit exists and 0 < L = 10 < ∞, and since n=1 n1 diverges,
P∞
the limit comparison test yields that n=2 10n2 /(n3 − 1) diverges.
134 PERIŠIĆ

(8) converges absolutely: again we’ll √ use the limit comparison test: for
large values of n, it seems that 1/ 37n3 + 3 behaves like 1/n3/2 , and
in fact

1/ 37n3 + 3 n3/2 1 1
lim = lim √ = lim p = √ = L.
n→∞ 1/n3/2 n→∞ 3
37n + 3 n→∞ 37 + 3/n 3 37

Since the limit exists and 0 < L = √137 < ∞, and since n=1 n3/21
P
con-
P∞ √
3
verges, the limit comparison test yields that n=1 1/ 37n + 3 con-
verges.
(9) converges absolutely: we start this one with a bit of algebra, namely
√ √
n n 1
< 2 = 3/2 .
n2 + n n n
P∞
¿From Example 10.7,P we know that n=1 1/n3/2 converges, and so by
∞ √ 2
the comparison test, n=1 n/(n + n) converges.
1
(10) diverges: since ln(n) < n for all n ≥ 2, we have that ln(n) > n1
P∞
for all n ≥ 2, and so n=2 2/ ln(n)P diverges by the comparison test,

comparing it to the harmonic series n=1 n1 .
(11) converges absolutely: since 0 < sin2 (n) ≤ 1 for all n ≥ 1, we have
that
sin2 (n) 1 1
0< 2 ≤ 2 < 2
n +1 n +1 n
n ≥ 1. Since we are dealing with a series with positive
for allP P∞ terms and

since n=1 n12 converges by Example 10.7, we have that n=1 sin2 (n)/(n2 +
1) converges by the comparison test.
(12) converges absolutely: for this series, we start with a bit of algebraic
massage:
 n
n + 2n n + 2n 2n + 2n 2
< < =2 .
n + 3n 3n 3n 3
So,
Pthe comparison
n test, comparing with the convergent geometric series
∞ P∞
2 n=0 32 yields that n=1 (n + 2n )/(n + 3n ) converges.
(13) converges absolutely: since 1/(n2 ln(n)) < 1/n2 for n ≥ 3, since
P∞ ≥ 1 2for n ≥ 3, we have by the second comparison test that
ln(n)
n=2 1/(n ln(n)) converges.

(14) diverges: for large values of n, it seems that the nth in the series is
approximately n1 , and so we might guess that the series diverges by the
limit comparison test. To check this guess, we need to evaluate
(n3 + 1)/(n4 + 2) n4 + n
lim = lim 4 = 1 = L.
n→∞ 1/n n→∞ n + 2

P∞limit exists and since 0 < L =P1∞< ∞, and since the harmonic
Since the
series n=1 n1 diverges, we have that n=1 (n3 + 1)/(n4 + 2) diverges
by the limit comparison test.
MATH2039 ANALYSIS 135

(15) converges absolutely: since n+n1 3/2 < n3/2


1
for all n ≥ 1 and since
P∞ 1
P∞
n=1 n3/2 converges by Example 10.7, we have that n=1 1/(n+n3/2 )
converges by the second comparison test.
(16) converges absolutely: for large values of n, it seems that the nth
term in this series is approximately equal to n102 , and so we might guess
that this series converges by use of the limit comparison test. To verify
this guess, we calculate
10n2 /(n4 + 1) n4
lim 2
= lim 4 = 1 = L.
n→∞ 10/n n→∞ n + 1
P∞
Since the limit exists and since 0 < L = 1 < ∞, and since n=1 n102
P∞
converges by Example 10.7, we have that n=1 10n2 /(n4 +1) converges
by the limit comparison test.
(17) converges absolutely: for large values of n, it seems again that the
nth term in this series is approximately equal to n12 , and so we might
guess that this series converges by use of the limit comparison test. To
verify this guess, we calculate
(n2 − n)/(n4 + 2) n4 − n3
lim 2
= lim = 1 = L.
n→∞ 1/n n→∞ n4 + 2
P∞
Since the limit exists and since 0 < L = 1 < ∞, and since n=1 n12 con-
P∞
verges by Example 10.7, we have that n=2 (n2 − n)/(n4 + 2) converges
by the limit comparison test.
(18) diverges: for large values of n, it seems that the nth term of this series
is approximately equal to n1 , and so we might guess that this series then
diverges by the limit comparison test. To verify this guess, we calculate

1/ n2 + 1 n n
lim = lim √ = lim p = 1 = L.
n→∞ 1/n n→∞ n2 + 1 n→∞ n 1 + 1/n2
P∞
Since the limit exists and since 0 < L = 1 < ∞, and since n=1 n1
P∞ √
diverges by Example 10.7, we have that n=2 1/ n2 + 1 diverges by
the limit comparison test.
(19) converges absolutely: since
 n
1 1 1
< n = ,
3 + 5n 5 5
1 n
P∞  P∞
and since n=0 5 converges, the comparison test yields that n=1 1/(3+
5n ) converges.
(20) diverges: first note that since ln(n) < n for all n ≥ 2, this is a series of
positive
P∞ terms. Also, n − ln(n) < n, and
Pso 1/(n − ln(n)) > 1/n. Hence,

since n=1 n1 diverges, we have that n=2 1/(n − ln(n)) diverges, by
the comparison test.
(21) converges absolutely: since 0 <Pcos2 (n) ≤ 1 for all N ≥ 1, we

have that cos2 (n)/3n < 1/3n . Since n=0 31n is a convergent geometric
P∞
series, we have by the comparison test that n=1 cos2 (n)/3n converges.
136 PERIŠIĆ

P∞ 1
(22) converges absolutely: since 1/(2n + 3n ) < 1/2n and
P∞since nn=0 2nn
converges, the second comparison test yields that n=1 1/(2 + 3 )
converges.

(23) converges

absolutely: since 1 + n√≥ 2 for n ≥ 1, we have that
n1+ nP≥ n2 for n ≥ 1, and so 1/n(1+ n) ≤ 1/n2 for n ≥ 1. Hence,

since n=1 n12 converges by Example 10.7, we have by the comparison
P∞ √
test that n=1 1/n(1+ n) converges.
(24) converges absolutely: since 2n (n + 1) > P 2n for n ≥ 1, we have

that 1/(2 (n + 1)) < 1/2 for n ≥ 1. Since n=1 21nP
n n
is a convergent

geometric series, we have by the comparison test that n=1 1/(2n (n +
1)) converges.
(25) diverges: since factorials are involved, we first see whether the ratio
test gives us any information, and so we evaluate
(n + 1)!/((n + 1)2 en+1 ) (n + 1)!n2 en n2 n+1
lim = lim = lim = ∞,
n→∞ n!/(n2 en ) n→∞ n!(n + 1)2 en+1 n→∞ (n + 1)2 e
P∞
and since ∞ > 1, the ratio test implies that n=1 n!/(n2 en ) diverges.

[Though it’s not obvious how, we could also have applied the nth term
test for divergence, since for large values of n we have
n! (n − 1)(n − 2)! n−1n−2 2 1 n−12 1 1
= = ··· 2 > > 3.
n2 en nen n e ee n e e2 e
We simplified by noting that the middle terms n−2 3
e , . . . , e are all greater
than 1 and that n−1 1 n!
n > 2 for n large. Hence, limn→∞ n2 en 6= 0.]

(26) converges absolutely: there is not an obvious comparison to make,


and so we try the ratio test:
√ r
n + 1/(3n+1 ln(n + 1)) 1 ln(n) n+1 1
lim √ = lim = ,
n→∞ n/(3n ln(n)) n→∞ 3 ln(n + 1) n 3
ln(n)
since limn→∞ ln(n+1) = 1, for instance using l’Hopital’s rule. Since
1
P∞ √ n
3 < 1, the ratio test yields that n=1 n/(3 ln(n)) converges.

(27) converges absolutely: since there are factorials involved, we first try
the ratio test:
(2(n + 1))!/((n + 1)!)3 (2n + 2)(2n + 1)
lim 3
= lim = 0 < 1,
n→∞ (2n)!/(n!) n→∞ (n + 1)3
P∞
and so the ratio test yields that n=2 (2n)!/(n!)3 converges.
(28) converges absolutely: note that the numerator of each term is either
0 or 2, and so this is a series with non-negative
P∞ terms. Also, (1 −
(−1)n )/n4 < 2/n4 for all n ≥ 1 and n=1 n14 converges by Example
P∞
10.7, and so by the comparison test n=1 (1 − (−1)n )/n4 converges.
MATH2039 ANALYSIS 137

(29) diverges: we start with a bit of algebraic simplification:


2 + cos(n) 1 1
≥ > .
n + ln(n) n + ln(n) 2n
(The first inequality holds since 2 + cos(n) ≥ 2 + (−1) = 1 for all
n ≥ 1, and the second inequality holds since
P∞ ln(n) < n for all n ≥ 1,
1
and so n + ln(n) < n + n = 2n.) Since n=1 2n diverges (as it is a
constant
P∞ multiple of the harmonic series), the comparison test yields
that n=1 (2 + cos(n))/(n + ln(n)) diverges.
(30) diverges: for this one, we use the integral test. Set
1
f (x) = p ,
x ln(x) ln(ln(x))

p an = f (n) for all n ≥ 3. (The restriction that n ≥ 3 is to ensure


so that
that ln(ln(n)) is well defined.) In order to apply the integral test,
we need to know that f (x) is decreasing, which involves calculating a
derivative and checking its sign:
 
p p 1 1
− ln(x) ln(ln(x)) + ln(ln(x)) + x ln(x) √
2 ln(ln(x)) x ln(x)
f 0 (x) = p < 0.
(x ln(x) ln(ln(x)))2
Hence, the integral test can be applied, and says that the sum

X 1
p
n=3 n ln(n) ln(ln(n))
R∞ RM
converges iff 3 f (x)dx = limM →∞ 3 f (x)dx exists. So, we calculate
Z M Z M
1
lim f (x)dx = lim p dx
M →∞ 3 M →∞ 3 x ln(x) ln(ln(x))
p
lim 2 ln(ln(x)) M

= 3
M →∞
P∞ p
which diverges, and so n=3 1/(n ln(n) ln(ln(n))) diverges.
(31) converges absolutely: try the ratio test, since there are factorials
about:
n
(n + 1)(n+1) /(π (n+1) (n + 1)!)

n+1 1
lim = lim
n→∞ nn /(π n n!) n→∞ n π
 n
1 1 e
= lim 1 + = = L.
n→∞ n π π
Since the limit exists and since L < 1, the ratio test yields convergence
of the sum

X nn
.
n=1
π n n!

(32) converges absolutely: since both the numerator and the denominator
are raised to (essentially) the same power, we try the root test, and so
138 PERIŠIĆ

 1/n
2n+1
need to calculate: limn→∞ nn = limn→∞ 21/n n2 = L = 0 (since
limn→∞ 21/n = 20 =P1). Since the limit exists and since L < 1, the

root test yields that n=1 2n+1 /nn converges.
(33) converges conditionally: P we first test for √
absolute convergence,
√ by
∞ P∞
considering the related series n=1 |(−1)n−1 / n| = n=1 1/ n, which
diverges by Example 10.7.

We now test for convergence. This is an alternating series, and so we


use the alternating series test: write
∞ ∞ ∞
X (−1)n−1 X (−1)n X
√ = (−1) √ = (−1) (−1)n an ,
n=1
n n=1
n n=1

where an = √1n > 0 for all n ≥ 1. Since limn→∞ an = limn→∞ √1n = 0


1
and since an+1 = √n+1 < √1n = an for all n ≥ 1, the alternating series
test applies and yields that this series converges.

Hence, this series converges but does not converge absolutely. That is,
the series converges conditionally.
(34) converges conditionally: we first check Pfor absolute convergence,

that is,Pconvergence of the associated series n=1 | cos(πn)/((n+1) ln(n+

1))| = n=1 1/((n + 1) ln(n + 1)). For this series, we apply the integral
test, with f (x) = 1/((x + 1) ln(x + 1)). Since
 
1
− ln(x + 1) + (x + 1) x+1 −(ln(x + 1) + 1)
f 0 (x) = 2 2
= <0
(x + 1) (ln(x + 1)) (x + 1)2 (ln(x + 1))2
for x ≥ 1, the integral test yields that the series converges if and only
R∞ RM
if 1 f (x)dx = limM →∞ 1 f (x)dx exists, so we calculate:
Z M
1
lim ln(ln(x + 1)) M

lim dx = 1
M →∞ 1 (x + 1) ln(x + 1) M →∞

= lim (ln(ln(M + 1)) − ln(ln(2))),


M →∞

which diverges (very very slowly). So, the series does not converge
absolutely.

We now test for convergence. Since cos(πn) = (−1)n , this is an al-


ternating series, and we start with the alternating series test. Since
(n + 1) ln(n + 1) < (n + 2) ln(n + 2) for all n ≥ 1, we have that 1/((n +
1) ln(n + 1)) > 1/((n + 2) ln(n + 2)) for n ≥ 1. Since limn→∞ 1/((n +
1) ln(n + 1)) = 0 (and since 1/((n + 1) ln(n + 1)) > 0 for n ≥ 1), the
alternating series test applies and yields that the series converges.

Hence, this series converges but does not converge absolutely. That is,
the series converges conditionally.
MATH2039 ANALYSIS 139

(35) diverges: since limn→∞ (n2 − 1)/(n2 + 1) = 1, we have that


limn→∞P (−1)n (n2 − 1)/(n2 + 1) does not exist,

and so n=1 (−1)n (n2 − 1)/(n2 + 1) diverges by the nth term test for
divergence.
(36) converges absolutely: we first Ptest for absolute convergence, by con-
∞ n n
P∞ n
sidering the associated series n=1 |(−1) /(nπ )| = n=1 1/(nπ ).
∞ 1
Since 1/(nπ n ) ≤ 1/π n for n ≥ 1 P
P
and since n=0 π n converges, the
∞ n
second
P∞ comparison test yields that n=1 1/(nπ ) converges, and hence
that n=1 (−1)n /(nπ n ) converges absolutely.
(37) converges conditionally: we first test P∞for absolute convergence, that
n 2 3
is, convergence
P∞ of the associated series n=1 |(−1) (20n −n−1)/(n +
2 2 3 2 th
n +33)| = n=1 (20n −n−1)/(n +n +33). Since the n term looks
like a constant multiple of n1 for large n, let’s try the limit comparison
test:
(20n2 − n − 1)/(n3 + n2 + 33) 20n3 − n2 − n
lim = lim = 20 = L.
n→∞ 1/n n→∞ n3 + n2 + 33

Since the limit exists and 0 < L < ∞, the series being considered here
diverges, since the harmonic series diverges. So, the original series does
not converge absolutely.
P∞
We now test for convergence. The series n=1 (−1)n (20n2 −n−1)/(n3 +
∞ 2
−n−1
n2 + 33) = n=1 (−1)n an is an alternating series, since 20n
P
n3 +n2 +33 > 0
for n ≥ 1, and so let’s check whether it satisfies the conditions of the
alternating series test. Since (20n2 − n − 1)/(n3 + n2 + 33) is a rational
function and the denominator has higher degree than the numerator,
we have that limn→∞ (20n2 −n−1)/(n3 +n2 +33) = 0. All that remains
to check is whether the an are monotonically decreasing. For this, let
f (x) = (20x2 − x − 1)/(x3 + x2 + 33), so that f (n) = an , and check
that it’s decreasing, which involves calculating f 0 (x):
−20x4 + 2x3 + 4x2 + 1322x − 33
f 0 (x) = <0
(x3 + x2 + 33)2
for all x greater than any of the roots of the numerator. So, the alter-
nating series test applies, and yields that this series converges.

Hence, this series converges but does not converge absolutely. That is,
the series converges conditionally.
(38) diverges: note that, for n ≥ 101, we have
n! n(n − 1) · · · 1 n n−1 101 100 99 1 99 1
n
= n
= ··· ··· > ··· ,
100 100 100 100 100 100 100 100 100 100
and so limn→∞ n!/(−100)n does not exist. Hence, by the nth term test
for divergence, the series diverges.
(39) converges absolutely: we apply the integral test, with the function
1
f (x) = x ln(x)(ln(ln(x))) 2 . First, we check to see that f (x) is decreasing,
140 PERIŠIĆ

by calculating its derivative:


−(ln(x)(ln(ln(x)))2 + (ln(ln(x)))2 + 1)
f 0 (x) = <0
(x ln(x)(ln(ln(x)))4
for x ≥ 2 (and the denominator is non-zero for x ≥ 3). So, now we
need to calculate
Z ∞ Z M
1
f (x)dx = lim 2
dx
3 M →∞ 3 x ln(x)(ln(ln(x))
 
1 M −1 1 1
= lim 3 = lim + = .
M →∞ ln(ln(x) M →∞ ln(ln(M )) ln(ln(3)) ln(ln(3))
P∞
Since the limit converges, n=3 1/(n ln(n)(ln(ln(n)))2 ) converges ab-
solutely.
(40) diverges: we start with a bit of arithmetic, noting that the numerator
satisfies: (1 + (−1)n ) = 0 for n odd and (1 + (−1)n ) = 2 for n even.
Hence, the terms of the series are non-zero only for n even, so let’s make
the substitution n = 2k for k ≥ 1. Then, for n even, we have that

1 + (−1)n 2 2 1
√ =√ =√ >√ .
n 2k k k
P∞
Hence, by√ the comparison test and Example 10.7, we have that n=1 (1+
(−1)n )/ n diverges.
(41) converges absolutely: again, we begin with a bit of algebra, simpli-
fying the nth in the series by noting that
en cos2 (n) en en  e n
n
≤ n
≤ n = ,
1+π 1+π π π
where Pthe first inequality follows from cos2 (n) ≤ 1 for all n ≥ 1.
∞ n
P∞ nn=02(e/π) converges,
Since the second comparison test yields that
n
n=1 e cos (n)/(1 + π ) converges.
(42) converges absolutely: since there are factorials involved, let’s first
try the ratio test:
4 4
(n + 1)4 /(n + 1)!
 
n+1 n! n+1 1
lim 4
= lim = lim = 0.
n→∞ n /n! n→∞ n (n + 1)! n→∞ n n+1
Since the limit exists and since 0 = L < 1, the ratio test yields that the
series converges.
(43) converges absolutely: again, since there are factorials involved, we
first try the ratio test:
(2(n + 1))!6(n+1) /(3(n + 1))! 6(2n + 2)(2n + 1)
lim = lim = 0.
n→∞ (2n)!6n /(3n)! n→∞ (3n + 3)(3n + 2)(3n + 1)

Since the limit exists and since 0 = L < 1, the ratio test yields that the
series converges.
MATH2039 ANALYSIS 141

(44) converges absolutely: and yet again, since there are factorials in-
volved, our first attempt should be with the ratio test:
p 100
(n + 1)100 2(n+1) / (n + 1)!

n+1 2
lim √ = lim √ = 0 = L.
n→∞ 100 n
n 2 / n! n→∞ n n+1
Since the limit exists and since L < 1, the ratio test yields that this
series converges.
(45) diverges: since there are factorials involved, we first try the ratio test:
(1 + (n + 1)!)/(1 + (n + 1))! 1 + (n + 1)!
lim = lim
n→∞ (1 + n!)/(1 + n)! (1 + n!)(n + 2)
n→∞

1/n! + n + 1
= lim =1
n→∞ (1/n! + 1)(n + 2)

and so the ratio test gives no information. (This discussion was put in
to remind you that the ratio test doesn’t always work with factorials.)

Hmm. Notice that when n is large, 1 + n! is very nearly equal to n!,


and so (1 + n!)/(n + 1)! is very nearly equal to n!/(n + 1)! = 1/(n + 1).
So, let’s try the limit comparison test with 1/(n + 1):
(1 + n!)/(1 + n)! (n + 1)(1 + n!) 1 + n!
lim = lim = lim = 1 = L.
n→∞ 1/(n + 1) n→∞ (n + 1)! n→∞ n!
P∞
Since the limit exists and since n=0 1/(n + 1)Pdiverges (as it’s the

harmonic series less the leading term), the series n=3 (1 + n!)/(1 + n)!
diverges by the limit comparison test.
(46) diverges: again, since there are factorials involved, we first try the
ratio test:
22(n+1) ((n + 1)!)2 (2n)! 4(n + 1)2
lim 2n 2
= lim = 1,
n→∞ (2(n + 1))! 2 (n!) n→∞ (2n + 1)(2n + 2)

and so the ratio test yields no information.

So, let’s explicitly try the nth term test for divergence. We start with
a bit of algebraic massage, namely:
22n (n!)2 = (2n · n!)2 = ((2n) · (2n − 2) · (2n − 4) · · · 4 · 2)2 ,
and so
22n (n!)2 (2n) · (2n) · (2n − 2) · (2n − 2) · · · 2 · 2
=
(2n)! (2n) · (2n − 1) · (2n − 2) · (2n − 3) · · · 2 · 1
(2n) · (2n − 2) · · · 2
= > 1.
(2n − 1) · (2n − 3) · · · 1
2n 2
In particular, the limit limn→∞
P∞2 (n!) /(2n)! cannot be zero, and so
the n term test yields that n=1 2 (n!)2 /(2n)! diverges.
th 2n

(47) converges absolutely: we first


P∞ check for absolute convergence,
P∞ namely
the convergence of the series n=1 |(−1)n /(n2 +ln(n))| = n=1 1/(n2 +
ln(n)). Since n2 + ln(n) > n2 , we have P
that 1/(n2 + ln(n)) < 1/n2 , and

so by the comparison test, the series n=1 1/(n2 + ln(n)) converges.
142 PERIŠIĆ

P∞ n 2
That is, the original series n=1 (−1) /(n + ln(n)) converges abso-
lutely.
(48) converges absolutely: we begin with a bit of algebraic massage, not-
ing that
∞ ∞ ∞ ∞  n
X (−1)2n X ((−1)2 )n X 1 X 1
n
= n
= n
= .
n=1
2 n=1
2 n=1
2 n=1
2
This is a convergent geometric series, converging to
1
− 1 = 1.
1 − 12
(The subtraction of 1 arises from the fact that the starting index in this
series is not 0, so that
∞ ∞  0 X ∞
X 1 X 1 1 1
n
= n
− = n
− 1 = 2 − 1 = 1.)
n=1
2 n=0
2 2 n=0
2

(49) converges absolutely: we first Pcheck for absolute convergence, namely


∞ P∞ n
the convergence of the series n=1 |(−2)n /n!| = n=1 2 /n!. Since
there are factorials involved, we make use of the ratio test:
2(n+1) /(n + 1)! 2
lim = lim = 0 = L.
n→∞ 2n /n! n→∞ n + 1

Since
P∞ this limit exists and satisfies L < 1, the ratio test P∞yields that
n n
n=1 2 /n! converges, and hence that the original series n=1 (−2) /n!
converges absolutely.
(50) diverges: first, note that this is not an alternating series, but is a
series with all non-positive terms. Hence, for this series, convergence
and absolute convergence are equivalent, as they are for series with
non-negative terms.

Now, for n large, n/(n2 + 1) is approximately equal to 1/n, and so let’s


try the limit comparison test with n1 . So, we calculate:
n/(n2 + 1) n2
lim = lim 2 = 1 = L.
n→∞ 1/n n→∞ n + 1
P∞
Since the limit exists and since 0 < L = 1 < ∞, and since n=1 −1/n
diverges (as it is a constant multiple of
Pthe harmonic series), the limit

comparison test yields that the series n=1 −n/(n2 + 1) diverges.
(51) converges conditionally: we start by noting that cos(nπ) = (−1)n ,
and so this is an alternating series. So, we first Pcheck for absolute con-

vergence,
P∞ namely the convergence of the series n=1 |100 cos(nπ)/(2n+
3)| = n=1 100/(2n + 3). Here, there are many tests that yield di-
vergence, for instance
P∞ we may use the limit comparison test with the
harmonic series n=1 n1 :
100/(2n + 3) 100n
lim = lim = 50 = L;
n→∞ 1/n n→∞ 2n + 3
MATH2039 ANALYSIS 143

since this limit exists and satisfies 0 < L = 50 < ∞, and since
the
P∞ harmonic series diverges, the limit comparison test yields that
n=1 100/(2n + 3) diverges.

100 100 100 100


However, since 2(n+1)+3 = 2n+5 < 2n+3 and since limn→∞ 2n+3 =
P∞
0, the alternating series test yields that n=1 100 cos(nπ)/(2n + 3)
converges.

Hence, this series converges but does not converge absolutely. That is,
the series converges conditionally.

(52) converges conditionally: as before, we begin by simplifying the ex-


pression of each term. Here, note that sin((n + 1/2)π) = (−1)n , and
so this is an alternating series. As always, we first P check for abso-

lute convergence, namely
P∞ the convergence of the series n=10 | sin((n +
1/2)π)/ ln(ln(n))| = n=10 1/ ln(ln(n)). Since n > ln(ln(n)) for all
n ≥ 10,P∞ we have that 1/ ln(ln(n)) > 1/n for all n ≥ 10, and so the
series n=10 1/ ln(ln(n)) diverges by the comparison test. That is, the
original series does not converge absolutely.

We are now ready to determine convergence of the original series. As


this is an alternating series, let’s check whether the hypotheses of the
alternating series test are satisfied. Since 1/ ln(ln(n)) > 1/ ln(ln(n + 1))
and since limn→∞ 1/ ln(ln(n)) = 0 (since limn→∞ ln(ln(n)) = ∞), the
alternating
P∞ series test applies to this series, and yields that the series
n=10 sin((n + 1/2)π)/ ln(ln(n)) converges.

Hence, this series converges but does not converge absolutely. That is,
the series converges conditionally.

(53) diverges: similar to the algebraic manipulation we performed on the


series whose terms were the reciprocals of the terms in this series, we
calculate:
(2n)! (2n)!
=
22n (n!)2 (2n n!)2
(2n) · (2n − 1) · (2n − 2) · (2n − 3) · · · 2 · 1
=
(2n) · (2n) · (2n − 2) · (2n − 2) · · · 2 · 2
(2n − 1) · (2n − 3) · · · 3 · 1
=
(2n) · (2n − 2) · · · 4 · 2
1 2n − 1 2n − 3 53 1
= ··· > .
2n 2n − 2 2n − 4 42 2n
P∞ 1
Hence, since the series n=1 2n diverges (as it is a constant multiple
of
P∞ the harmonic series), the first comparison test yields that
2n 2
n=1 (2n)!/(2 (n!) ) diverges.
144 PERIŠIĆ

(54) converges absolutely: since each term is a power, we first attempt


to apply the root test, and so we calculate:
" n2 #1/n  n  −n
n n n+1
lim = lim = lim
n→∞ n+1 n→∞ n + 1 n→∞ n
1 1
= n = = L.
limn→∞ 1 + n1 e
Since the limit exists and since L < 1, the root test yields that
P∞ n2
n=1 (n/(n + 1)) converges.
(55) converges absolutely: we begin with a bit of algebraic manipulation,
namely noting that
n(n + 1)
1 + 2 + ··· + n =
2
for n ≥ 1, and so
1 2 2
= < 2
1 + 2 + ··· + n n(n + 1) n
P∞ 2
for n ≥ 1. Since n=1 P∞1/n converges, by Example 10.7, the compari-
son test yields that n=1 1/(1 + 2 + · · · + n) converges.
(56) converges absolutely: we begin with a bit of simplification, namely
noting that
ln(n) n n 1
0≤ ≤ 3 ≤ 3 = 2
2n3 − 1 2n − 1 n n
for n ≥ 1. (The first inequality follows since ln(n) ≤ n for n ≥ 1, while
the second inequality follows since 2n3 − 1 ≥ n3 for n ≥ 1.) Since
P ∞ 2
P∞ n=1 1/n converges by Example 10.7, the comparison test yields that
3
n=1 ln(n)/(2n − 1) converges.

(57) converges absolutely: note that this is not an alternating series,


even though the terms are not all of the same sign (since sin(n) be-
haves a bit strangely). However, we still begin testing for convergence
by testing
P∞ for absolute convergence, namely the convergence of the se-
2
ries
P∞ n=12 | sin(n)/n |. Since | sin(n)| ≤ 1 for all n ≥ 1, and since
Pn=1 1/n converges by Example 10.7, the comparison test yields that
∞ 2
n=1 sin(n)/n converges absolutely.
(58) diverges: since limn→∞ (n−1)/n = 1, we have that limn→∞ (−1)n (n−
1)/n does not exist (since for large n, it is oscillating between numbers
near 1 and numbers near −1). SincePthis limit does not exist, the nth

term test for divergence yields that n=1 (−1)n (n − 1)/n diverges.
(59) diverges: we can rewrite this series as a geometric series, to whit:
∞ ∞ ∞  n
X (−1)n 23n X (−8)n X −8
= = .
n=1
7n n=1
7n n=1
7

Since | − 78 | ≥ 1, this is a divergent geometric series.


MATH2039 ANALYSIS 145

(60) converges absolutely: this is similar to a series we handled a few


problems ago. Even though the terms are not of the same sign and
are not of alternating signs, we still begin our check for convergence
by checking for absolute convergence. P Since | cos(n)/n4 | ≤ 1/n4 (since

| cos(n)| ≤ 1 for all n ≥P1) and since n=1 1/n4 converges, the com-

parison test yields that n=1 cos(n)/n4 converges absolutely.

(61) diverges: even though this is an alternating series, I personally feel


the need to try the nth term test first, since for n large, the dominant
terms are the 3n in the numerator and the 2n in the denominator, and
so I expect that the value of 3n /(n(2n + 1)) to be large for large values
of n. Let’s check this:
 n
3n 3n 3n 3n 3 1
n
= n
> n n
= n
= .
n(2 + 1) n2 +n n2 +n2 2n 2 2 2n

Now, notice that (3/2)n > n for n ≥ 3 (since (3/2)3 > 3 and the
derivative of (3/2)n − n is positive for n ≥ 3), and so
 n
3n 3 1 1
> >
n(2n + 1) 2 2n 2

for n ≥ 3. (So, not exactly large for large values of n, but big enough
to do the trick.) Hence, the limit limn→∞ (−1)n 3n /(n(2n + 1)) does not
exist (as it oscillates positive and negative and
P∞ never settles down to 0),
and so by the nth term test for divergence, n=1 (−1)n 3n /(n(2n + 1))
diverges.

(62) converges conditionally: we first check for absolute convergence,


namely
P∞ the convergence of the P
series
n−1 ∞
n=1 |(−1) n/(n2 + 1)| = 2
n=1 n/(n +P1). Since n/(n2 + 1) >
2 2 ∞
n/(n + n ) = 1/(2n) for all n ≥ 1 and since n=1 1/(2n) diverges (as
it is a constant
P∞ multiple of the harmonic series), the comparison test
yields that n=1 n/(n2 + 1) diverges, and so the original series does
not converge absolutely.

As it is an alternating series, we can attempt to check convergence by


seeing if we can apply the alternating series test. Since limn→∞ n/(n2 +
1) = 0 and since n/(n2 +1) > (n+1)/((n+1)2 +1) for Pall n ≥ 1, the hy-

potheses of the alternating series test are met, and so n=1 (−1)n−1 n/(n2 +
1) converges.

Hence, this series converges but does not converge absolutely. That is,
the series converges conditionally.

(63) converges absolutely: we first check absolute convergence, namely


the
P∞convergence of the series P

n=2 |(−1) n−1
/(n ln2 (n))| = 2
n=2 1/(n ln (n)). For this series, we
2
use the integral test: set f (x) = 1/(x ln (x)). We need to check that
146 PERIŠIĆ

f (x) is decreasing, which we do by calculating its derivative:


−(ln2 (x) + 2 ln(x))
f 0 (x) = <0
x2 ln4 (x)
for x ≥ 2 (since ln(x) > 0 for x ≥ 2). We now calculate:
Z ∞ Z M
1
f (x)dx = lim dx
2 M →∞ 2 x ln2 (x)
−1 M
= lim 2
M →∞ ln(x)
 
−1 1 1
= lim + = .
M →∞ ln(M ) ln(2) ln(2)
Since
P∞ this limit exists, the integral test yields that the series
n=2 1/(n ln2 (n)) converges, and hence that the original series
P∞
n=2 (−1)
n−1
/(n ln2 (n)) converges absolutely.
(64) diverges: we apply the ratio test (Proposition 10.22), as this is a series
with non-zero terms:
(−1)n 2(n+1) /(n + 1)2 2n2

lim n−1 n 2
= lim = 2 = L.
n→∞ (−1) 2 /n n→∞ (n + 1)2
P∞
Since this limit exists and satisfies L > 1, the series n=1 (−1)n−1 2n /n2
diverges.
(65) converges absolutely: we first check for absolute convergence, namely
the
P∞convergence of
√ the series P∞ √
n
n=1 |(−1) sin( n)/n3/2 | = n=1 | sin( n)|/n3/2 .
√ 3/2

Since | sin( n)|/nP ≤ 1/n3/2 for n ≥ 1 (since | sin( n)| ≤ 1 for

n ≥ 1), and since n=1 1/n3/2 converges by Example 10.7, the second
P∞ √
comparison test yields that n=1 | sin( n)|/n3/2 converges, and hence
P∞ √
that the original series n=1 (−1)n sin( n)/n3/2 converges absolutely.
(66) converges absolutely: even though there are no factorials, let us
apply the ratio test. So, we calculate:
2 4
(n + 1)4 e−(n+1)

n+1
lim = lim e−2n−1 = 0 = L.
n→∞ n4 e−n2 n→∞ n
Since this limit exists and since L < 1, the ratio test yields that the
P∞ 2
series n=1 n4 e−n converges.
(67) converges conditionally: before testing for absolute convergence, we
perform a bit of algebraic simplification, by noting that
 nπ   
2kπ
sin = sin = sin(kπ) = 0
2 2
for n even and
 πn   
π(2k + 1)  π
sin = sin = sin kπ + = (−1)k
2 2 2
MATH2039 ANALYSIS 147

for n = 2k + 1 odd. Hence, setting n = 2k + 1 for k ≥ 0, we may rewrite


the series as
∞ ∞ ∞
X sin(nπ/2) X sin(π(2k + 1)/2) X (−1)k
= = .
n=1
n 2k + 1 2k + 1
k=0 k=0

We first
P∞ test for absolute convergence,
P∞ namely the convergence of the
series k=0 |(−1)k /(2k+1)| = k=0 1/(2k+1).
P∞ However, since 1/(2k+
1) > 1/(2k + 2) =P1/2(k + 1) and since k=0 1/(k + 1) is the harmonic

series, the series k=0 1/(2k + 1) diverges by the first comparison test,
and hence the original series does not converge absolutely.

To test convergence, we use the alternating series test. Since 1/(2k +


1) > 1/(2(k + 1) + 1) for all k ≥ 0 andP∞since limk→∞ 1/(2k + 1) = 0,
the alternating series test yields that k=0 (−1)k /(2k + 1) converges.

Hence, this series converges but does not converge absolutely. That is,
the series converges conditionally.
(68) diverges: for this series, we first note that ln(x) < x1/8 for x large
(x > e32 works), as follows: consider the function f (x) = x1/8 − ln(x),
and note that
f (e8k ) = (e8k )1/8 − ln(e8k ) = ek − 8k,
and so f (e32 ) = e4 − 32 = 22.5982... > 0.

Moreover, for x ≥ e32 , we have that f (x) is increasing: differentiating,


we see that
 
1 1 1 1
f 0 (x) = x−7/8 − = x−1 ,
8 x x 8
and so f 0 (x) > 0 for x > 8.

So, for n > e32 , we have that


1 1 1
8
> 1/8 8 = ,
ln(n) (n ) n
P∞ 8
and hence by the comparison test, n=2 1/(ln(n)) diverges. (Note
that we are making heavy use of Fact 10.4, that ignoring finitely many
terms of a series does not affect its convergence or divergence.)
(69) converges absolutely: (note that the lower limit 13 for the series
yields that ln(n) and ln(ln(n)) are positive for all terms in the series.)
We apply the integral test, using the function
1
f (x) = .
x ln(x)(ln(ln(x)))p
We first check that f (x) is decreasing:
−(ln(x) ln(ln(x))p + ln(ln(x))p + p)
f 0 (x) = <0
(x ln(x) ln(ln(x))p )2
148 PERIŠIĆ

for x > 13, since both ln(x) > 0 and ln(ln(x)) > 0 for x > 13 and since
p > 0 by assumption.

In order to apply the integral test, we now need to calculate:


Z ∞ Z M
1
f (x)dx = lim .
13 M →∞ 13 x ln(x)(ln(ln(x)))p

There are two cases: if p = 1, we get


Z M
1
lim ln(ln(ln(x))) M

lim = 13
M →∞ 13 x ln(x)(ln(ln(x)) M →∞

= lim (ln(ln(ln(M ))) − ln(ln(ln(13)))) = ∞,


M →∞
and so for p = 1 the series diverges.

For p 6= 1, we get:
Z M
1 1 1 M
limM →∞ p
= lim p−1
13
13 x ln(x)(ln(ln(x)) M →∞ −p + 1 ln(ln(x))
1
lim ln(ln(M ))−p+1 − ln(ln(13))−p+1 ,

=
−p + 1 M →∞
which converges for p > 1 (since −p + 1P< 0)and diverges for p < 1
∞ p
(since −p + 1 > 0). Hence, the series n=13 1/(n ln(n)(ln(ln(n))) )
converges if and only if p > 1. (Note that this is really just Example
10.7 in a bit of disguise.)
th
Solution 10.27: P∞By the contrapositive to the n term test for divergence,
since the series n=1 an converges, we have that limn→∞ an = 0. In particular,
taking ε = 1 and remembering that each an > 0, there exists M so that
0 < an < 1 for all n > M . Since 0 < an < 1 for n > M and since s ≥ 1,
s
P∞that ans < an for n > M , and so byPthe
we have comparison test,
P∞we have

that n=M +1 an converges by comparison to n=M +1 an . Since n=M +1 asn
P∞
converges, we see that n=0 asn converges, as desired.
MATH2039 ANALYSIS 149

Solutions to Exercises, Chapter 12

Solution 12.5:
(1) radius of convergence is ∞, interval of convergence is R: Apply
the ratio test and calculate:
(−1)n+1 xn+1 /(n + 1)!

= |x| lim 1
lim
n n
= 0.
n→∞ (−1) x /n! n→∞ n+1
Hence, this series converges absolutely for all values of x (since this
limit is 0 for every value of x).
(2) radius of convergence is 51 , interval of convergence is − 15 , 15 :
 

Apply the ratio test and calculate:


n+1 n+1
/(n + 1)2 5n2

5 x
lim n n 2
= |x| lim = 5|x|.
n→∞ 5 x /n n→∞ (n + 1)2

Hence, this series converges absolutely for 5|x| < 1, that is for |x| < 15 ,
and so the radius of convergence is 15 . We now need to check the
endpoints of the interval (− 15 , 15 ):
P∞ P∞
At x = − 51 , the series becomes n=1 5n (−1/5)n /n2 = n=1 (−1)
n
/n2 ,
which converges absolutely.
P∞ P∞
At x = 51 , the series becomes n=1 5n (1/5)n /n2 = n=1 1/n2 , which
converges absolutely.

So the series converges absolutely for all x in the closed interval − 51 , 15 ,


 

and diverges elsewhere.


(3) radius of convergence is 1, interval of convergence is [−1, 1]:
Apply the ratio test and calculate:
n+1
x /((n + 1)(n + 2)) n
lim
n→∞ xn /(n(n + 1)) = |x| n→∞
lim
n+2
= |x|.

Hence, this series converges absolutely for |x| < 1, and so the radius of
convergence is 1. We now need to check the endpoints of the interval
(−1, 1):
P∞ n
At x = −1, the series becomes n=1 (−1) /(n(n + 1)), which converges
absolutely.
P∞
At x = 1, the series becomes n=1 1/(n(n + 1)), which converges ab-
solutely.

So, the series converges absolutely for all x in the closed interval [−1, 1],
and diverges elsewhere.
150 PERIŠIĆ

(4) radius of convergence is 1, interval of convergence is [−1, 1):


Apply the ratio test and calculate:

(−1)n+1 xn+1 / n + 1
r
n
lim √ = |x| n→∞
lim = |x|.
n→∞ (−1)n xn / n n+1
Hence, this series converges absolutely for |x| < 1, and so the radius of
convergence is 1. We now need to check the endpoints of the interval
(−1, 1):
P∞ √
At x = −1, the series becomes n=1 (−1)n / n, which converges condi-
tionally. (The alternating series test yields convergence, but this series
does not converge absolutely, by comparison to the harmonic series.)
P∞ √
At x = 1, the series becomes n=1 1/ n, which diverges.

So, the series converges absolutely for all x in the open interval (−1, 1),
converges conditionally at x = −1, and diverges elsewhere.
(5) radius of convergence is ∞, interval of convergence is R: Apply
the ratio test and calculate:
(−1)n+1 x2(n+1)+1 /(2(n + 1) + 1)!

lim = |x|2 lim (2n + 1)!
n→∞ (−1)n x2n+1 /(2n + 1)! n→∞ (2n + 3)!
1
= |x|2 lim = 0.
n→∞ (2n + 3)(2n + 2)
Hence, this series converges absolutely for all values of x (since this
limit is 0 for every value of x).
(6) radius of convergence is ∞, interval of convergence is R: Apply
the ratio test and calculate:
n+1 n+1
3 x /(n + 1)! 3
lim = |x| n→∞
lim = 0.

n→∞ 3n xn /n! n+1
Hence, this series converges absolutely for all values of x (since this
limit is 0 for every value of x).
(7) radius of convergence is 1, interval of convergence is [−1, 1]:
Apply the ratio test and calculate:
n+1
/(1 + (n + 1)2 ) 1 + n2

x
lim
= |x| lim = |x|.
n→∞ xn /(1 + n2 ) n→∞ 2 + 2n + n2

Hence, this series converges absolutely for |x| < 1, and so the radius of
convergence is 1. We now need to check the endpoints of the interval
(−1, 1):
P∞ n
At x = −1, the series becomes n=0 (−1) /(1 + n2 ), which converges
absolutely.
P∞
At x = 1, the series becomes n=0 1/(1 + n2 ), which converges abso-
lutely.
MATH2039 ANALYSIS 151

So, the series converges absolutely for all x in the closed interval [−1, 1],
and diverges elsewhere.
(8) radius of convergence is 1, interval of convergence is (−2, 0]:
Apply the ratio test and calculate:
(−1)(n+1)+1 (x + 1)n+1 /(n + 1)

lim
= |x + 1| lim n = |x + 1|.
n→∞ (−1) n+1 n
(x + 1) /n n→∞ n + 1

Hence, this series converges absolutely for |x + 1| < 1, and so the radius
of convergence is 1. We now need to check the endpoints of the interval
(−2, 0):
P∞ P∞
At x = −2, the series becomes n=1 (−1)n+1 (−1)n /n = − n=1 1/n,
which diverges, being a constant multiple of the harmonic series.
P∞
At x = 0, the series becomes n=1 (−1)n+1 /n, which converges condi-
tionally, as it is the alternating harmonic series.

So, the series converges absolutely for all x in the open interval (−2, 0),
converges conditionally at x = 0, and diverges elsewhere.
(9) radius of convergence is 43 , interval of convergence is (− 19 11
3 , − 3 ):
Apply the ratio test and calculate:
n+1
(x + 5)n+1 /4n+1 3

3
lim
n→∞ 3n (x + 5)n /4n = 4 |x + 5|.

Hence, this series converges absolutely for 43 |x + 5| < 1, that is for


|x + 5| < 43 , and so the radius of convergence is 43 . We now need to
check the endpoints of the interval (− 19 11
3 , − 3 ).

At x = − 19
3 , the series becomes
∞ n ∞
X 3n − 19
3 +5
X
n
= (−1)n ,
n=0
4 n=0

which diverges (being, for instance, a divergent geometric series).

At x = − 11
3 , the series becomes
∞ n ∞
X 3n − 11
3 +5
X
n
= 1,
n=0
4 n=0

which diverges (again being, for instance, a divergent geometric series).

So, the series converges absolutely for all x in the open interval (− 19 11
3 , − 3 ),
and diverges elsewhere.
(10) radius of convergence is 1, interval of convergence is [−2, 0]:
Apply the ratio test and calculate:
(−1)n+1 (x + 1)2(n+1)+1 /((n + 1)2 + 4) n2 + 4

lim
= |x+1|2 lim = |x+1|2 .
n→∞ (−1)n (x + 1)2n+1 /(n2 + 4) n→∞ n2 + 2n + 5
152 PERIŠIĆ

Hence, this series converges absolutely for |x + 1|2 < 1, that is for
|x + 1| < 1, and so the radius of convergence is 1. We now need to
check the endpoints of the interval (−2, 0).

P∞x = −2,n the series


At
2n+1
becomes
2
P∞
n=1 (−1) (−1) /(n + 4) = n=1 (−1)n+1 /(n2 + 4), which con-
verges absolutely.
P∞ n 2
At x = 0, the series becomes n=1 (−1) /(n + 4), again which con-
verges absolutely.

So, the series converges absolutely for all x in the closed interval [−2, 0],
and diverges elsewhere.
(11) radius of convergence is ∞, interval of convergence is R: Apply
the ratio test and calculate:
n+1
(x − 1)2(n+1) /(2(n + 1) + 1)!

π π
lim
n 2n
= |x − 1|2 lim = 0.
n→∞ π (x − 1) /(2n + 1)! n→∞ (2n + 2)(2n + 3)
Hence, this series converges absolutely for all values of x (since this
limit is 0 for every value of x).
(12) radius of convergence is ∞, interval of convergence is R: This
time, since the coefficients are nth powers, we apply the root test and
calculate:
xn 1/n

1
lim = |x| lim = 0.
n→∞ (ln(n))n n→∞ ln(n)

Hence, this series converges absolutely for all values of x (since this
limit is 0 for every value of x).
(13) radius of convergence is 13 , interval of convergence is (− 31 , 13 ):
Apply the ratio test and calculate:
n+1 n+1
3 x
lim = 3|x|.
n→∞ 3n xn
Hence, this series converges absolutely for 3|x| < 1, that is |x| < 13 , and
so the radius of convergence is 13 . We now need to check the endpoints
of the interval (− 13 , 13 ).

At x = − 13 , the series becomes


∞  n X ∞
X
n 1
3 − = (−1)n ,
n=0
3 n=0

which diverges (being, for instance, a divergent geometric series).

At x = 13 , the series becomes


∞  n X ∞
X 1
3n = 1,
n=0
3 n=0
MATH2039 ANALYSIS 153

which diverges (again being, for instance, a divergent geometric series).

So, the series converges absolutely for all x in the open interval (− 31 , 13 ),
and diverges elsewhere.
(14) radius of convergence is 0, interval of convergence is {0}: Apply
the ratio test and calculate:
(n + 1)!xn+1 /2n+1

lim = |x| lim n + 1 = ∞.
n→∞ n!xn /2n n→∞ 2
Hence, this series converges only for x = 0 and diverges elsewhere.
(15) radius of convergence is 21 , interval of convergence is (− 21 , 12 ]:
Apply the ratio test and calculate:
(−2)n+1 x(n+1)+1 /((n + 1) + 1)

lim = |x| 2(n + 1) = 2|x|.
n→∞ (−2)n xn+1 /(n + 1) n+2
Hence, this series converges absolutely for 2|x| < 1, that is |x| < 21 , and
so the radius of convergence is 12 . We now need to check the endpoints
of the interval (− 21 , 12 ).

At x = − 12 , the series becomes


∞ n+1 ∞
X (−2)n − 12 1X 1
=− ,
n=1
n+1 2 n=1 n + 1
which diverges, as it is a constant multiple of the harmonic series.

At x = 12 , the series becomes


∞ n+1 ∞
X (−2)n 21 1 X (−1)n
= ,
n=1
n+1 2 n=1 n + 1
which converges, as it is a constant multiple of the alternating harmonic
series.

So, the series converges absolutely for all x in the open interval (− 21 , 12 ),
converges conditionally at x = 21 , and diverges elsewhere.
(16) radius of convergence is ∞, interval of convergence is R: Apply
the ratio test and calculate:
(−1)n+1 x2(n+1) /(2(n + 1))!

= |x|2 lim 1
lim = 0.
n→∞ (−1)n x2n /(2n)! n→∞ (2n + 2)(2n + 1)

Hence, this series converges absolutely for all values of x (since this
limit is 0 for every value of x).
(17) radius of convergence is 1, interval of convergence is [−1, 1]:
Apply the ratio test and calculate:
(−1)n+1 x3(n+1) /(n + 1)3/2 n3/2

lim
= |x|3 lim = |x|3 .
n→∞ (−1)n x3n /n3/2 n→∞ (n + 1)3/2
154 PERIŠIĆ

Hence, this series converges absolutely for |x|3 < 1, that is |x| < 1, and
so the radius of convergence is 1. We now need to check the endpoints
of the interval (−1, 1).
P∞ P∞
At x = −1, the series becomes n=1 (−1)n (−1)n /n3/2 = n=1 1/n3/2 ,
which converges, by Example 10.7.
P∞ n
At x = 1, the series becomes n=1 (−1) /n3/2 , which converges abso-
lutely, by Example 10.7.

So, the series converges absolutely for all x in the closed interval [−1, 1],
and diverges elsewhere.
(18) radius of convergence is 1, interval of convergence is [−1, 1]:
Apply the ratio test and calculate:
(−1)(n+1)+1 xn+1 /((n + 1) ln2 (n + 1)) n ln2 (n)

lim
= |x| lim = |x|.
n→∞ (−1)n+1 xn /(n ln2 (n)) n→∞ (n + 1) ln2 (n + 1)

Hence, this series converges absolutely for |x| < 1, and so the radius of
convergence is 1. We now need to check the endpoints of the interval
(−1, 1).

P∞x = −1,n+1
At the series becomes P∞
n=2 (−1) (−1)n /(n ln2 (n)) = − n=2 1/(n ln2 (n)), which converges
by the integral test: take f (x) = 1/(x ln2 (x)). Then,
−(ln2 (x) + 2 ln(x))
f 0 (x) = <0
x2 ln4 (x)
for x ≥ 2, and so f (x) is decreasing. Then, we evaluate
Z ∞ Z M
1
f (x)dx = lim 2 dx
2 M →∞ 2 x ln (x)
−1 M
= lim 2
M →∞ ln(x)
 
−1 1 1
= lim + = ,
M →∞ ln(M ) ln(2) ln(2)
which converges. Hence, by the integral test, the series converges.
P∞
At x = 1, the series becomes n=2 (−1)n+1 /(n ln2 (n)), which converges
absolutely by the argument just given.

So, the series converges absolutely for all x in the closed interval [−1, 1],
and diverges elsewhere.
(19) radius of convergence is 2, interval of convergence is (1, 5):
Apply the ratio test and calculate:
(x − 3)n+1 /2n+1 1

lim = |x − 3|.
n→∞ (x − 3)n /2n 2
MATH2039 ANALYSIS 155

Hence, this series converges absolutely for 12 |x − 3| < 1, that is |x − 3| <


2, and so the radius of convergence is 2. We now need to check the
endpoints of the interval (1, 5).
P∞ n n
P∞ n
At x = 1, the series becomes n=0 (−2) /2 = n=0 (−1) , which
diverges, being for instance a divergent geometric series.
P∞
At x = 5, the series becomes n=0 1, which diverges, again being for
instance a divergent geometric series.

So, the series converges absolutely for all x in the open interval (1, 5),
and diverges elsewhere.
(20) radius of convergence is 1, interval of convergence is [3, 5]: Ap-
ply the ratio test and calculate:
(−1)n+1 (x − 4)n+1 /((n + 1) + 1)2 2

lim
= |x − 4| lim (n + 1) = |x − 4|.
n→∞ (−1)n (x − 4)n /(n + 1)2 n→∞ (n + 2)2

Hence, this series converges absolutely for |x − 4| < 1, and so the radius
of convergence is 1. We now need to check the endpoints of the interval
(3, 5).
P∞ P∞
At x = 3, the series becomes n=1 (−1)n (−1)n /(n+1)2 = n=1 1/(n+
1)2 , which converges by Example 10.7.
P∞
At x = 5, the series becomes n=1 (−1)n /(n + 1)2 , which converges
absolutely, again by Example 10.7.

So, the series converges absolutely for all x in the closed interval [3, 5],
and diverges elsewhere.
(21) radius of convergence is 0, interval of convergence is {2}: Apply
the ratio test and calculate:
(2(n + 1) + 1)! (x − 2)n+1 /(n + 1)3 (2n + 3)! n3

lim
= |x−2| lim =∞
n→∞ (2n + 1)! (x − 2)n /n3 n→∞ (2n + 1)! (n + 1)3

for all x 6= 2. Hence, the series converges only for x = 2.


(22) radius of convergence is 1, interval of convergence is [2, 4):
Apply the ratio test and calculate:
ln(n + 1)(x − 3)n+1 /(n + 1)

lim = |x − 3| n ln(n + 1) = |x − 3|.
n→∞ ln(n)(x − 3)n /n (n + 1) ln(n)
Hence, this series converges absolutely for |x − 3| < 1, and so the radius
of convergence is 1. We now need to check the endpoints of the interval
(2, 4).
P∞
At x = 2, the series becomes n=1 ln(n)(−1)n /n, which converges by
the alternating series test (but does not converge absolutely).
156 PERIŠIĆ

P∞
At x = 4, the series becomes n=1 ln(n)/n, which diverges by the
comparison
P∞ test, since ln(n)/n > 1/n for n ≥ 3 and the harmonic series
n=1 1/n diverges.

So, the series converges absolutely for all x in the open interval (2, 4),
converges conditionally at x = 2, and diverges elsewhere.
(23) radius of convergence is 8, interval of convergence is (− 13 19
2 , 2 ):
Apply the ratio test and calculate:
(2x − 3)n+1 /42(n+1)

1 1 3
lim = |2x − 3| = x − .
n→∞ (2x − 3)n /42n 16 8 2

1

3
Hence, this series converges absolutely for 8 x − 2 < 1, that is for
x − 3 < 8, and so the radius of convergence is 8. We now need to
2
check the endpoints of the interval (− 13 19
2 , 2 ).
P∞ P∞
At x = − 13
2 , the series becomes
n
n=0 (2(−13/2)−3) /4
2n
= n=0 (−1)
n
,
which diverges.
P∞ P∞
At x = 192 , the series becomes n=0 (2(19/2) − 3)n /42n = n=0 1,
which diverges.

So, the series converges absolutely for all x in the open interval (− 13 19
2 , 2 ),
and diverges elsewhere.
(24) radius of convergence is b, interval of convergence is (a−b, a+b):
Apply the ratio test and calculate:
(x − a)n+1 /bn+1 1

lim = |x − a|.
n→∞ (x − a)n /bn b
Hence, this series converges absolutely for 1b |x − a| < 1, that is for
|x − a| < b, and so the radius of convergence is b. We now need to check
the endpoints of the interval (a − b, a + b).
P∞ P∞
At x = a − b, the series becomes n=2 (a − b − a)n /bn = n=2 (−1)
n
,
which diverges.
P∞ n n
P∞
At x = a + b, the series becomes n=2 (a + b − a) /b = n=2 1, which
diverges.

So, the series converges absolutely for all x in the open interval (a −
b, a + b), and diverges elsewhere. (Note that the previous series is a
specific example of this general phenomenon, with a = 23 and b = 8.)
(25) radius of convergence is ∞, interval of convergence is R: Apply
the ratio test and calculate:
((n + 1) + p)!xn+1 /((n + 1)!((n + 1) + q)!)

limn→∞
(n + p)!xn /(n!(n + q)!)
n+1+p
= |x| lim = 0.
n→∞ (n + 1)(n + 1 + q)
MATH2039 ANALYSIS 157

Hence, this series converges absolutely for all values of x (since this
limit is 0 for every value of x).
(26) radius of convergence is 3, interval of convergence is [−3, 3):
Apply the ratio test and calculate:
(n+1)−1
/((n + 1)3n+1 )

x n 1
lim
n−1 n
= |x| lim = |x|.
n→∞ x /(n3 ) n→∞ 3(n + 1) 3
Hence, this series converges absolutely for 13 |x| < 1, that is for |x| <
3, and so the radius of convergence is 3. We now need to check the
endpoints of the interval (−3, 3).
P∞ P∞ n−1
At x = −3, the series becomes n=1 (−3)n−1 /(n3n ) = 31 n=1 (−1)n ,
which converges conditionally, as it is a constant multiple of the alter-
nating harmonic series.
P∞ P∞
At x = 3, the series becomes n=1 3n−1 /(n3n ) = 13 n=1 n1 , which
diverges, as it is a constant multiple of the harmonic series.

So, the series converges absolutely for all x in the open interval (−3, 3),
converges conditionally at x = −3, and diverges elsewhere.
(27) radius of convergence is ∞, interval of convergence is R: Apply
the ratio test and calculate:
(−1)(n+1)−1 x2(n+1)−1 /(2(n + 1) − 1)!

= |x|2 lim 1
lim
n−1 2n−1
= 0.
n→∞ (−1) x /(2n − 1)! n→∞ 2n(2n + 1)
Hence, this series converges absolutely for all values of x (since this
limit is 0 for every value of x).
(28) radius of convergence is 0, interval of convergence is {a}: Apply
the ratio test and calculate:
(n + 1)!(x − a)n+1

lim = |x − a| lim (n + 1) = ∞
n→∞ n!(x − a)n n→∞

for all x 6= a. Hence, the series converges only for x = a.


(29) radius of convergence is 2, interval of convergence is (−1, 3):
Apply the ratio test and calculate:
(n + 1)(x − 1)n+1 /(2n+1 (3(n + 1) − 1))

limn→∞
n(x − 1)n /(2n (3n − 1))
(n + 1)(3n − 1) 1
= |x − 1| lim = |x − 1|.
n→∞ 2n(3n + 2) 2
Hence, this series converges absolutely for 21 |x − 1| < 1, that is for
|x − 1| < 2, and so the radius of convergence is 2. We now need to
check the endpoints of the interval (−1, 3).
P∞ n n
P∞x = −1,
At
n
the series becomes n=1 n(−1 − 1) /(2 (3n − 1)) =
th
n=1 (−1) n/(3n − 1), which diverges by the n n term test for di-
vergence, as limn→∞ 3n−1n
= 13 , and so limn→∞ (−1) n
3n−1 does not exist.
158 PERIŠIĆ

P∞ P∞
At x = 3, the series becomes n=1 n(3−1)n /(2n (3n−1)) = n=1 n/(3n−
1), which again diverges by the nth term test for divergence.

So, the series converges absolutely for all x in the open interval (−1, 3),
and diverges elsewhere.

Solution 12.6: The condition that the an satisfy is similar to the condition
P∞
of the root test, and so we apply the root test to the power series n=0 an xn .
Namely, we calculate
1/n 1/n
lim |an xn | = |x| lim |an | = L|x|.
n→∞ n→∞

Hence, the series converges absolutely for L|x| < 1, that is |x| < L1 , and di-
verges for L|x| > 1, and so the radius of convergence of this series is L1 , as
desired.

Solution 12.7: We can use the same techniques that we have developed for
power series for other series, that are not strictly speaking power series. For
instance, we can apply the ratio test to the series, for all the values of x for
which the terms are defined.
(1) first, we note that this series is not defined at x = 1, but is defined for
all other values of x. Applying the ratio test, we calculate:
((x + 2)/(x − 1))n+1 /(2(n + 1) − 1) |x + 2|

2n − 1 |x + 2|
lim
n
= lim = .
n→∞ ((x + 2)/(x − 1)) /(2n − 1) |x − 1| n→∞ 2n + 1 |x − 1|
|x+2|
Hence, this series converges absolutely for |x−1| < 1, that is for |x+2| <
|x+2|
|x − 1|, which is the open ray (−∞, − 12 ), and diverges for |x−1| > 1,
which is the union (− 12 , 1) ∪ (1, ∞).

At x = − 12 , the only remaining point at which to test for convergence,


the series becomes
∞  1 n X ∞
X 1 −2 + 2 1
1 = (−1)n ,
n=1
2n − 1 − 2 − 1 n=1
2n − 1
which converges conditionally, by the alternating series test. Hence, the
series converges on the closed ray (−∞, − 21 ].
(2) for this series, first note that the series is not defined at x = 0, x = 1,
x = 2, et cetera, and so the domain of consideration is the comple-
ment in R of the non-negative integers W = {0, 1, 2, . . .} (the whole
numbers). Applying the ratio test, we calculate

1/((x + n + 1)(x + n + 1 − 1)) x + n − 1
lim
= n→∞
lim
=1
n→∞ 1/((x + n)(x + n − 1)) x + n + 1
for every (allowable) value of x, and so yields no information. However,
we are saved by the observation that the series

X 1
n=1
(n + α)(n + β)
MATH2039 ANALYSIS 159

P∞ 1
converges for all α, β, by limit comparison to P the series n=1 n2 .

Hence, taking α = x and β = x − 1, we have that n=1 1/((x + n)(x +
n − 1)) converges at every value of x for which it is defined, namely the
union
(−∞, 0) ∪ (0, 1) ∪ (1, 2) ∪ (2, 3) ∪ · · · = R − W.
160 PERIŠIĆ

Solutions to Exercises, Chapter 13

Solution 13.2:
(1) we start by calculating the derivatives of f at a = 6:
f (0) (6) = f (6) = 455; f (1) (6) = f 0 (6) = 185; f (2) (6) = 48;
f (3) (6) = 6; f (n) (6) = 0 for n ≥ 4.
Hence, the Taylor series for f centred at a = 6 is

X 1 (n) 1 1
f (6)(x − 6)n = 455 + 185(x − 6) + 48(x − 6)2 + 6(x − 6)3 .
n=0
n! 2 6
The radius of convergence of this series is ∞ (using the root test, for
instance), and so the interval of convergence is R.
(2) we start by calculating that f (n) (x) = 3n e3x for n ≥ 0, and so f (n) (−2) =
3n e−6 . Hence, the Taylor series for f centred at a = −2 is
∞ ∞
X 1 (n) X 3n
f (−2)(x + 2)n = e−6 (x + 2)n .
n=0
n! n=0
n!
The radius of convergence of this series is ∞ (using the ratio test, for
instance), and so the interval of convergence is R.
(3) we start here by recalling that

(n) cosh(x) for x even, and
f (x) =
sinh(x) for x odd.
So, we have that f (n) (1) = cosh(1) = 12 (e+ 1e ) for n even, and f (n) (1) =
sinh(1) = 12 (e − 1e ) for n odd. Hence, the Taylor series for f centred at
a = 1 is
∞ ∞ ∞
X 1 (n) X 1 (k) X 1 (k)
f (1)(x − 1)n = f (1)(x − 1)k + f (1)(x − 1)k
n=0
n! (k)! (k)!
k even k odd
∞ ∞
X 1 X 1
= f (2k) (1)(x − 1)2k + f (2k+1) (1)(x − 1)2k+1
(2k)! (2k + 1)!
k=0 k=0
∞ ∞
e2 + 1 X 1 e2 − 1 X 1
= (x − 1)2k + (x − 1)2k+1 .
2e (2k)! 2e (2k + 1)!
k=0 k=0
The radius of convergence of this series is ∞ (using the ratio test, for
instance), and so the interval of convergence is R.
MATH2039 ANALYSIS 161

Appendix B: Examination Papers

2013-14 Examination Paper


Duration: 2 hours
Rubric: The exam has 4 questions. Each question is worth 25 marks, for a
total of 100 marks.
Answer ALL questions.

Show your working and justify all steps in your answers.

You MUST write the question number at the top of each page of the answer
booklet.

1. (i) Let A be a bounded non-empty subset of R. Define the supremum


sup(A) and the infimum inf(A) of A.
[ 5 marks]
(ii) Let A be the subset of R defined by
nm o
A= n
| m, n ∈ N .
3
Determine inf(A) and use the definition of the infimum to prove
that your answer is correct.
[ 5 marks]
(iii) Prove the statement: Let the subset A ⊂ R be non-empty and
bounded below. Then for every  > 0 there exists a ∈ A such that
inf(A) ≤ a < inf(A) + .
[ 5 marks]
(iv) Let A be a bounded non-empty subset of R. Show that if sup(A)
exists then
sup(A) = inf{y | y is an upper bound of A}.
[ 5 marks]
(v) Let A be a bounded non-empty subset of R. Define −A = {−a | a ∈
A}.
Show that if sup(A) exists then inf(−A) exists and
inf(−A) = − sup(A).
[ 5 marks]

2. (i) State the definition of the limit L = limn→∞ an of a sequence


(an )n∈N .
[4 marks]
162 PERIŠIĆ

(ii) A sequence (an ) is defined by


1
1 + a2n , for n = 0, 1, 2, . . . .

a0 = 0, an+1 =
2
Prove that this sequence converges and determine its limit.
[6 marks]
(iii) State what it means for a sequence (an ) to be a Cauchy sequence.
[ 4 marks]
(iv) Show that if the sequence (an ) converges then (an ) is a Cauchy
sequence.
[ 6 marks]
(v) Prove that the sequence (an ) defined by an = 2 + (−1)n , n ∈ N,
diverges.
[ 5 marks]
3. (i) State the Intermediate Value Theorem.
[ 4 marks]
(ii) Use the Intermediate Value Theorem to determine whether there
exists a solution to the equation 3 sin2 (x) = 2 cos3 (x) on x > 0.
[ 6 marks]
(iii) Let f be a continuous function
R∞ on [a, ∞). Define what it means for
the improper integral a f (x)dx to converge.
[ 4 marks]
(iv) Show that the improper integral
Z ∞
dx
2 (x − 1)s
converges if and only if s > 1.
[ 6 marks]
(v) Let f (x) be a real-valued function which is differentiable for x > 0
and continuous at x = 0, with f (0) = 0. If f 0 (x) is increasing,
prove that g(x) = f (x)/x is also increasing.
[ 5 marks]
P∞
4. (i) State what it means for the series n=1 an
(a) to converge
(b) to converge absolutely
(c) to converge conditionally.
[6 marks]
MATH2039 ANALYSIS 163

P∞ (−1)n
(ii) Determine whether the series n=1

n
converges absolutely,
converges conditionally or diverges.
[5 marks]
P∞
(iii) Show that if the series n=1 an converges absolutely then it con-
verges.
[5 marks]
(iv) Determine the radius and interval of convergence for the power
series

X n(x − 1)n
.
n=1
2n (3n − 1)

[9 marks]
END OF PAPER

2016-17 Examination Paper


Rubric is unchanged -see 2013-14 exam paper

1. (i) State the definition of the limit L of a sequence (an )n∈N , symbol-
ically written as limn→∞ an = L.
[2 marks]
(ii) State what it means for a sequence (an )n∈N to diverge to infinity,
symbolically written as limn→∞ an = ∞.
[2 marks]
(iii) Show, if the sequence (an )n∈N diverges to infinity, then the se-
quence of reciprocals (bn )n∈N , bn = a1n , converges to 0.
[4 marks]
√ √
(iv) Determine whether the sequence (an )n∈N , an = n + 2 − n,
converges or diverges. If it converges, determine the limit and
using the definition of convergence, show that your limit is correct.
[5 marks]
(v) State what it means for a sequence (an )n∈N to be a Cauchy se-
quence.
[2 marks]
(vi) Show that, if the sequence (an )n∈N converges, then (an )n∈N is a
Cauchy sequence.
[5 marks]
(vii) Show that the sequence (an )n∈N , an = cos( nπ
2 ), diverges.

[5 marks]
164 PERIŠIĆ

2. (i) Let A be a bounded non-empty subset of R. Define the infimum


inf(A) and the supremum sup(A) of A.
[4 marks]
(ii) Let A = {2 − n1 |n ∈ N}. Using the definition of supremum, show
that sup(A) = 2.
[5 marks]
(iii) Let A be a subset of R. Prove that if sup(A) exists, it is unique.
[5 marks]
(iv) Let A and B be bounded non-empty subsets of R such that A∩B 6=
∅. Prove that
sup(A ∩ B) ≤ min{sup(A), sup(B)}.
[5 marks]
(v) Let A be a bounded non-empty subset of R. Let
B = {a + 1|a ∈ A}.
Prove that sup(B) = sup(A) + 1.
[6 marks]
3. (i) State what it means for a function f (x) to be continuous on R.
[3 marks]
(ii) Prove directly from the definition that the function f (x) = 4x − 3
is continuous on R. Is f uniformly continuous on R?
[6 marks]
(iii) Let S ⊆ R and let f, g : S → R be continuous functions at a ∈ S.
Prove that h(x) = f (x) + g(x) is continuous at a.
[6 marks]
(iv) State the Mean Value Theorem.
[4 marks]
(v) Let S = [0, ∞) and let f : S → R be a differentiable function.
Suppose that the limit limx→∞ f 0 (x) exists and limx→∞ f 0 (x) = a.
Show
lim (f (x + 1) − f (x)) = a.
x→∞

[6 marks]
4. (i) Let a > 0 be a real number. Show that the following improper
integrals converge and determine their limits:
R∞
(a) [4 marks] 0 e−ax dx;
R∞
(b) [7 marks] 0 xe−ax dx.
MATH2039 ANALYSIS 165

(ii) Determine whether the series


∞ √
n sin( n)
X
(−1)
n=1
n3/2
converges absolutely, converges conditionally, or diverges. Justify
your answer.
[6 marks]
(iii) Determine the radius and interval of convergence for the power
series
X∞
(−1)n (x + 1)2n+1 .
n=1

[8 marks]
END OF PAPER
Appendix C: Learning Outcomes (LOs)

Upon successful completion of the module, students should be able to:


(LO 1) determine whether a sequence of real numbers converges, either by eval-
uating the limit directly or by showing the sequence is bounded and
monotone;
(LO 2) prove using the definition that a given sequence converges to a given
limit;
(LO 3) determine whether a series of positive terms converges, either by ex-
plicitly summing the series or by using a test, such as the comparison
test, the ratio test, the root test, or the integral test;
(LO 4) define an improper integral;
(LO 5) determine whether a given improper integral converges;
(LO 6) determine a radius and interval of convergence for a given power series;
(LO 7) understand when one can differentiate and integrate power series;
(LO 8) Apply the Weierstrass M-test;
(LO 9) find the Taylor series of a given function and evaluate the error;
(LO 10) determine if the Taylor series of a function converges.

166
Appendix D: Further reading

There are many excellent analysis books around. Here is a selection of books
that you may find helpful:

ADAMS, R. A. , Calculus - A complete course


ALCOCK, L., How to think about analysis
APPELBAUM, D., Limits, Limits Everywhere
BURN, R.P., Numbers and Functions Steps into Analysis, Second edition 2000
BURTON, D. M., The History of Mathematics An Introduction, Seventh edi-
tion 2011 (available online)
HOWLAND, J.S., Basic Real Analysis
NIKOLSKY, S. M., A Course of Mathematical Analysis 1
RUDIN, W., Principle of Mathematical Analysis (or baby Rudin)
SPEIGHT, J.M., A Sequential Introduction to Real Analysis
WADE W. R., An Introduction to Analysis

If you like comics, you may want to read the world’s first calculus comic book:

Prof. E.McSquared’s Calculus Primer: Expanded Intergalactic Version


by Swann Howard

Excellent learning resources covering topics in mathematical analysis are also


free available online. If you find something you really like please do share with
us.

167
Index
k-point refinement, 54 Euler’s constant, 27
exponential function, 90
Riemann integral, 57
Fibonacci sequence, 32
absolute convergence of series, 73 Field F , 7
absolute value, 10 finite sets, 5
addition +, 7 floor function, 26
additive identity, 7 function, 6
additive inverse, 7 functional analysis, 53
alternating harmonic series, 74 Fundamental theorem of calculus,
alternating series test, 68 61
Archimedean property, 12
arithmetic of limits, 20 geometric series, 64
arithmetic of power series, 85 greatest lower bound, 13
arithmetic of sequences, 20 Greek letters, 4
arithmetic of series, 65
associativity, 7 harmonic series, 65

Bernoulli inequality, 28 i-th subinterval of π, 53


bijection, 6 image, 6
binary operation, 7 improper integral, 61
Bolzano-Weierstrass Theorem, 35 improper integral - convergence,
bounded above, 13 61
bounded below, 13 improper integral - divergence,
bounded sequence, 19 61
bounded set, 13 indeterminate form, 50
indexing set, 17
Cantor set, 35 Inequalities of sequences, 27
Cauchy criterion, 24 infimum inf(A), 13
Cauchy Mean Value Theorem, 49 infinite sets, 5
Cauchy sequence, 24 injection, 6
centre of power series, 82 integer part, 26
commutative group, 7 Integers Z, 5
commutativity, 7 integers modulo p, 9
Comparison test, 68 Integral test, 68
complete ordered field, 23 Integration, 53
Completeness Axiom, 14 Intermediate Value Theorem, 42
Completeness properties, 33 intersection, 5
conditional convergence of series, interval of convergence, 82
74
continuous function, 39 k-th partial sum, 64
convergence (of a sequence), 18
convergence (of a series), 64 l’Hopital’s rule, 49
convergence (of an improper integral), least upper bound, 13
61 left-handed limit, 37
less than <, 9
Differentiability, 45 limit, 18
distributive law, 7 Limit comparison test, 68
divergence (of a sequence), 18 limit of a function, 37
divergence (of an improper integral), linear map, 53
61 Lipschitz functions, 39
divergence - n-th term test, 68 lower bound, 13
divergence to infinity, 25 lower Darboux integral, 56
divergence(of a series), 64 lower Darboux sum, 55
domain, 6
maximum, 12
elements, 5 Maximum Value Theorem, 41
168
MATH2039 ANALYSIS 169

Mean Value Theorem (MVT), 46 series of functions - pointwise


method of bisection, 43 convergence, 80
minimum, 12 series of functions - uniform convergence,
Minimum Value Theorem, 42 80
Monotone Property, 16 set, 5
monotone sequence, 23 Squeeze rule, 22
monotonic sequence, 23 squeeze rule, 38
multiplication ·, 7 subfield, 7
multiplicative identity, 7 subsequence, 17
multiplicative inverse, 7 subset, 5
sum, 5
n-th term test for divergence, 68 supremum sup(A), 13
Natural numbers N, 5 surjection, 6
not a Cauchy sequence, 24
Taylor series, 86
onto, 6 Taylor series centred at a, 86
order, 9 Taylor’s theorem, 87
ordered field, 10 transitivity, 9
oscillate, 25 triangle inequality, 11
trichotomy, 9
partition π of an interval [a, b],
53 Uniform continuity, 41
partition points of π, 53 uniform continuity, 40
Power series, 82 union, 5
uniqueness of series representations,
quantifiers, 6 86
upper bound, 13
radius of convergence, 82 upper Darboux integral, 56
range, 6 upper Darboux sum, 55
Ratio test, 68
Rational numbers Q, 5 Weierstrass M -test, 80
Real numbers R, 5
real power series, 82
refinement of π, 54
reverse triangle inequality, 11
Riemann integrable, 57
Riemann integral, 53
right-handed limit, 37
Rolle’s theorem, 45
Root test, 68

sequence, 17
sequence (monotone or monotonic),
23
sequence of functions, 79
sequence of functions - pointwise
convergence, 79
sequence of functions - uniform
convergence, 79
sequence of partial sums, 64
sequentially compact set, 35
series (or infinite series), 64
series - absolutely convergence,
73
series - converges conditionally,
74
series convergence tests, 68
series of functions, 79
Vesna Perišić received her mathematics de-
grees from the University of Zagreb, Croa-
tia. After spending several years at the
University of Saarbrücken, Germany, in De-
cember 2000 she moved with her family to
UK. Vesna has held a teaching position at
the University of Southampton since 2001.

MATH2039 Analysis is a compulsory module for all second year math-


ematics students at the University of Southampton. The module studies
real analysis concepts using mathematical rigor. It builds on the first year
modules, in particular on MATH1059 Calculus.
Mathematical Analysis is a fascinating part of mathematics with impor-
tant applications in areas such as applied mathematics, statistics or op-
erational research. In other words, as the French mathematician Joseph
Fourier (1768-1830) stated:
Mathematical Analysis is as extensive as nature herself.

Das könnte Ihnen auch gefallen